You are on page 1of 164

COVER

FRONT INSIDE
IJCP Group
Daryacha, 39, Hauz Khas Village, New Delhi - 110016, India
Published in India by IJCP Group
© 2010 by IJCP Publications (P) Ltd.
Printed in India
Printed at:
Contents

Allergic Conjunctivitis
1. Diagnosis of Type 2 Diabetes Mellitus -
Simple if we are careful------------------------------------------- 11
—Kathige N Manohar and Arpandev Bhattacharyya

2. Type 2 Diabetes in India- An Increasing Threat----------- 18


—A Ramchandran and C Snehalatha

3. Type 2 Diabetes in the Young -


An Emerging Epidemic! ----------------------------------------- 30
—Geeta K Bhat and Prasanna Kumar

4. Therapeutic LIfestyle----------------------------------------------- 38
—Sonal V Modi and Hemraj B Chandalia

5. Team Work is What we Need------------------------------------ 43


—Rishi Shukla

6. Oral Hypoglycemic Agents - Present Status--------------- 47


—Debashish Maji

7. Insulin Therapy ------------------------------------------------------ 56


—Bipin Sethi

8. Diabetes in Hospital------------------------------------------------ 60
—Menaka Ramprasad and Arpandev Bhattacharyya

9. Hypoglycemia in Type 2 Diabetes Mellitus----------------- 72


—Subranmanian Kannan and Shriraam Mahadevan

10. Hyperglycemic Emergencies Diagnosis


and Management---------------------------------------------------- 82
—Nilanjan Sengupta
Contents
11. Diabetes Mellitus Going throgh Pregnancy---------------- 89
—Auti B Rajendra, Rajendra Pradeepa
and Viswanthan Mohan

12. Diabetic Retinopathy----------------------------------------------- 98


—Chandra Kumar, Sri Ganesh and Gauri Kharosekar

13. Diabetic Neuropathy---------------------------------------------- 106


—Uday Murdgod

14. Sexual Dysfunction in Men and


Women with Diabetes---------------------------------------------114
—Vageesh Ayyar

15. Diabetic Nephropathy in India-------------------------------- 122


—SN Tiwari

16. A Look at the Feet------------------------------------------------- 127


—Vijay Viswanthan

17. Type 2 Diabetes Mellitus: More than Sugar--------------- 133


—Ajay K Ajmani and Sachin K Jain

18. Type 2 Diabetes - Look at the Heart------------------------- 141


—KK Aggarwal

19. Are we Communicating to our


Patient’s Well? Some Issue------------------------------------ 147
—Sri Nagesh Simha

20. Slide Quiz------------------------------------------------------------ 153


—Arpandev Bhattacharyya and Menaka Ramprasad
From the Desk of Group Editor-in-Chief

Dr KK Aggarwal
Dr BC Roy Awardee
Sr. Physician and Cardiologist, Moolchand Medcity
President, Heart Care Foundation of India
Group Editor-in-Chief, IJCP Group
drkk@ijcp.com

American College of Cardiology and American Diabetes Association recommends


that cardiac testing be done irrespective of the persons’ of CAD symptoms in
patients with diabetes. Especially in presence of two or more atherogenic risk
factors. Diabetes today is considered CAD equivalent as mortality and morbidity
in diabetics has the same spectrum as that of a patient who has suffered from
a heart attack in the past.

CAD should be screened amongst diabetics with exercise test, preferably


exercise echocardiography in asymptomatic patients especially if high blood
pressure and smoking are present.

Another important non invasive parameter for diagnosis of coronary artery


disease amongst diabetics is measurement of intima media thickness of carotid
arteries in the neck. The measurement include mean media thickness, maximum
media thickness both in carotid arteries and in the bulb. Comparison is also
done in proximal femoral artery, intima media thickness.

Earliest atherosclerosis is seen in the carotid bulbs and in the femoral arteries.
A thickness of more than 0.7 mm suggests generalized atherosclerosis and a
thickness of more than 1 mm is consistent with established coronary artery
disease with multi vessel involvement.
Other surrogate markers of atherosclerosis in patients with diabetes are
appearance of microalbumin in the urine, retinopathy in the eyes, ankle brachial
index less than one and symptoms suggestive of erectile dysfunction.

As most heart attacks amongst diabetes are silent, blockage needs to be found
out early in the course of the disease so as to prevent sudden cardiac deaths in
diabetic patients. Apart from cardiac involvement diabetic retinopathy, nephropathy
and neuropathy are important preventable complications of diabetes.

Diabetes is now being increasingly noted in the elderly population and


requires a careful analysis and management.

Today diabetes is not just controlling of the sugar but is also keeping the
blood pressure lower than 120/80 and LDL cholesterol lower than 70 mg%.
The management can easily be remembered by the word ‘ABC’ where A stands
for A1c to be kept lower than 6.5% and abdominal circumference to be kept
lower than 90cm and 80 cm in males and females, respectively. B stands for
Blood pressure to be kept lower than 120/80 and C stands for LDL cholesterol
to be kept lower than 70 mg% and saying no to Cigarettes.

This compilation from IJCP Group by Dr. A Bhattacharyya involving top


experts of the country is a maiden attempt of ours in updating the knowledge,
skill and care qualities of our fellow colleagues. I hope this attempt of ours will
serve as a real academic feast.

Long live IJCP.

Regards,

Dr. KK Aggarwal


About the Editor

Dr Arpan, born of Joydev Bhattacharyya and Swaraswati Devi in 1961,


obtained his MBBS in 1986 standing first from North Bengal University in Siliguri,
Darjeeling district. He secured honours in six subjects in the Medical curriculum.
He completed MD in Internal Medicine and DM in Endocrinology from the
Postgraduate Institute, Chandigarh. While in Chandigarh, he also completed his
DNB in Medicine. Dr Arpan then moved for further clinical experience to UK.
There in UK after getting MRCP he completed the five-yr programme named
CCST (Certificate of Completion of Speciality Training) following which he was
awarded dual accreditation in Internal Medicine and Diabetes-Endocrinology.
After a short period of five months working as Consultant Endocrinologist in
Stockport, UK he returned to India.
In India he joined as Consultant Endocrinologist at Manipal Hospital,
Bangalore in August 2001 which is his place of work till date. Dr Arpan has
pioneered the ongoing Fellowship programme in Diabetes and Endocrinology
with Manipal Academy of Higher Education (deemed University) to train post
MD physician in the speciality. He is currently visiting Professor in Diabetes
and Endocrinology in JJMM College, Davangere where he is actively involved
in post-graduate teaching.
Till date he has more than 60 International publications on top of 30
publications in various National Journals. He has keen interest in presentations in
International, national and State conferences. While working at Manipal Hospital
he has produced guidelines/protocols in hospital management of Diabetes,
Diabetes management after CABG/Cardiac Surgery, Diabetes management after
Acute Myocardial Infarction, Endocrine management after Pituitary surgery,
management of thyrotoxicosis, glycaemic management in Pregnancy. Several
ongoing audits are going in the department on various aspects of Diabetes and
Endocrinology to continuously improve the quality of care. He has produced
patient information booklets on Diabetes and Thyroid in English, Hindi, Bengali
and south Indian languages. His keen interest in patient education in the field
of Diabetes and Endocrinology via a dedicated non-commercial website (www.
DiabetesEndocrinology.in) has been well appreciated.
Dr Arpan is married to Dr Shaila, Paediatric Endocrinologist at Manipal Hospital,
Bangalore and they are blessed with daughter Shreyashi and son Amartya.
Foreword

Dr A Bhattacharyya
Editor

Diabetes is a new epidemic of the society. 2.46 crore people in the age group
20-29 have diabetes world wide of which 80% contribution comes from developing
countries. The global prevalence is expected to increase to 38 crores by the year 2025
of which almost seven crore will be from India.

Diabetes is now considered a CAD equivalent, is a multi specialty disease and its
management needs to be known by general practitioners as well as specialist from
every field.

The management of diabetes is changing day by day with more and more emphasis
on diet management of blood sugar with or without insulin.

Aggressive diabetes management has been shown to reduce both mortality and
morbidity.

There is always a need for a diabetes ready reckoner to update the knowledge of
our fellow colleagues. This manual is a compilation of articles of clinical importance
contributed by experts in their respective fields.

I hope this academic feast, yet another from IJCP Group will be of immense help
to all of you in your day today clinical practice. If you have any suggestions kindly
pass it on to us for incorporation in our future issues.

Regards,

Dr. A Bhattacharyya

Editor


Diagnosis of Type 2 Diabetes Kathige N Manohar
Arpandev Bhattacharyya
Mellitus – Simple if we are Careful

Introduction
The use of classification systems and standardized diagnostic criteria
facilitates a common language among patients, physicians and other healthcare
professionals. The diagnostic criteria’s have been evolving mostly based on the
recent evidences about what blood glucose levels are considered safe and at
what level, the diabetes-related complications show a rising trend.
Diabetes being a progressive disease, worsens with time. Most chronic
complications are irreversible, and can be delayed if not completely prevented
by early intervention with optimal therapy. It is important to remember that
the process of diabetes starts much before the diagnosis and is approximately
10 years. Most patients with diabetes may be asymptomatic, detected on routine
screening. Some have non-specific symptoms like tiredness, aches and pains,
headache, blurring of vision, increased susceptibility to infection or delayed wound
healing. The classical presentation of diabetes includes polyurea, polydypsia
and weight loss despite good appetite, the osmotic symptoms. In most severe
cases, patients may present with either acute complications like ketoacidosis, or
non-ketotic hyperosmolar state or chronic complications with renal, neural or
retinal involvement; or may first be diagnosed when the present with myocardial
infarction or stroke.

Clinical Presentations in Type 2 Diabetes1

Diabetes symptoms (53%)


Incidental finding (29%)
Infections, e.g., Candida (16%)
Coplications, e.g., retinopathy (2%)

11
Diagnosis of Type 2 Diabetes Mellitus- Simple if We are Careful

Cases in Context - Diabetes or Not?


Case 1:
Forty-six-year-old female has been admitted for laparoscopic cholecystectomy. Her
random blood sugar done for preoperative fitness was noted to be 210 mg/dl. On
questioning she gives history of polyurea and weight loss.
Case 2:
Sixteen-year-old girl not doing well in school, presented with vomiting. Her urine was
positive for ketones and glucose.
Case 3:
A 36-year-old lady presents with polyurea, tiredness and palpitation. A random blood
sugar was 166 mg/dl and urine sugar shows 2+ glucose.
An oral glucose tolerance test was performed:

Plasma sugar
Time (mg/dl) Urine sugar
0 98 Nil
30 minutes 141 Nil
60 minutes 192 ++
90 minutes 172 +
120 minutes 112 Nil

Diagnosis of diabetes in non-pregnant adults2


(See Other Chapter for Gestational Diabetes)

Diabetes is diagnosed by raised blood sugar (one of the following) and


must be confirmed on a subsequent day unless unequivocal symptoms of
hyperglycemia are present. The classic symptoms of diabetes include polyurea,
polydypsia, unexplained weight loss, balanoposthitis, vulvovaginitis and recent
change of eyesight.
1. Random plasma glucose more than 200 mg/dl. Random is defined as
any time without regard to time since last meal. This is most helpful for
epidemiological purpose, health camp, etc.
Or
2. Fasting plasma glucose (FPG) more than 125 mg/dl. Fasting is defined as
10-12 hours nil orally after dinner. Only not eating for 10 hours does not
make fasting, it should be 10-12 hours after last night’s dinner and sample
to be taken around 8 a.m.
Or
3. Two-hour plasma glucose more than 200 mg/dl during an oral glucose
tolerance test (OGTT).
4. Glycosylated hemoglobin is not recommended for the diagnosis of
diabetes.

12
Management of Type 2 Diabetes

Oral glucose tolerance test


Rationale
Determines how the body handles a supraphysiological doses of
glucose challenge. It is considered the most robust way of diagnosing
diabetes.

Protocol for Gtt


 Done on outpatient basis and atleast 6 weeks after any significant
medical illness.
 Normal diet for 3 days.
 Overnight fasting, meaning nothing to eat after dinner.
 Blood sample for fasting sugar, urine sample not needed.
 75 g* of glucose mixed in 300 ml of water to drink over
5 minutes.
 Blood sample 2 hours later for sugar, urine sample not needed.
 Breakfast and end of test.
*1.75 g/kg in children, upto a maximum of 75 g.

Indications
 To diagnose diabetes in individuals with non-conclusive FPG.
 To diagnose impaired glucose tolerance and impaired fasting glucose.
 To ascertain the glycemic status of an individual who had stress
hyperglycemia (gestational diabetes, sepsis, steroid induced after
withdrawal).
 For diagnosis of gestational diabetes mellitus (criteria differs, please refer
chapter on gestational diabetes).

Interpretation
Along with the host factors like preceding diet, inactivity, illness and
medications, which may interfere with the values, technical variability should
always be considered. A 2-hour value of more than 200 mg/dl is diagnostic
of diabetes and that in between 140 and 200 mg/dl categorizes as impaired
glucose tolerance.

Drawbacks
 Not physiological.
 Intra-individual variation is 6% for fasting and 16.7% for postglucose
value.
 Cost and time.

13
Diagnosis of Type 2 Diabetes Mellitus- Simple if We are Careful

Current criteria - How achieved?


As we compare the older criteria’s to the newer ones, we note that the
changes are predominantly based on our understanding of the cut-off level of
blood sugars above which the chronic complications (especially retinopathy)
show a significant rise. These values are ably supported by the evidence-based
medicine.
Schematic representation of glycemic parameters and the risk of
retinopathy3.

15
FPG
Retinopathy (%)

2-hour plasma glucose


HbA1C
10

0
FPG mg/dl 42- 87- 90- 93- 96- 98- 101- 104- 109- 120-
(mmol/l) (2.35) 4.80) (5.00) (5.15) (5.30) (5.45) (5.60) (5.75) (6.05) (6.65)
2-hour PG, mg/dl 34- 75- 86- 94- 102- 112- 120- 133- 154- 195-
(mmol/l) (1.90) (4.15) (4.75) (5.20) (5.65) (6.20) (6.65) (7.40) (8.55) (10.80)
HbA1C 3.3- 4.9- 5.1- 5.2- 5.4- 5.5- 5.6- 5.7- 5.9- 6.2-
(fraction) (0.033) (0.049) (0.051) (0.052) (0.054) (0.055) (0.056) (0.057) (0.059) (0.062)

Fasting glucose values considered diagnostic of diabetes has been lowered to


more equivalent to those found where 2-hour plasma value after an oral glucose
load is 200 mg/dl or greater4. This increases the likelihood of diagnosing diabetes
when only fasting values are determined. It is also noted that the individual
co-efficient of variation has been estimated to be 6.4 for fasting compared to
16.7 for postglucose values5, making fasting a more reliable and convenient test
if used as a single test to diagnose diabetes.

14
Management of Type 2 Diabetes

Indian context - What fasting is safe?


It is said that Life does not live by guidelines, so we have to adopt as
to our situation. With the present burden of diabetes and prediabetes in
our set up; for a better and early identification of individuals we have noted
that a fasting value of more than 90 mg/dl should call for an OGTT, rather than
a value of 110 mg/dl6. In this study it was clearly shown that a fasting glucose
above 90 mg/dl increases the 2 hours value above 140 mg/dl.

Case discussion
Case 1
She has diabetes. Her random blood sugar is more than 200 mg/dl and she
has symptoms. She needs good glycemic control in the perioperative period for
smooth recovery.

Case 2
This girl has unwanted pregnancy. She does not have diabetes. Her ketone
in urine was related to starvation and sugar in urine was due to renal glycosuria
seen in pregnancy.

Case 3
She does not have diabetes. This case is an example of alimentary
glycosuria. She has hyperthyroidism, which is well known cause of alimentary
glycosuria.
A finding of glycosuria indicates that the person is either hyperglycemic,
has alimentary glycosuria or has a lowered renal threshold for glucose.
Occasionally, glycosuria may be a normal finding, such as after eating a heavy
meal or during times of emotional stress. In pregnancy, the renal threshold
for glucose may be lowered so that small amounts of glycosuria may be
present. However, urine glucose is not a reliable method as renal threshold
varies between patients and also within patients and is also affected by the
patient’s hydration status.
The renal threshold for glucose is approximately 160-190 mg/dl of blood.
Either this patient has an alimentary glycosuria or a reduced renal threshold
for glucose (pseudo renal glycosuria). Alimentary glycosuria develops during
the postprandial period and after glucose load while 2 hours postprandial
value does not exceed 140 mg/dl, because of 30/60 minutes value above the
180 mg/dl.

15
Diagnosis of Type 2 Diabetes Mellitus- Simple if We are Careful

Screening - How often to recommend?


Aim: To identify asymptomatic individual with diabetes.

Where?
The effectiveness of screening depends on the setting in which it is
performed. Community screenings are poorly targeted and may fail to reach
the groups most at risk and inappropriately test those at low-risk (the worried
well) or already diagnosed cases and miss the high-risk group7. It also may
yield abnormal tests that are never discussed with a primary care provider,
low compliance with recommendations and a very uncertain impact on long-
term health. So it is prudent to screen in a healthcare setting, rather than in
the community.

When?
Most western literature recommends the age of screening as 45 years and
above. In our set up we recommend to start screening at 25 years and if normal
to repeat it annually. This is irrespective of family history as all of us are at
an increased risk.

Testing for type 2 diabetes in children


and adolescents
We recommend test for diagnosing diabetes in children and adolescents
when they are.
 Overweight with body mass index more than 85th percentile for age and
sex, weight for height more than 85th percentile, or weight more than 120%
of ideal for height
 Signs of insulin resistance or conditions associated with insulin resistance
(acanthosis nigricans, hypertension, dyslipidemia or polycystic ovarian
syndrome).

Learning Points
1. Patients of diabetes can be asymptomatic, so early detection and appropriate
management improves outcome.
2. Glycosylated hemoglobin and urine sugars are not recommended for diagnosis of
diabetes.
3. OGTT is the most robust way of diagnosing diabetes.
4. A fasting value of 90 mg/dl should be the indication for OGTT in our set up.

16
Management of Type 2 Diabetes

References
1. Presentations of patients with type 2 diabetes. From UKPDS. IV. Characteristics of
newly presenting type 2 diabetic patients: Male preponderance and obesity at different
ages. Diabet. Med. 1988;5:154-159.
2. Report of the expert committee on the diagnosis and classification of diabetes mellitus.
Diabetes Care 2007 Jan.;30(Suppl. 1).
3. National Diabetes Data Group. Diabetes in America 2d Edition, National Institutes of
Health, National Institute of Diabetes and Digestive and Kidney Diseases, 1995; NIH
publication no. 95-1468 Bethesda, Md.
4. Gavin JR III, Alberti KGMM, Davidson MB, et al. Report on the expert committee
on the Diagnosis and Classification of Diabetes Mellitus. Diabetes Care 1997;20:
1183-1197.
5. Report of the expert committee on the diagnosis and classification of diabetes mellitus.
Diabetes Care 1997;20:1183-1187.
6. Bhattacharya A, Menaka R, Sabeer TK and Latha M. At what fasting glucose should
we recommend OGTT? Diabetic Medicine 2006;23(4):P1074.
7. USPSTF: Screening for type 2 diabetes mellitus in adults: Recommendations and
rationale. Ann. Intern. Med. 2003;138:212-214.

17
Type 2 Diabetes in India - A Ramachandran,

An Increasing Threat C Snehalatha


A Ramachandran’s

The rise in prevalence of diabetes is a global phenomenon and the recent


prevalence estimate of the International Diabetes Federation (IDF) confirms
beyond all doubt that diabetes epidemic is real. It is becoming the major
cause of morbidity and mortality in the populations. In the year 2007, 246
million people (6.0%) in the age group of 20-79 will have diabetes world wide,
of which 80% contribution will be from the developing countries1. Another
notable feature is that the 40-59 year group has the highest number of people
with diabetes (113 million). The global prevalence is expected to increase to
380 million (7.3%) by the year 2025, of which 69.9 million will be from India
(40.9 million in 2007) (Table 1).

The diabetes epidemic in India


In India, the first national study on the prevalence of diabetes was
conducted by the Indian Council of Medical Research between 1972 and 1975
and was reported to be 2.1% in urban population and 1.5% in rural population2.
The World Health Organization (WHO) criteria were not available then. In
2000, the National Urban Diabetes Survey was conducted in six metropolitan
cities in India and the prevalence was found to be 13.9%3. The PODIS study
in 2003, showed a prevalence of 5.6% in urban and 2.7% in rural areas4.
Different sampling method had been adopted in this study. Regional studies
conducted, in rural and urban areas have shown an increasing trend in the
prevalence of diabetes3,5-13. In Chennai, India, a metropolitan city with a mixed
population of varied ethnic and socio economic characteristics, where data
on the prevalence of diabetes are available from 1984 onwards, an escalating
trend could be noted. The prevalence was 5.2% in 1984; 8.2% in 1989; 11.6%
in 1995 and 13.5% in 2000 (Fig. 1). In 2003, a further increase to 14.3% was
reported14. There is a 74.4% increase in the prevalence of diabetes in Chennai
between 1989 and 2003.
Studies from other parts of India also showed an increasing trend in the

18
Management of Type 2 Diabetes

Table 1 World Population and India at a Glance

World Population India


2007 2025 2007 2025
Total (billions) 6.6 7.9 (millions)
Adult population (age 20 – 79) (billions) 4.1 5.2 (millions) 659.5 918.7
World Diabetes and IGT (20-79 age group)
Diabetes

Regional Prevalence (%) 6.2 7.6


Comparative prevalence (%) 6.0 7.3 6.7 8.2
Number of people with DM(millions) 246 380 40.9 69.9
IGT
Regional Prevalence (%) 5.4 6.1
Comparative prevalence (%) 7.5 8.0 5.6 6.3
Number of people with IGT (millions) 308 418 35.9 56.2
Diabetes Atlas, 3rd Edition, 2006

prevalence of diabetes. A very high prevalence of 16.8% was reported in a


survey done in Thiruvanathapuram in Kerala state in the year 1999.10 Diabetes
prevalence was reported to be 8.2% in Guwahati15. Prevalence studies had
been carried out in the western parts of India, where in the urban areas the
age-standardised prevalence of 8.6% was reported16. From rural Maharashtra
a prevalence of 9.3% was reported17.

The impending threats


The major threat to a further rise in the prevalence of diabetes in the Indian
population appears to be the existing prediabetic pool, early onset of diabetes,
obesity and urbanization which leads to lifestyle changes (esp. unhealthy diet
practices and sedentary lifestyle).

Impaired glucose tolerance and impaired fasting glucose


The diabetes situation is exaggerated by a high prevalence of impaired
glucose tolerance (IGT) which has a high potential for conversion to diabetes
and also carries a high risk for cardiovascular diseases (CVD). The prevalence
of IGT is more than twice that of diabetes in the African and South East
Asian regions1,18.
A high prevalence of IGT has been reported in several recent epidemiological

19
Type 2 Diabetes in India - An Increasing Threat

studies in developing nations such as India3, Pakistan19, Bangladesh20, South


Korea21 and Africa1. Even when a marked urban-rural difference was seen in
the prevalence of diabetes, the prevalence of IGT was similar in many Asian
populations1,19,20. This probably indicates a genetic basis for type 2 diabetes as
IGT is a prediabetic condition in most of the ethnic groups.
Prevalence of impaired fasting glucose (IFG) is also high in Indians and
in many other Asian populations22,23. IGT and IFG show several metabolic
aberrations which are risk factors for diabetes and cardiovascular diseases. They
include insulin resistance, presence of hypertension and obesity. Clustering of
the risk factors was also common in these conditions24.
It is worth stressing here that the results of the National Urban Diabetes
Survey (NUDS) study showed that IGT occurred at a very young age in the
Indians and its prevalence was significantly more than that of diabetes in the
young population3. The need for an early screening for glucose intolerance in
the population and also the need for institution of preventive measures at an
early age are highlighted by these observations.

Aging
Continued increase in life expectancy and a decline in fertility are expected

20
Management of Type 2 Diabetes

to result in faster aging of the global population in the next 50 years. Diseases
of the elderly, such as diabetes, hypertension, cardiovascular diseases and
cancer will become commoner due to the aging of the population. Prevalence
of diabetes is known to increase linearly with increasing age. In the developed
western countries diabetes generally occurs in individuals aged ≥65 years.
In the developing countries onset of diabetes occurs at a younger age (45-
65 years)1,25. Studies from India have shown a much younger age at onset of
diabetes compared to the western population26. The DECODA study has made
a comparative analysis of age at diagnosis of diabetes in different races26. The
association between age and diabetes was higher in the Indian and the Maltese
population compared to all other populations studied (Europeans, Chinese
and Japanese). At each age group the fasting and 2 hours plasma glucose
were significantly higher for Indians than Chinese and Japanese populations.
The age and sex specific prevalence and the peak prevalence of diabetes
were higher in the Indian and in Singapore cohorts than in the Chinese and
Japanese cohorts. Reports form Pakistan also showed an age-related diabetes
prevalence data similar to that in India. The peak age was in the age group
of 55-64 years78.
IGT was more prevalent than IFG in almost all age groups in Asian subjects.
The prevalence of IGT increased with age whereas IFG did not. This finding
was consistent with the finding in the European populations. If only fasting
glucose determination were used for the diagnosis of diabetes there would
be a significant under estimation in the Asians as a post load hyperglycaemia
was more common in them26.
An early occurrence of diabetes in the population has severe economic
impact as severe morbidity and early mortality occurs in the most productive
years of life. The diabetic subjects live long enough to develop the debilitating
vascular complications of diabetes.

Obesity
An excess of body fat especially concentrated within the abdomen has a
range of potential harmful effects. It includes increased risk for diabetes, blood
pressure, dyslipidaemia, insulin resistance, coronary artery disease and some
forms of cancer. For diabetes, obesity and specifically abdominal obesity is a
major risk factor. In the white population, overweight is defined as the body
mass index (BMI), of ≥25kg/m2 and obesity as a BMI of >30kg/m2. However,
several studies have now shown that an average BMI level in Asians ranges

21
Type 2 Diabetes in India - An Increasing Threat

Figure 2.

between 20-23 kg/m2 and the risk of metabolic disease increases progressively
above 22 kg/m2. The WHO Expert group has considered a BMI of ≤23 kg/m2
has the cut off for normal in Asian population28. A study from the Diabetes
Research Centre had shown that the risk of diabetes in Indian population
increases significantly above a BMI of 23 kg/m2 indicating that a healthy BMI
for Indians was definitely lower than this limit (Fig. 2)27.
Similarly the waist circumference which is an index of upper body
adiposity is also lower in the Asian population. WHO and the ATP III criteria
recommend a cutoff limit for waist circumference of 102 cm and 88 cm in men
and women respectively29. However, the recent studies have shown that the
cut off for normal waist circumference are significantly lower for the Asian
population (Fig. 3). The cut off values for normal waist circumference are
85cm and 80 cms for men and women respectively in Indian population28.
China has adopted its own standards for defining overweight in the BMI of
24 or more and obesity at BMI of 28 or more. Abdominal obesity is defined
by waist circumference of 85 cms in men and 80 cms in women1.
It has also been observed that Asian Indians have a higher percentage
of body fat, for a given BMI, when compared with the white population30-33.
This could be one of the contributory factors for the higher insulin resistance
found in Asian Indian populations.

22
Management of Type 2 Diabetes

Urbanization - The narrowing of urban-rural divide


The developing countries are undergoing rapid urbanization and migration
of population to urban areas. In the next 30 years a major redistribution of
the population will occur and by the year 2030, 60% of the world’s population
would be living in urban areas18. According to the WHO estimates in the last
5 decades 2- to 5-fold increase in urban population has occurred in most of
the SEA countries. In India, the percentage of urban population was 25.5% in
1990, 28.4%, in 2000 and it is expected to increase to 45.8% in 2030.18
Impact of urbanization on the prevalence of diabetes was evident from
the two studies conducted in Chennai, India. In the first study, the prevalence
of diabetes in a semi-urban area was found to be 5.9% in comparison with
the prevalence of 2.4% in the rural and 11.6% in the urban population9. The
social and economic changes occurring in rural India have produced significant
changes in the occupational, dietary and activity levels. As a result, increasing
number have sought jobs in urban areas involving less of manual labour.
Better transport facilities were available and physical activity levels were
significantly reduced. Increased calorie consumption with more of fat and refined
carbohydrates had also become common. Impacts of the lifestyle transition
were evident in a more recent rural study in which the prevalence of diabetes
had increased to 6.3% in the adult population13. As shown in the Figure 4,

Figure 3.

23
Type 2 Diabetes in India - An Increasing Threat

Figure 4.

a 3-fold increase in diabetes had occurred in a span of 15 years. However,


the prevalence of IGT had remained similar. A study from rural Bangladesh
showed the prevalence of type 2 diabetes was low 3.8%, but prevalence of
IFG was 13.0%22.
High prevalences of IGT and IFG were reported recently by the DECODA
study group26.

Type 2 diabetes in children


Recent studies in migrants in UK34 and USA35, studies in India36 and
Singapore37 show a rising trend in type 2 diabetes in children and adolescents.
Overweight in childhood is a forerunner of overweight in adulthood. The
association of obesity with metabolic diseases such as diabetes and cardiovascular
diseases is well known. Reports of increasing overweight among children in
urban India indicate that the epidemic of diabetes could become worse with
the increasing epidemic of obesity now seen even among children. In a study
in urban southern India, the prevalence of overweight was 17.8% among boys
and 15.8% among girls aged 14-19 years38. There was a strong association of
overweight with lack of physical activity and higher socioeconomic group.
A recent study by us showed that 68% of normal weight children had one
abnormality or other39.
With threat of rise in the prevalence of diabetes looming large, the need

24
Management of Type 2 Diabetes

of the hour is the identification high risk subjects and institution of measures
of lifestyle modification in preventing the early onset of diabetes.

Preventive measures
Diabetes risk score
Screening for diabetes is impractical in India, which has a very large
population. Screening of the high-risk group has been advocated as the
feasible strategy to identify early diabetes. If the high risk population can be
identified using a risk score from simple questionnaire and anthropometric
measurements and no laboratory investigations, it would be practical and also
cost effective. Indians have several peculiarities in the risk factors for diabetes
and therefore the risk score that have been developed in the white population
would not apply to Indians. We have developed a simple risk score using the
risk variables including age, BMI, waist circumference, physical activity levels
and presence of family history of diabetes. This was specific for the Indian
population and this simple score can be used in any clinical setting without
any special investigations (Table 2). It is found to be highly specific for the
population and is sensitive in picking up the high-risk group40.

Prevention of diabetes
The etiological factors for diabetes include genetic and environmental

Table 2 Diabetes risk score*

Variables Risk score


Age (30 – 44) yrs 10
Age (45 – 59) yrs 18
Age (>59) yrs 19
FH-DM 7
BMI (> 25) kg/m2 7
Waist (M = >85 ,W = >80 cm) 5
Sedentary physical activity 4
Maximum Score 42
An ROC procedure showed a cutoff score of > 21 having sensitivity and specificity close
to 60 %.
* Ref 40

25
Type 2 Diabetes in India - An Increasing Threat

influences both of which are equally strong. Obesity, diet and physical activity
are the modifiable factors. A few prospective studies in white populations and
also in Chinese had shown the beneficial effects of lifestyle modification in
reducing the risk of diabetes41-43. Subjects with IGT or with history of gestational
diabetes have been studied in the prevention programmes.
Primary prevention of diabetes is urgently needed in India as it is facing an
enormous burden from the large number of subjects with diabetes. The Indian
Diabetes Prevention Programme was the first preventive study conducted in
Asia clearly demonstrated the advantage of moderate lifestyle modification in
the prevention of diabetes44.

Summary
A major share of the global burden of diabetes is on the developing
countries in the South East Asian region. The prevalence is on the increase
in Arab nations and in Mauritians also. It is estimated that by 2025 there will
be approximately 380 million diabetic subjects in the world. In developing
countries diabetic subjects are of younger age and live long enough to develop
the chronic complications. India has the largest number of diabetic population.
It is approximately 10.9 million in 2007 and may reach 69.9 million in 2025.
The major threat to a further rise in the prevalence of diabetes in the Indian
population appears to be the existing prediabetic pool, early onset of diabetes,
obesity and urbanization which leads to lifestyle changes (esp. unhealthy diet
practices and sedentary lifestyle).
Prevalence of type 2 diabetes in children and adolescents have been
increasing. Childhood obesity is on the increase which is a fore runner of
obesity related diseases in the adulthood. Rapid urbanization has resulted in
a reduction in the urban rural difference in the prevalence of diabetes. This
is likely to cause a manifold increase in the number of subjects affected with
the disease. Prediabetic conditions such as impaired glucose tolerance and
impaired fasting glucose are prevalent in the SEA countries which indicate the
possibility of a further increase in the number of diabetic subjects. Diabetes
and prediabetic conditions are also associated with many cardiovascular risk
factors which tend to cluster as the metabolic syndrome.
Diabetes care causes a major socio economic burden. Primary prevention of
diabetes, early diagnosis of diabetes and secondary prevention of complications
should be given priority in health care planning.

26
Management of Type 2 Diabetes

References
1. Sicree R, Shaw J and Zimmet P. Diabetes and impaired glucose tolerance. In: Diabetes
Atlas. International Diabetes Federation 3rd Edition, Gan D (Ed.), International Diabetes
Federation, Belgium 2006:15-103.
2. Ahuja MMS. Epidemiological studies on diabetes mellitus in India. In: Epidemiology of
Diabetes in Developing Countries Ahuja MMS (Ed.) Interprint, New Delhi 1979:29-38.
3. Ramachandran A, Snehalatha C, Kapur A, et al. For the Diabetes Epidemiology Study
Group in India (DESI). High Prevalence of diabetes and impaired glucose tolerance
in India: National Urban Diabetes Survey. Diabetologia 2001;44:1094-1101.
4. Sadikot SM, Nigam A, Das S, et al. Diabetes India. The burden of diabetes and
impaired glucose tolerance in India using the WHO 1999 criteria: Prevalence of
diabetes in India study (PODIS). Diabetes Research Clinical Practice 2004;66:301-307.
5. Gupta OP, Joshi MH and Dave SK. Prevalence of diabetes in India. Advances in
Metabolic Disorders 1978;9:147-165.
6. Ramachandran A, Jali MV, Mohan V, et al. High prevalence of diabetes in an urban
population in South India. Br. Med. J. 1988;297:587-590.
7. Ramachandran A, Snehalatha C, Daisy Dharmaraj, et al. Prevalence of glucose
intolerance in Asian Indians: Urban rural difference and significance of upper body
adiposity. Diabetes Care 1992;15:1348-1355.
8. Ramachandran A, Snehalatha C, Latha E, Vijay V and Viswanathan M. Rising
prevalence of NIDDM in urban population in India. Diabetologia 1997;40:232-237.
9. Ramachandran A, Snehalatha C, Latha E, Manoharan M and Vijay V. Impacts of
urbanisation on the lifestyle and on the prevalence of diabetes in native Asian Indian
population. Diabetes Research and Clinical Practice 1999;44:207-213.
10. Kutty R, Soman CR, Joseph A, et al. Type 2 diabetes in southern Kerala. Variation in
prevalence among geographic divisions within a region. The National Medical Journal
of India 2000;13:287-292.
11. Misra A, Pandey RM, Rama Devi J, et al. High prevalence of diabetes, obesity and
dyslipidaemia in urban slum population in northern India. International Journal of
Obesity 2001;25:1-8.
12. Ramachandran A, Snehalatha C and Vijay V. Temporal Changes in Prevalence of
Type 2 Diabetes and Impaired Glucose Tolerance in Urban Southern. India Diabetes
Research Clinical Practice 2002;58:55-60.
13. Ramachandran A, Snehalatha C, Baskar ADS, et al. Temporal changes in prevalence of
diabetes and impaired glucose tolerance associated with lifestyle transition occurring
in rural population in India. Diabetologia 2004;47:860-865.
14. Deepa M, Pradeepa R, Rema M, et al. The Chennai Urban Rural Epidemiology Study
(CURES)- study design and methodology (urban component) (CURES-I). Journal of
Associations of Physicians of India 2003;51:863-870.
15. Shah SK, Saikia M, Burman NN, et al. High prevalence of type 2 diabetes in urban
population in north eastern India. International Journal of Diabetes in Developing
Countries 1999;19:144-147.
16. Gupta A, Gupta R, Sarna M, et al. Prevalence of diabetes, impaired fasting glucose
and insulin resistance syndrome in an urban Indian population. Diabetes Research
Clinical Practice 2003;61:69-76.
17. Deo SSS, Zantye A, Mokal R, et al. To identify the risk factors for high prevalence
of diabetes and impaired glucose tolerance in Indian rural population. International
Journal of Diabetes in Developing Countries 2006;26:19-23.
18. Health Situation in the South East Asia Region 1998-2000. World Health Organization,
Regional Office for South East Asia, New Delhi, India 2002.
19. Shera AS, Rafique G, Khawaja IA, et al. Pakistan National Diabetes Survey. Prevalence
of glucose intolerance and associated factors in Baluchistan Province. Diabetes
Research Clinical Practice 1999;44:49-58.

27
Type 2 Diabetes in India - An Increasing Threat

20. Abu Sayeed M, Zafirul Hussain M, et al. Prevalence of diabetes in a suburban


population of Bangladesh. Diabetes Research Clinical Practice 1997;34:149-155.
21. Park Y, Lee H, Koh Chang S, et al. Prevalence of diabetes and IGT in Yonchon
Country, South Korea. Diabetes Care 1995;18:545-548.
22. Abu Sayeed M, Hajera Mahtab, Parvin Akter Khanam, et al. Diabetes and impaired
fasting glycemia in a rural population of Bangladesh. Diabetes Care 2003;26:
1034-1039.
23. Ramachandran A, Snehalatha C, Satyavani K and Vijay V. Impaired fasting glucose
and impaired glucose tolerance in urban population in India. Diabetic Medicine 2003;
20:220-224.
24. Snehalatha C, Ramachandran A, Satyavani K, et al. Clustering of cardiovascular risk
factors in impaired fasting glucose and impaired glucose tolerance. International Journal
of Diabetes in Developing Countries 2003;23:58-60.
25. Ramachandran A, Snehalatha C and Vijay V. Low risk threshold for acquired
diabetogenic factors in Asian Indians. Diabetes Research Clinical Practice 2004;65:
189-195.
26. Qiao Q, Hu G, Tuomilehto J, et al. Age and sex-specific prevalence of diabetes and
impaired glucose regulation in 11 Asian cohorts. Diabetes Care 2003;26:1770-1780.
27. Snehalatha C, Vijay V and Ramachandran A. Cut off values for normal anthropometric
variables in Asian Indian adults. Diabetes Care 2003;26:1380-1384.
28. Obesity: Preventing and managing the global epidemic: Report of a WHO consultation.
WHO Tech Rep Ser 2000;894:1-253.
29. Expert Panel on Detection, Evaluation and Treatment of high blood cholesterol in
adults. Executive summary of the third report of the National Cholesterol Education
Program (NCEP) of high blood cholesterol in adults (adult treatment panel 111).
Journal of American Medical Association 2001;285:2486-2497.
30. Snehalatha C, Ramachandran A, Satyavani K, et al. Difference in body fat percentage
does not explain the gender dimorphism in leptin in Asian Indians. Journal of Association
of Physicians of India 1999;47:1164-1167.
31. Banerji MA, Faridi N, Atluri R, et al. Body composition, visceral fat, leptin and insulin
resistance in Asian Indian men. Journal of Clinical Endocrinology & Metabolism 1999;
84:137-144.
32. Dudeja V, Misra A, Pandey RM, et al. BMI does not accurately predict overweight
in Asian Indians in Northern. India British Journal of Nutrition 2001;86:105-112.
33. Gallagher D, Visser M, Sepulveda D, et al. How useful is body mass index for
comparison of body fatness across age, sex and ethnic groups? American Journal of
Epidemiology 1996;143:228-239.
34. Ehtisham S, Barrett TG and Shaw NJ. Type 2 diabetes mellitus in UK children – an
emerging problem. Diabetic Medicine 2000;17:867-871.
35. Sinha R, Fisch G, Teague B, et al. Prevalence of impaired glucose tolerance among
children and adolescents with marked obesity. New England Journal of Medicine 2002;
346:802-810.
36. Ramachandran A, Snehalatha C, Satyavani K, et al. Type 2 diabetes in Asian – Indian
urban children. Diabetes Care 2003;26:1022-1025.
37. Warren Lee WR. The changing demography of diabetes mellitus in Singapore. Diabetes
Research Clinical Practice 2000;50:S35-S39.
38. Ramachandran A, Vinitha R, Megha Thayyil, et al. Prevalence of overweight in
urban Indian adolescent school children. Diabetes Research Clinical Practice 2002;57:
185-190.
39. Ramachandran A, Snehalatha C, Yamuna A, et al. Insulin resistance and clustering
of cardiometabolic risk factors in urban teenagers in sourthern India. Diabetes Care
2007;30:1828-1833.
40. Ramachandran A, Snehalatha C, Vijay V, et al. Derivation and validation of diabetes

28
Management of Type 2 Diabetes

risk score for urban Asian Indians. Diabetes Research Clinical Practice 2005;70:63-70.
41. Diabetes Prevention program Research group. Reduction in the Incidence of Type 2
diabetes with lifestyle intervention or metformin. N. Engl. J. Med. 2002;346:393-403.
42. Tuomilehto J, Lindstrom J, Eriksson JG, Valle TT, Hamalainen H, Ilannie-Parikka
P, Keinanen-Kiukaanniemi S, Laakso M, Louheranta A, Rastas M, Salminen V and
Uusitupa M. Prevention of type 2 diabetes mellitus by changes in lifestyle among
subjects with impaired glucose tolerance. N. Engl. J. Med. 2001;344:1343-1350.
43. Chiasson JL, Josse RG, Gomis R, Hanefeld M, Karasik A and Laakso M. Acarbose for
prevention of type 2 diabetes mellitus; the STOP – NIDDM randomized trial. Lancet
2002;359:2072-2777.
44. Ramachandran C, Snehalatha S, Mary B, Mukesh AD and Bhaskar V Vijay. Indian
Diabetes Prevention Programme (IDPP). The Indian Diabetes Prevention Programme
shows that lifestyle modification and metformin prevent type 2 diabetes in Asian
Indian subjects with impaired glucose tolerance (IDPP 1). Diabetologia 2006;49:
289-297.

29
Type 2 Diabetes in the Young –
An Emerging Epidemic! Geetha K Bhat
Prasanna Kumar

Introduction
One of the alarming consequences of the diabetes epidemic is the
appearance of type 2 diabetes in children, adolescents and young adults.
Once considered as a disease of the middle aged and elderly, onset of
diabetes is now increasingly seen in the 20-30 years age group and also in
children. This new epidemic heralds an emerging public health problem of
major proportions. The rising prevalence of obesity and type 2 diabetes in
children and adolescents is yet another symptom of the effects of sedentary
lifestyles as part of globalization and industrialization affecting all societies1.
Onset of diabetes at a younger age heralds many years of disease and an
increased risk that the full range of both macrovascular and microvascular
complications.

Epidemiology
The available information globally on type 2 diabetes incidence and
prevalence in childhood and adolescence is sparse compared with that of adults.
The largest study reported is from Japan1. Type 2 diabetes incidence increased
10-fold in children aged 6-12 years. Incidence doubled among 13-15 years olds
from 7.3 to 13.9 per 100,000 per year with type 2 diabetes now outnumbering
type 1 diabetes in that country1. Among children in Japan, type 2 diabetes is
already more common than type 1 diabetes and accounts for 80% of childhood
diabetes. Prevalence rates increased 6-fold for Pima Indian adolescents and
10-fold for adolescents in Cincinnati.
Clinic based register studies in the Unites States reveal increasing annual
incidence of type 2 diabetes in 10-19 years olds by 7-9% per year. The limited
data available from our country suggests a similar trend.
Ramachandran, et al., from Chennai studied total of 18 children (five boys
and 13 girls) aged 9–15 years with insidious onset of diabetes responding to
oral antidiabetic agents (ODAs) for periods from 2 months to 12 years. Clinical
details, anthropometry, and details of family history of diabetes were elicited. All
of them were tested for the presence of anti-GAD65 antibodies and for pancreatic

30
Management of Type 2 Diabetes

b-cell reserve by measuring serum C-peptide response (radioimmunoassay


procedures). All children showed a response to ODAs, had good ß-cell reserve
(≥0.6 pmol/ml on stimulation), and negligible GAD65 antibodies indicating the
presence of type 2 diabetes. The children were non-ketotic; nine were obese,
four had acanthosis nigricans, and one had polycystic ovary syndrome (PCOS).
Positive family history of diabetes was present in all cases3.

Classification and diagnosis


Clinical differentiation of etiology of diabetes in children and adolescents
can be ambiguous and necessitates laboratory tests. These tests are not routinely
available in the primary care environment and in India. Table 1 summarizes
the clinical features which can aid us in the differentiation. The following case
is a typical example which we encounter in clinical practice-
A 15-year-old adolescent girl was evaluated in the department. She had been
diagnosed with type 1 diabetes and was on biphasic insulin 12 units twice a
day. Examination revealed a body mass index of 27 kg/m2, acanthosis nigricans
and family history of type 2 diabetes. There was no history of ketosis. All the

Table 1. Features to differentiate type 1 and 2 diabetes in the young


Type 1 diabetes Type 2 diabetes
Onset Acute-symptomatic Slow-often asymptomatic
Weight loss Obese
Polyuria Strong family history of type 2 diabetes
mellitus
Polydypsia Ethinicity - high prevalence populations
Acanthosis nigricans
PCOS
Race Less in Asian, native Increased in African-American, Hispanics,
Americans Asians
Ketosis Almost always present Usually absent
Insulin sensitivity Normal Gross reduction
Insulin antibodies C-peptide negative C-peptide positive
ICA positive ICA negative
Anti-GAD positive Anti-GAD negative
ICA 512 positive ICA 512 negative
Therapy Insulin invariably Oral antihyperglycemic agents usually
Associated Yes No
autoimmune
diseases

31
Type 2 Diabetes in the Young – An Emerging Epidemic!

above data suggested that this teenager probably had type 2 diabetes. Further
testing revealed a positive C-peptide stimulation test. She was weaned off insulin
and currently has good glycemic control with metformin.
At the root of the classification problem is the incomplete understanding of
the pathogenesis of the two major types of diabetes. Classification is complicated
because autoimmunity (strongly implicated in type 1 diabetes) does not contribute
substantially in non-europoid populations. Further differential diagnoses are
maturity onset diabetes of young (MODY) which are attributable to specific
mutations of certain genes.(18) In ideal circumstances it is recommended that
apparently typical cases. Therefore we have to rely more on clinical presentation,
family history and features and a trial of therapy. The levels of plasma glucose
used to define impaired fasting glucose, impaired glucose tolerance (IGT) and
type 2 diabetes remain the same in the young and adults.

Pathogenesis
Key factors
Genetic factors: The genetic component of type 2 diabetes in children and
adolescents has yet to be explored. A strong family history is noted as in adults.
Only a small percentage of cases of diabetes in young can be attributed to single
gene defects as in MODY.
Obesity: Obesity appears to be the key link which is exemplified by the
rising incidence of type 2 diabetes and levels of obesity. In India, study by
Ramachandran, et al. found that the age adjusted prevalence of being overweight
among 13-18 years olds was 18% in boys and 16% in girls6. Prevalence rates
increased with age, decreasing physical activity and higher socioeconomic
status6. Excessive intake of easily available high energy foods, sedentary lifestyle
inadequate activity are all responsible for the increasing incidence of
obesity.
Physical inactivity: Physical inactivity is one of the major contributors
to overweight and obesity. Studies have documented decrease in leisure time
physical activity and exercise. Coupled with increase in television viewing
time, computer and video games, internet surfing daily exercise seems to have
become obsolete in children, adolescents and young adults. Work atmosphere
where one is seated for 10-14 hours with a lot of mental stress adds fuel to
fire in young adults.
Insulin resistance: In a variety of ethnic groups, obesity is associated with
evidence of insulin resistance and impaired insulin secretion among children

32
Management of Type 2 Diabetes

Table 2. ADA recommendations for testing for type 2 diabetes in children


Criteria for screening:

Overweight (BMI 85th percentile for age and sex, weight for height 85th percentile or
weight 120% of ideal for height)

Plus any two of the following risk factors:

l Family history of type 2 diabetes in first or second degree relative

l Race/ethnicity (American Indian, Hispanic, Asian/pacific islander)


l Signs of insulin resistance or conditions associated with insulin resistance
(acanthosis nigricans, hypertension, dyslipidemia, PCOS)

Age of initiation: Age 10 years or at onset of puberty if puberty occurs at a


younger age

Frequency: Every 2 years

Test: Fasting plasma glucose preferred

and adolescents like adults. The importance of insulin resistance can be drawn
from the observation that the onset of type 2 diabetes frequently occurs around
the time of puberty when insulin sensitivity declines. Acanthosis nigricans is
a well established physical marker of insulin resistance and is reported to
occur in 60-90% of young people with type 2 diabetes. PCOS is associated
with a state of insulin resistance and compensatory hyperinsulinemia and a
50% reduction in peripheral tissue insulin sensitivity and evidence of hepatic
insulin resistance. Approximately 30% of adolescent girls with PCOS have been
found to have IGT and 4% with type 2 diabetes.
Intrauterine environment: Birth weight and maternal hyperglycemia may
possibly affect the development of type 2 diabetes in the young. There is now
abundant evidence that low birth weight predicts type 2 diabetes in the middle
age. Recent evidence from India seems to indicate that the greatest risk of
obesity and glucose intolerance in adulthood is in those with low birth weight
who gain weight very rapidly in childhood7.
Gestational diabetes mellitus (GDM) also seems to increase the risk of
diabetes developing in the offspring. Krishnaveni, et al. from Mysore, India
showed that maternal GDM is associated with adiposity and higher glucose
and insulin concentrations in female children at 5 years10.
Other factors like socioeconomic factors also seem to play a role in increasing
obesity and type 2 diabetes.

33
Type 2 Diabetes in the Young – An Emerging Epidemic!

Screening
Treatment: The management of diabetes in the young raises new issues
about oral pharmacologic therapies for glycemic control and for control of blood
pressure and dyslipidemia. Goals of treatment in the young are:
l Physical and psychological well being
l Long-term glycemic control
l Prevention of microvascular complications
l Prevention of macrovascular disease
l Normal growth in children and younger adolescents.
Short-term goals include management of acute decompensation if present,
treatment of associated infections. Long-term goals include normal growth,
achieving and maintaining a reasonable body weight, regular physical activity,
avoidance of smoking and prevent complications.
Glycemic control: The goal is to maintain a HbA1C of <7% in the young.
Lifestyle intervention encompassing diet, exercise and weight control is the
cornerstone of management. It also has beneficial effect on lipids and blood
pressure. Initial management depends on the presentation. Patients with acute
symptoms and weight loss will benefit from initial insulin therapy.
Experience with oral agents is scant in children and young adolescents. In
the US about 50% of young type 2 diabetics receive insulin and other half receive
oral agents. Among the oral agents majority receive metformin.
Insulin: There is neither data regarding specific types of insulins indicated
in young diabetics nor a magic formula for dosing. All the types of insulins
currently available can be used in the young, though one should use caution in
prescribing analogs to children. Short acting insulins are recommended during
acute metabolic decompensation following which biphasic insulins like 30/70
insulins can be used. Often it is feasible to wean off patients from insulins and
can be treated with oral agents.
Oral agents: Metformin has now been approved for pediatric use. It is
the first choice especially in obese young diabetics in the absence of severe
hyperglycemia. Studies in 10-16 years olds have documented its safety.
Metformin is best taken with food or immediately after food to avoid its
gastrointestinal side effects.
Sulfonylureas can be safely used in postadolescent young diabetics. All the
available sulfonylureas glibenclamide, glipizide, glimiperide, gliclazide can be
used. The dose should be titrated slowly to avoid hypoglycemia.

34
Management of Type 2 Diabetes

Thiazolidinediones are not approved for use in children and adolescents


but can be prescribed to young adults. The effects of edema and significant
weight gain are often less in young adults.
a-glucosidase inhibitors acarbose, miglitol and now voglibose are good
options to control postprandial hyperglycemia in young adults. Multiple dosing
is needed and gastrointestinal side effects like bloating and flatulence may
decrease acceptance among the young.

Lipid lowering therapy


It is recommended that lipids be measured every 2 years in children with
type 2 diabetes and yearly in young adults. Initial dietary intervention followed
by medication as needed is advised. Resins should be the initial choice in children
which is a problem for us due to non-availibilty. The next option are statins.
Young adults can be started on statins.

Hypertension
It is a comorbidity of type 2 diabetes in youth and is a major risk factor
for nephropathy and atherosclerosis. First-line therapy is with an angiotensin
converting enzyme inhibitor, with angiotension receptor blockers being the
second-line of therapy.

Aspirin
It is not recommended under the age of 21 years due to the risk of Reye’s
syndrome. Adults <30 years have not been studied.

Barriers to Treatment
The greatest obstacle we face during treatment is in the adolescent age group.
During adolescence the developmental stage is towards independence, often
with risk taking behavior, non-compliance and resistance to lifestyle changes.
Children too are not able to understand the implications of the disease. Family
involvement plays a pivotal role in the management of diabetes in young.

Prevention
Type 2 diabetes in the young is a public health issue. Implementation of
measures to prevent this emerging epidemic should be the responsibility of all of
us. Prevention should be initiated at the primary care level where opportunistic
screening can be done in obese children and adolescents. Prevention of childhood
obesity should be a primary goal. Weight control for all overweight children

35
Type 2 Diabetes in the Young – An Emerging Epidemic!

>2 years of age and young adults should be initiated by the family physician.
Lifestyle modification which constitutes primarily dietary modification which
should be age specific, promote healthy eating habits and regular physical activity.
Family meals have been shown to reduce unhealthy eating patterns. Families
should be encouraged to be physically active together rather than huddle on
the couch watching hours of television. Television, with its numerous unhealthy
food messages, computer and video games and constant internet use which all
promote obesity should be kept to the minimum.
Behavioral therapy is often needed in young obese individuals.
The American Diabetes Association (ADA) recommends a fasting plasma
glucose in children for screening once in 2 years. The oral glucose tolerance
test can be used in high risk adolescents and young adults.
The only evidence on diabetes prevention in children relates to the
importance of breastfeeding which may limit excessive energy intake and perhaps
improve insulin sensitivity. Prevention should be implemented at the school
system where young children are very receptive. Government intervention can
include mandating a greater emphasis on more exercise and dietary education
in schools, banning advertisements of unhealthy food products.

Summary
With increasing rates of obesity, age of onset of type 2 diabetes is falling
in our nation. The pathophysiology of type 2 diabetes in young appears to be
similar to that of adults. Management often poses problems in differentiation
regarding the type of diabetes at the initial presentation. Successful treatment
needs intensive efforts and family involvement.
More evidence is needed about long-term safety in relation to all the oral
agents available in children. Prevention must remain a high priority and can
be successful only if governments and communities provide the environment
within which individuals can make lifestyle changes. It would be prudent for
us to address this emerging problem as a public health issue under the heading
of primary care and prevention rather than dealing with the consequences of
diabetes and its complications in a young population.

References
1. Kitagawa T, Owada M, et al. Increased incidence of non insulin dependent diabetes
mellitus among Japanese school children correlates with a an increased intake of
animal protein and fat. Clin. Pediatr. 1998;37:111-116.
2. Dabelea D, Hanson RL, Bennett PH, Roumain J, et al. Increasing prevalence of
type 2 diabetes in American Indian Children. Diabetologia 1998;41:904-910.

36
Management of Type 2 Diabetes

3. Ramachandran A, Snehalatha C, Satyavani K, et al. Type 2 diabetes in Asian Indian


urban children. Diabetes Care 2003;26:1022-1025.
4. Pinhas-Hamiel O, Dolan L, Daniels S, et al. Increased incidence of non insulin
dependent diabetes among adolescents. J. Pediatr. 1996;128:608-615.
5. Narayan KM and Fagot–Camapgna A. Type 2 diabetes in children, a lurking problem
for India? Indian Pediatr. 2001;38:701-704.
6. Ramachandran A, Snehalatha C, Vinutha R, et al. Prevalence of overweight in urban
Indian adolescent school children. Diabetes Res. Clin. Pract. 2002;57:185-190.
7. Bavdekar A, Yajnik CS, Fall CH, et al. Insulin resistance syndrome in 8-year-old
Indian children: Small at birth, big at 8 years, or both? Diabetes 1999;48:2422-2429.
8. Sinha R, Fisch G, Teague B, et al. Prevalence of impaired glucose tolerance among
children and adolescents with marked obesity. N. Engl. J. Med. 2002;346:802-810.
9. Alberti G, Zimmet P, Shaw J, Bloomgarden Z, Kaufman F and Silink M. Type 2
diabetes in the young: The evolving epidemic: The International Diabetes Federation
Consensus Workshop. Diabetes Care 2004 July 1;27(7):1798-1811.
10. Krishnaveni GV, Hill JC, Fall C, et al. Anthropometry, glucose intolerance and insulin
concentrations in Indian children. Diabetes Care 2005;28:2919-2925.

37
Therapeutic Lifestyle Sonal V Modi
Hemraj B Chandalia

Lifestyle modification plays a crucial role in the management of a large


number of diseases. More specifically, it is an important tool in the management
of obesity, diabetes mellitus and hyperlipoproteinemia.
Lifestyle modification primarily uses two forms of therapy, diet and exercise.
Not infrequently and quite mistakenly, a variety of alternative therapies like
acupressure, acupuncture and magnetotherapy are considered lifestyle therapies.
Even Ayurvedic treatment is considered a lifestyle therapy when in fact only the
diet and exercise components of Ayurveda should be considered lifestyle therapy.
On the other hand, a large number of simple therapeutic measures like marital,
family and time management counseling to exalted spiritual and meditational
counseling can be considered lifestyle management. Lifestyle modifications
are often considered components of alternative therapy but modern medicine;
especially the complementary medicine includes lifestyle management. It is a
different matter that such therapies do not form an important component of
medical college curriculum, but most modern practitioners have their skills honed
in lifestyle management slowly over years, until they become quite accomplished
in this regard. To simplify, only two modalities of lifestyle management, nutrition
management and exercise will be discussed in this article.

Nutrition management
In most diseases, a holistic approach of prescribing a healthy diet is used. In
some diseases, further modification of the diet is called for. A healthy diet has a
balanced macronutrient composition. It derives 50-60%, 15% and 20-30% of calories
from carbohydrate, proteins and fats, respectively. It includes vegetables, fruits,
whole grain cereals and pulses as the main components of the diet. It includes
moderate amount of protein foods like milk or milk products, fish, eggs and

38
Management of Type 2 Diabetes

lean meat. It restricts fat and oil intake to a minimum amount. At the Diabetes
Endocrine Nutrition Management and Research Centre, Mumbai the pattern of
consumption of dietary fats in diabetics and non-diabetics was assessed. The total
fat intake stood at about 38% of the total calories. The saturated fat intake was
high (13-15% of fat intake) whereas the monosaturated fat intake was poor (5%
of fat intake). The protein intake was low, (12% of total calories). It is obvious
that saturated fat intake should be lowered and monounsaturated fat increased
in our population. The protein intake also needs to be enhanced.
A large number of important modifications can be made on this basic theme.
In obese subjects, overall caloric restriction will be important. The curtailment
of stable caloric intake by 500 kcal/day will induce a weight loss of about
one pound/week. In lipid problems, as recommended by NCEP III, a step 1
and step 2 diets can be used. In step 1 diet the fat intake makes 30% of the
calories; with saturated fat being <10% of calories. The cholesterol consumption
is restricted to 300 mg/day. The step 2 diet is recommends 7%, 13% and 10%
saturates, monounsaturates and polyunsaturates, respectively. The cholesterol
content of the diet should be limited to 200 mg/dl. The same diet is advised to
patients with atherosclerosis, angina or ischemic heart disease. In hypertension,
a low sodium, high potassium and high magnesium diet is prescribed. This diet
should contain plenty of fruits and nuts, which are known to be low in sodium
and high in potassium and magnesium. The diet in diabetes requires further
reduction of concentrated carbohydrates. The diet in hypertriglyceridemia also
requires restriction of fruits and simple sugars.

Implementation of a Diet
In an effort to assess the knowledge of diabetes, the attitude of patients
towards their disease and the barriers to adherence to the advice given; we
conducted a study in our patients at our center. Eighty patients were administered
a questionnaire comprising of questions on methods of counseling; perceptions,
attitudes and behavior; socio-economic factors and family support; locus of control
and empowerment. Other questions were related to adherence. Our patients
preferred detailed diet advice with exchanges and both verbal and written
diets for reinforcement. Positive family support enhanced adherence whereas
eating out diminished the same. Most people identified locus of control as self
or family members. The group, which was adhering to instructions, was highly
educated and showed better metabolic control of diabetes.

39
Therapeutic Lifestyle

In lifestyle management, faddistic diets should be avoided. Patients’


dietary habits, food preferences and food allergies must be taken into account.
The prescription should avoid monotony by educating the patient on food
exchanges.

Exercise therapy
Exercise in various forms can make an attractive lifestyle modification. As
in case of diet therapy, individualization is the key to success. The goals of
exercise therapy are promoting caloric expenditure without undue stress or
injury to musculoskeletal system. Hence the prescription has to vary according
to age, gender and different background of patients (Table 1).

Table 1. Rules regarding exercise therapy


 Exercise should be individualized
 Exercises should be escalated in a graded fashion
 The onset and offset of each exercise session should be slow
 Intensity, frequency and duration of exercise should be well defined

Those used to sports activities, especially the young adults can engage in
tennis, badminton, swimming or indigenous games like Kabaddi or Hu tu tu.
However, as age advances high impact sport should be minimized. Walking about
5 km/day in a span of 45 minutes is most suitable for middle-aged persons
and constitutes a physical fitness level exercise. Elderly people will do well on
a combination of walk and yogic exercises to maintain flexibility of joints. All
forms of exercises carried out appropriately are beneficial. The caloric expenditure
with these exercises and daily activities in shown in Table 2.

Table 2. Calories expended in various activities and exercises


Consumption
Activity listed in increasing order of caloric consumption of calories/min
Self care activities
 Resting, supine, sitting, studying, eating, dressing 1-2
 Showering, walking downstairs, walking upstairs 5-10
Recreational activities
 Driving a car, painting, bowling, golfing, dancing, gardening 2.5-5
 Swimming, cycling, playing tennis, playing squash, skiing 5-10
Household work
 Hand-sewing, sweeping floor, scrubbing, washing clothes, ironing, 2-5
mopping floor
Work activities
 Brick-laying, plastering tractor ploughing 4-5
 Carpentry, shoveling, mowing lawn, tending furnace 6-10

40
Management of Type 2 Diabetes

It is recommended that 1,500 calories be expended weekly in the exercise.


The frequency of exercise should atleast be 5 days a week. The intensity to
achieve should be 75% of maximal heart rate and sustainable at that level for
15 min/day. An increase in daily activities like walking to work or climbing
stairs can also constitute part of lifestyle modification. This can also increase
caloric expenditure.
Yogic exercises and pranayam are attributed with special benefits. This needs
to be proven conclusively. The Research Society for Study of Diabetes in India
has therefore launched a collaborative national study for prevention of type 2
diabetes where yogic exercise is an important treatment arm. The results of this
study will be made available after about 3 years.

Benefits of Lifestyle Modifications


Lifestyle management offers great therapeutic advantages with minimal cost
and side effects. It has been tried in many a disease, with spectacular results.
The lifestyle management program by Dean Ornish’s method produced
remarkable improvement in ischemic heart disease1. In any weightreduction
plan, lifestyle modifications are essential, because drug therapy of this common
metabolic problem is as yet inadequate. Lifestyle modifications produced about
58% prevention of type 2 diabetes in atleast three landmark studies2-4 and
improved obesity control in type 2 diabetes5.
Although we often talk of pleiotropic effects of many drugs like statins,
we do not view lifestyle management as a superb modality of treatment for
all-pervasive, pleiotropic effects. Amongst diabetics, about 30% suffer from a
multi-metabolic syndrome consisting of obesity, hypertension and hyperlipidemia.
Many authors talk of a polypill to combat this multiplicity of risk factors.
However, lifestyle management can achieve this quite effectively. The magnitude
of lipid lowering effect or blood pressure lowering effect is considered modest,
but a sum total of benefits accrued is substantial. The side effects of this form
of therapy is minimal.
It is often argued that adherence to lifestyle management is poor. This is true
to some extent, but it will not be correct to abandon the same for this reason.
If this form of therapy is pursued with the same tenacity and conviction as one
applies while prescribing a statin for hyperlipidemia or aspirin for ischemic heart
disease, many a patient will adhere to the program. In obesity, usual lifestyle
management is associated with a high relapse rate of 90% within 2 years.

41
Therapeutic Lifestyle

However, an evaluation, education and empowerment plan at our center showed


only a relapse rate of 23% within 27 months. This was possible by intensive
interaction, individualization of the plan and patient education.
We also need to research more into the behavioral sciences to study the
factors which will promote adherence to such programs.

References
1. Ornish D, Brown SE, Scherwitz LW, et al. Can lifestyle changes reverse coronary
heart disease? Lancet 1990;336:129-133.
2. Toumileheto J, Lindstrom J, Eriksson JG, et al.; Finnish Diabetes Prevention Study
Group. Prevention of type 2 diabetes mellitus by changes in lifestyle among subjects
with impaired glucose tolerance. N. Engl. J. Med. 2001;344:1343-1350.
3. Pan XR, Li GW, Hu YH, Wong JX, et al. Effects of diet and exercise in preventing
NIDDM in people with impaired glucose tolerance. The Da Qiang IGT and diabetes
study. Diabetes Care 1997;20:537-544.
4. Diabetes Prevention Program Research Group. Reduction in the incidence of type 2
diabetes with life style intervention or metformin. N. Engl. J. Med. 2002;346:393-403.
5. Wolf AM, Conaway MR, Crowther JQ, et al. Translating lifestyle intervention to
practice in obese patients with type 2 diabetes: Improving Control with Activity and
Nutrition (ICAN) Study. Diabetes Care 2004;27:1570-1576.

42
Team Work is What we Need Rishi Shukla

Diabetes mellitus is a disorder where self dependence of the individual


plays major role in disorder management. It has been observed that education
plays significant role in making life comfortable compared to other disorders.
The meaning of care is that the life of people with diabetes should be as normal
as of a non-diabetic individual. This includes aspects like education, lifestyle
changes, metabolic treatment, screening and treatment of complications, as well
as psychological support. The ultimate goal is to improve health-care to the tune
that people with diabetes should live life near to normal. The above mentioned
statement is not an imagination but may be converted into reality. This day
dreaming is not possible without team work.
Let us look at “Diabetic Team” what it comprise of - the team consists of
doctor(doctors), nurses, dieticians and chiropodists. The extended part of the
team includes psychologists, social workers, ophthalmologist, administrative
staff, family and friends. Therefore the team is very big. Is this a practical or
hypothetical team. The type of team we have discussed is only seen in big
hospitals abroad.
Before discussing team let us see what are the existing “Diabetes care”
facilities in our country. The service providers in our country may be divided into
two broad groups: 1) Government sector and 2) private sector. The government
sector may further be divided into: a) Central institutes, b) medical colleges
c) district hospitals and d) primary health centers. The private sector may be
divided into: a) Corporate hospitals, b) smaller hospitals and C) individual
practicing clinicians. Most of the institutes have dieticians however other facilities
are variable. Most of the medical colleges usually do not have even dieticians
however they have force of residents to take care the various aspects of the
diabetes care. The district hospitals and primary health centers usually have
the availability of clinicians. The private sector is better equipped with respect
to the facilities however patients have to pay for the services. Due to their
pay structure, the services are not available to whole society. Majority of the

43
Team Work is What we Need

corporate hospitals have dieticians, psychologists and educators. There are many
good specialized hospitals in the country dedicated to diabetology and they have
practically everything as in any other good center around the world. The services
available with individually, practicing clinicians have lots of variations. Some of
the clinicians feel they are capable to render all sorts of services without any
help and they do not need any paramedical assistance. Some of the practicing
physicians have dieticians to assist them.
Diabetes is the disorder that has maximum dietary restrictions compared to
other diseases. The usual feel in the society is that if they can not eat what is
fun in living. People feel dejected and depressed as they perceive that they have
to eat bare minimum. The role of dietician is very significant to give people
with diabetes a feel that they are eating almost normal food. Many clinicians
give printed diet charts of different calorie values to be followed by people with
diabetes. This is the practical experience that these charts stay with prescription
files only without being understood and followed. The real role of dietician is
to make people with diabetes understand that what they are eating is good and
near normal food. The type of information that is required to be given just can
not be given by single handed physicians.
The dieticians are most important constituent of the team and it is almost
impossible to provide genuine advice without them. For those clinicians who
have lesser role for dieticians they may use them as dietician cum educator.
The time has come when diabetic educators in our country should be given
due importance. It is impossible to make people with diabetes independent
without the help of educators. Of those taking antihyperglycemic agents 16.6%
feel that their treatment is too complicated and 33% people with diabetes are
tired of complying with their medications. The distress related to diabetes is high
at the time of diagnosis. People have fear of complications. Majority of Indians
feel that they will not be able to take care of their families as the life span is
going to be short. The usual sentence is “I want to live for my children not for
myself’. These all issues may well be taken care of by the diabetic educators. The
hard fact is that majority of practicing clinicians are so busy that they just can
not offer other than medical prescription. This is also common thought among
clinicians that if people with diabetes are made independent, their utility would
be reduced along with reduction in frequency of clinic visits.
Chiropodist is still a name with surprise in our country. The most of diabetic
foot are being handled by surgeons who have are too busy in surgical practices.
Therefore interest is automatically diluted. There are not many chiropodist in

44
Management of Type 2 Diabetes

the country. Thus the clinicians interested in foot care can provide a baseline
training for foot awareness to their paramedical staff so a lot of morbidity
may be avoided. It is still a dream of our diabetic population to take services
of chiropodist.
More than 60% of diabetic population (type 1 + type 2) have psychological
burdens. People with diabetes feel burnt out because of diabetes. Some of the
patients feel diabetes is preventing them from what they want to do. Diabetes
gives lot of insecurity about financial future of patients. These all issues may be
handled nicely by the psychologist. Many of our patients do not have money to
visit doctor or purchasing medicines so what is the fun of psychologist. As we
know we have heterogenous population and a subset of the population needs
help. If clinical psychologist is available we can send some of our patients even
if he/she is not the part of the team.
Role of the family is always there without any geographical distribution. The
family system is very strong in our country so role becomes very significant.
Possibly, we are not taking full advantage of our family values in handling
diabetes. Morale boosting is very important component in diabetes management.
It becomes all the more important for people with type 1 diabetes. The part of
psychological support may very well be provided by close family members. If
through educator/psychologist/clinician some counseling is provided to spouse,
a lot of morbidity may be avoided.
Type 1 diabetes is different subset of people and they need entirely different
dimension of efforts. Each of type 1 diabetic friend has to complete their schooling,
college, struggle for the jobs and fighting for the marriage with diabetes. Not
only to srruggle but at the same time they have to control their diabetes to make
themselves complication free. The separate meetings and camps are invaluable
for them. The senior friends with type 1 diabetes play as role model to many
younger children and their parents. The fiiendship that they develop during
such meetings is very beneficial. We have organized a 2 days meeting “National
type 1 diabetes meet“ at Kanpur. And people from almost whole country have
participated. We covered lots of medical and social issues including hot topic
like marriage and type 1 diabetes. It brought lot of confidence and will power
to type 1 friends and their families.
As we know India is going to be the future world capital of diabetes.
It is the responsibility of all of us to make general population aware of diabetes
and related problems. By doing this not only we can control diabetes but also
save people from the complications. We can also offer some sort of primary

45
Team Work is What we Need

prevention through awareness. It is our experience that if education is discussed


in group with simultaneous blood sugar monitoring, the motivation levels are
very high. The bigger group (group of approx. 50) discussions are very fruitful
because learning and encouragement is maximum when you see somebody
doing well. If others can control and be complication free why not me, is a
great booster to many. The various non-government organizations can also play
very important role by regularly organizing awareness programs. We have been
organizing regular monthly awareness program (NGO by the name of society for
prevention and awareness of diabetes-SPAD). It is our observation that regular
programs bring out better control and drastically reduce anxiety levels among
people with diabetes. If we want to save people from developing diabetes and
making people with diabetes complication free, we will have to seek help of
various similar organizations dedicated for the help of the society.

Suggested readings
1. Skovlund SE, et al. Diabetes attitudes wishes, wishes, and needs (DWAN) program.
Diabetes Spectrum 2005;18(2).
2. Diabetes care at diabetes camps. Diabetes Care 2006 Jan.;29(Suppl. 1):S56-S68.
3. Pickup JC and Williams G (Eds.). Text Book of Diabetes Third Edition 2003.
4. Thorn PA, et al. Diabetic clinics today and tomorrow. BMJ 1973;2:534-536.
5. Recommendations for the structure of specialist diabetes care. Diabetes UK 1999.
6. Greeenhalagh PM. Shared care for diabetes, a systemic review. Occas. Pap. R. Coll.
Gen. Pract. 1994;67:i-viii, 1-35.
7. Clark CM Jr. The National Diabetes Education Program. Changing the way diabetes
is treated. Ann. Int. Med. 1999;130(4):324-326.
8. Greco PJ and Eisenberg JM. Changing physicians’ practices. NEJM 1993;329:
1271-1273.

46
Oral Hypoglycemic Agents -
Present Status Debashish Maji

Use of oral antihyperglycemic agents in the treatment of type 2 diabetes


mellitus (DM) is increasing day-by-day. This increase is mainly due to three
reasons: 1) Increase in the prevalence of diabetes, currently estimated at around
5-6% per year, 2) aggressive treatment of type 2 diabetes after the publication
of UKPDS results and 3) increasingly demanding guidelines for glycemic
control. As a result, 70% of type 2 DM patients are on two oral agents and
10% are on three durgs1. However, the approach to treat diabetes with oral
agents has changed because of better understanding of the pathophysiology
and the course of the disease.
Type 2 diabetes is the end result of a process that involves progressive
loss of B-cell function. The initial phase (7-12 years) of this process in the
majority of the patients is the development of insulin resistance, resulting
in compensatory increased insulin secretion and hyperinsulinemia. Next
stage is the loss of 1st phase insulin release resulting in postprandial
hyperglycemia i.e., the stage of impaired glucose tolerence (IGT). As
pancreatic B-cell function continues to deteriorate, fasting hyperglycemia
ensures. Most patients would require oral antihyperglycemic agents in
these years (7-12 years); finally many patients will become severely
enough insulin deficient and would then require insulin to maintain
euglycemia. So drug treatment of type 2 DM would depend upon the stage
of the disease2.
Advances in type 2 DM management have generated considerable interest
for oral glucose lowering agents that target different pathophysiologic process
in type 2 DM3.
Different oral agents which are in current use can be divided into mainly
three groups.
1. Those that decrease insulin resistance (e.g., metformin, thiozolidinediones).
2. Those that increase insulin secretion (e.g., sulphonylureas, meglitinides).
3. Those that delay glucose entry from the gut (a-glucosidase inhibitors).

47
Oral Hypoglycemic Agents - Present Status

1. Those that decrease insulin resistance includes


metformin and thiozolidinedions (rosiglitazone and
pioglitazone).
Biguanides
Phenformin and metformin are the two biguanides available in India since
1957. Phenformin use declined as it was implicated in the UGDP study to produce
lactic audosis in 1970. Inspite of their being banned in USA in 1970 both metformin
and phenformin was available in India. Metformin was available in many other
countries and came to limelight again after its favorable reports in UKPDS study.
Its approval was finally granted by the Food and Drug Administrations (FDA) in
1994, as a new drug. Metformin thus, is a representative of this class of biguanides
and is now a widely used drug all over the world.
Metformin hydrochloride is a stable hydrophilic biguanide and is quickly
absorbed and quickly excreted unchanged in the urine. Thus, it is well appreciated
that metformin is only prescribed to patients with sufficient renal function to avoid
accumulation of the drug. There is little binding of metformin to plasma proteins,
so metformin does not interfere with protein bound drugs.
Metformin acts partly by improving insulin action and partly by its effect
that are not directly insulin dependent. It reduces hepatic neoglucogenesis mainly
by increasing sensitivity to insulin. Insulin stimulated glucose uptake and glycogen
formation in muscle are enhanced by metformin4.
To summerise, metformin reduces fasting plasma glucose and insulin levels,

Metabolic and Other Effects of Metformin


Antihyperglycemic Suppresses hepatic glucose output
Increases insulin-mediated glucose use
Decreases fatty acid oxidation
Increases splanchnic glucose turnover
Weight control Weight stabilization. Can assist weight
reduction
Improved lipid profile Reduces hypertriglyceridemia. Lowers
NEFAs, lowers LDL-C and raises HDL-C
in some patient
Decreased hyperinsulinemia Decreases basal insulinemia
Not cause of serious hypo- Serious hypoglycemia very unlikely with
glycemia monotherapy
Counters insulin resistance Decreases the endogenous/exogenous
Insulin requirement
Vascular effects Increased fibrinolysis. Decreased PAI-1
NEFAs: Non-esterified fatty acids; LDL-C: Low-density lipoprotein cholesterol;
HDL-C: High-density lipoprotein cholesterol; PAI-1: Palsminogen activator inhibitor-1.

48
Management of Type 2 Diabetes

improves lipid profile, and promotes modest weight loss. It is the most suitable
agent of monotherapy in a newly diagnosed over weight type 2 diabetic
patients. The initial starting dose is 500 mg o.d. or b.d.. And can be increased
over a period of 3 weeks time to 1,000 mg b.d. An extended release form is
available. Combination formulation with glyburide, gliclazide and glimiperide
can also be given in selected patients when blood sugar is not controlled with
metformin alone5.
Metformin should not be used in patients with renal insufficiency (serum
creatinine >1.5 mg/dl); any form of hypoxia, congestic cardiac failure or liver
disease. Metformin should be discontinued in severely ill patients who cannot
take it orally. In general, metformin is well tolerated, some may develop nausea,
vomiting, upper abdominal discomfort, diarrhea, which can be minimized by
gradually building up the dose.

Thiozolidinediones
Thiozolidinediones are agents which improve insulin sensitivity by stimulating
nuclear receptor, peroxisome proliferator-activated receptor gamma (PPAR-g). They
are also referred as PPAR-g agonist, glitazones or insulin sensitizers. Troglitazone
was the first of its kind to be used clinically in 1997, but withdrawn soon in UK
(1999) and later in USA in 2000, because of its hepatotoxicity.
Rosiglitazone and pioglitazone (Table 1) are not been associated with such
severe side effects and are available for clinical use6,7.
Both of these agents are extensively metabolized in liver. Glitazones promote
adipocyte differentiation, reduce insulin resistance, circulating insulin level decreases.
The two glitazones have almost similar efficacy. Therepeutic range for pioglitazone
is 15-45 mg/day in a single dose and for rosiglitazone 2-8 mg/day once or twice

Table 1. Pharmacokinetic features of the thiazolidinediones


rosiglitazone and pioglitazone
Rosiglitazone Pioglitazone
Time to peak plasma Approximately 1 hour <2 hours
concentration
Plasma protein-bound >99% >99%
Hepatic metabolism Mainly by CYP2C8 to Mainly by CYP2C8
several weakly active and CYP3A4 to active
metabolites metabolites
Elimination t½ Approximately 3.5 (100- 3-7 (16-24)* hours
150)* hours
Elimination Mainly urine (>60%) Mainly bite (>60%)
*Values in parentheses include metabolites.

49
Oral Hypoglycemic Agents - Present Status

a day. Thiozolinideones raise low-density lipoprotein and high-density lipoprotein


slightly and lower triglycerides by 10-15%.
They are associated with minor weight gain, a small reduction in hemoglobin
concentration and mild increase in plasma volume. Peripheral edema may cause
some problems in some patients forcing them to withdraw the drug. They are
contraindicated in patients with liver disease and congestive cardiac failure.

Indication and contraindications


Rosiglitazone and pioglitazone are available for use as monotherapy in non-
obese and obese type 2 DM patients inadequate controlled by lifestyle measures.
They should be tried for monotherapy when metformin is not appropriate in
a given case. They can also be used in combination with other agents (like
sulphonylureas, metformin or a-glucosidase inhibitors) or insulin. The general
principles for initiation of treatment with glitazones are same as those for other
group of oral agents with special precaution to look for congestive heart failure
and chronic liver disease where they are contraindicated hemoglobin should be
checked and reduction of 1 g hemoglobin during the coarse of treatment may
occur and one need not withdraw the drug. Liver function should be checked
by measuring alanine transaminase level- before and 6 weeks after the start
of treatment. Pre-existing liver disease or high alanine transaminase level are
contraindications.
By and large hepatotoxicity has not been a major problem for their use
(both rosiglitazone and pioglitazone). The starting dose should always be low -
2 mg rosiglitazone o.d. or b.d. or 15 mg pioglitazone once a day. Dose is titrated
every 4 weeks till 8 mg of rosiglitazone and 45 mg of pioglitazone and atleast
for 3 months are to be.
Combination with metformin and triple combination with metformin and
sulfonylurea with pioglitazone and rosiglitazone are effective and it increases the
compliance of the patient to the treatment regimen.

2. Those that increase insulin secretion from


the b-cells of pancreas, the insulin secretagogues,
they are
Sulphonylureas (chlorpropamide, glibenclamide, glipizide, gliclazide and
glimiperide) and miglitinides (repaglinide and nateglinide).

Sulphonylureas
Sulphonylurea have been the mainstay of oral treatment for type 2 DM
patients for the past 40 years. This class of drugs (Diagram 1) stimulates insulin
secretion through a direct effect on the B-cells of the pancreas, by binding to the

50
Management of Type 2 Diabetes

Diagram showing the mechanism of action of Sulphonylureas

Glucose

Sulphonylureas
GLUT-2 Repaglinide
Succinate esters
Nateglinide
Glucokinase SUR1
Kir 6.2
K+ ATP
Glucose channel
metabolism ATP
Proinsulin K+
biosynthesis Depolarization

PKA Ca2+ - Sensitive Ca2+


proteins channel Ca2+
Adrenergic
receptors
CAMP Insulin

a2-antagonists Exocytosis

Receptors

GLP-1 PDF Insulin


and its analogs inhibitors

sulphonylurea receptors on these cells and close the ATP sensitive potassium-
channels, reducing the potassium efflux and favoring membrane depolarization, so
that the voltage dependent calcium channels are opened, increasing calcium influx,
raising intracellular concentration of calcium and activating calcium-dependent
proteins that control the release of insulin granules, leading to a prompt release
of pre-formed insulin granules adjascent to the plasma membrane8.
The increased release of insulin continues as long as drug stimulation
continues, provided b-cells are functionally capable. They can cause hypoglycemia
because they can stimulate the b-cells of the pancreas and release insulin when
glucose concentration are below normal.

Indication and contraindication


Sulphonylurea have remained the most widely used first choice oral agent for
type 2 DM patients who could not maintain euglycemia with life stype measures.
They are preferred for patients who are not over weight since they are known

51
Oral Hypoglycemic Agents - Present Status

to cause weight gain. They can be combined with any other oral agents who has
different mode of action (metformin, glitazones or a-glucosidase inhibitor).
Sulphonylurea can be continued after initiation of insulin therapy at bedtime
or before dinner in type 2 DM. Selection of a sulphonylurea in a given case
is important. The duration of action and route of elimination of a particular
sulphonylurea may be important consideration. Long acting sulphonyureas are
not a good choice for elderly people for the risk of hypoglycemia. Short acting
preperation and those without active metabolites are preferred for individuals at
risk of hypoglycemia. Starting dose should be small and dosage is increased every
4-6 weeks till the maximum allowance dose is used for the particular drug. Long-
term glycemic control is checked by HbA1C. If the glycemic control is not achieved,
addition of another class of agent may be added and glycemic status monitored.
A third group of drug may be added and tried in the subsequent period of
time. If adequate glycemic control is not achieved after the use of combination of
sulphonylurea with metformin, glitazones and a-glucosidase inhibitors, it is quite
likely that the natural history of type 2 DM has progressed to a state of severe
b-cell failure and insulin may be considered. At this stage the treating doctors
clinical skill is tested, how easily one can convince the patient and make him or
her as the partner in the therapeutic program of diabetes management where
insulin injection becomes a necessity.

Adverse effect
Hypoglycemia is the most common and serious adverse reaction with
sulphonylurea, patients should be properly educated to detect and treat hypoglycemia
events. Hypoglycemia is more likely to occur in patients with good glycemic control,
and with chlorpropamide (fasting hypoglycemia) and glybenclamide (interprandial
hypoglycemia). Hypoglycemia is less likely to occur in patients taking short acting
and slow-release formulations and those get a regular timely food intake. In the
UKPDS study, 20% patients treated with sulphonylureas reported one or more
hypoglycemia events in a year, though only 1% had severe hypoglycemia who
required another person’s assistance or medical intervention to treat it9.
Other rare adverse reactions with sulphonylureas are, drug interactions,
sensitivity reactions, precipitation of acute porphyria. Weight gain is a recognized
features of sulphonylurea therapy a gain of 1-4 kg is common which stabilizes
within a period of 4-6 months.

Non-sulphonylurea insulin secretagogues


Meglitimides produce a rapid but short lived stimulation of insulin secretion from
b-cells and can be taken immediately before meals Repaglimide and nateglinides are

52
Management of Type 2 Diabetes

the two molecules available for clinical use. These agents differ from sulphonylureas
in their structure and most importantly in their pharmacokinetics. Meglitinides are
rapidly absorbed and rapidly metabolized, producing prompt and short lasting
insulin release. The mechanism of action is same as that of sulphonylureas10. The
advantage is they can be taken before meals to prevent postprandial rise of blood
glucose and the disadvantage is its prohibitive cost.

3. Those drugs that delay glucose entry from the gut


a-glucosidase inhibitors
Inhibitors of intestinal a-glucosidase enzymes slow the rate of carbohydrate
absorption and provide one means of reducing postprandial hyperglycemia. Acarbose
was the first a-glucosidase inhibitor introduced in the early 1990s and recently two
more molecules miglitol and voglibose having similar mechanism of action are
available. Acarbose and voglibose are minimally absorbed in the gut, but miglitol
is almost completely absorbed and eliminated unhanged in the urine11.

Mechanism of action
a-glucosidases inhibitors competitively inhibit the a-glucosidase enzymes
located in the brush border lining of the intestinal villi. The main a-glucosidases
are glucoamylase, binds to there enzymes and thus preventing them from
cleaving their normal disaccarides and oligosaccarides subrates into the absorbable
monosaccharides. The varying affinity with which the inhibitors bind to the
enzymes give them slightly different activity profiles. The degree of affinity to
the enzymes for acarbose with decreasing order are glycoamylase, sucrase maltase
and dextrines. Miglitol is a potent inhibitor of sucrase, a-glucosidase inhibitors
thus defer the completion of carbohydrate digestion, until the substrate is further
along the intestinal track which reduces the height of postprandial blood sugar
peak. So they must be taken with meals containing digestible carbohydrate. They
do not affect the absorption of glucose. In hypoglycemia attacks in type 2 DM
patients taking a-glucosidase inhibitors, glucose should be given orally, not sweets
containing sucrose (Table 2).

Indications and contraindications


Because a-glucosidase inhibitors reduce postprandial hyperglycemia, they are
useful as monotherapy for those patients who are having near normal fasting plasma
glucose, but having high postprandial hyperglycemia. They can also be used as
combination with any other oral agents where postprandial blood sugar remains
high and with meals containing higher carbohydrate content. Limitations of their
use when someone has intestinal disorders or getting abdominal symptoms with

53
Oral Hypoglycemic Agents - Present Status

Table 2. Dosage and pharmacokinetics features


of the α-glucosidase inhibitors
Amount Plasma Elimination of absorbed
Dosage absorbed protein-bound drug and metabilites
Acarbose Upto 3*100 mg/day <2%* — Urine
Miglitol Upto 3*100 mg/day >95%† Negligible Urine
Voglibose Upto 3*5 mg/day <5% — Urine
•About 30% absorbed as metabolites produced by intestinal bacteria.

Where doses are submaximal, <70% of a maximum dose is absorbed.

these drugs. Patients who are having gastrointestinal symptoms with metformin
are not good candidates for a-glucosidase inhibitor use. Acarbose should not be
used in patients with chronic liver disease12.

Conclusion
Oral hypoglycemic agents are increasingly used in clinical practice
because of the increase in the prevalence of type 2 DM. Aggressive treatment
of type 2 DM to prevent long-term complications of diabetes and increasingly
demanding guidelines for glycemic control laid down by several international
bodies. From the natural course of type 2 DM (Diagram 2) it seems that after the
first few years of diabetes control by lifestyle measures, oral hypoglycemic agents
will become essential. For overwight diabetics; metformin is the ideal drug to
start with, though glitazones, sulphonylureas and a-glucosidase inhibitors may be
considered as monotherapy in a given case. During the course of time these drugs
may be combined in two or three to get the synergistic effect. Even combination
pills may increase compliance of a patient who takes a lot more drugs for other

Insulin

a-glucosidase inhibitors

Glitazones - metformin

Lifestyle Insulin secretagogues

IGT Type 2 DM

↑ PPBS ↑ FBS

Insulin resistance Beta cell failure

Figure 2. Natural history of type 2 DM and its management.

54
Management of Type 2 Diabetes

associated condition like hypertension, dyslipidemia, ischemic heart disease and so


on. After 10-15 years of diabetes; many of the type 2 DM patients will have severe
b-cell insufficiency to call for insulin therapy to maintain euglycemia. Judicial use
of antihyperglycemic agents, patient involvement, dietary regulation and patient
education are the key factor in the success of diabetes management.

Addendum from Editor


Since the receipt of the article one more medication is approved in USA and
UK for treatment of type 2 Diabetes and going to be available in our country
soon. The incretin effect, defined by a significantly greater insulin stimulatory effect
evoked after an oral glucose load than that evoked from an intravenous glucose
infusion when plasma glucose concentrations are matched, was first described in
the 1960’s. The majority of the effect is thought to be due to glucose-dependent
insulintropic peptide (GIO) and glucagons like peptide-1(GLP-1). Patients with type
2 diabetes have a significant reduction of the incretin effect, implying that these
patients either have decreased concentration of the incretin hormones (GLP-1) or
a resistance to their effects (GIP), thus making GLP-1 a more logical target for
therapeutic intervention. Incretins are degraded by an enzyme called as Dipeptiy1
peptidase (DPP)-IV.
DPP-IV inhibitors thereby will increase levels of increatins and hence a
promising approach to treat type 2 diabetes. Sitagliptin (Jannuva – dose 100 mg
once daily) is the first one available today, Vildagliptin (Glavus) is going to be
available soon.

References
1. Hauber A and Gale EAM. The market in diabetes. Diabetologia 2006;49:247-252.
2. Dostou J and Gerich J. Pathogenesis of type 2 diabetes mellitus. Exp. Clin. Endocrinol.
Diabetes 2001;109(Suppl. 2):s149-s156.
3. Lebovitz HP. Oral therapies for diabetic hyperglycaemia. Endocrinol. Metab. Clin. N.
Am. 2001;30:909-933.
4. Kirpicnikov D. Metformin: An update. Ann. Intern. Med. 2002;137:25.
5. Inzucchi SE. Oral antihyperglycemic therapy for type 2 diabetes. Scientific Review.
JAMA 2002;287:360-372.
6. Balfour JA and Plosker GL. Rosiglitazone. Drugs 1999;57:921-930.
7. Gillies PS and Dunn CJ. Pioglitazone. Drugs 2000;60:333-343.
8. Aschroft FM. Mechanisms of the glycaemic effects of sulphonylureas. Horm. Metab.
Res. 1996;28:456-463.
9. Rendell M. Role of sulphonylureas in the management of type 2 diabete mellitus.
Drugs 2004;64:1339-1358.
10. Lebovitz HE. Insulin secretagogues old and new. Diabetes Res. 1999;7:139-153.
11. Lebovitz HE. Alpha-glucosidase inhibitors as agents in the treatment of diabetes.
Diabetes Rev. 1998;6:132-145.
12. Lebovitz HE. Management of hyperglycemia with oral antihyperglycemic agents. In:
Type 2 Diabetes: Joslin’s Diabetes Mellitus 14th Edition 2005:687-710.

55
Insulin Therapy Bipin Sethi

By definition diabetes mellitus is a disorder of intermediary metabolism


charecterized, by chronic hyperglycemia due to relative or absolute insulin
deficiency accompanied by varying degree of insulin resistance. The therapy of
diabetes mellitus, the adequacy of which is judged by glycemic control generally
revolves around correcting these two basic pathophysiological mechanisms
namely the insulin deficiency and insulin resistance.
In type 1 diabetes mellitus the situation as far as therapeutic decision
making is concerned is simple, all patients need insulin and the contentious
issue is the regimen. The situation in diabetes type 2 mellitus is more complex.
Depending on the degree of insulin deficiency some patients may respond to
lifestyle modifications alone atleast initially while others require additional
administration drug treatment. The later category of patients have the choice
of oral antihyperglycemic drugs (OADs) or insulin. OADs act by enhancing the
insulin secretion or action, but depend on the availability of endogenous insulin
which is declining variable, with passage of time the available insulin so little
that OADs cannot control the blood glucose at this time insulin is eventually
necessary. OADs also have a finite range of efficacy and even at presentation
when blood glucose levels are very high and beyond the range of efficacy, that
insulin is needed and administered. All things considered both the physician
and the patient are comfortable with the OADs and often the goal of euglycemia
is overlooked for this comfort and newer agents are tried and added when the
OADs have truly outlived their utility and the patient is exposed to the hazards
of hyperglycemia.
In view of fact that type 2 diabetes mellitus is asymptomatic and is diagnosed
late, complacence after detection in approaching glycemic goals amounts to
negligence. Discussion of pathophysiology of disease with emphasis on key role of
insulin deficiency in not only causation but worsening of hyperglycemia whenever
the patient is seen and alluding to the eventual need of insulin can go a long way
in improving patient acceptance to insulin when it is eventually needed.
Before the nitty-gritty of insulin is discussed it is worth remembering that
the endogenous insulin secretion has two components the basal, which is the
insulin present 24 hours a day and the bolus which is the spike in insulin levels

56
Management of Type 2 Diabetes

in response to any meal. The latter is short lived and is precise in time and
quantity, the ideal insulin regimen should provide both these but like many
things in nature is difficult to replicate in practice for all patients.
For many years the only option of supplying bolus component was by
injecting short acting insulins which were to be taken half an hour prior to the
candidate meal because of the time required by aggregated hexamers insulin
molecules dissociate and form monomers which are then absorbed. Apart from
this delay in the onset of their action the duration of action is longer than
desirable. These two fallacies mandidate waiting for the meals after the injection
and the tendency for hypoglycemia in between meals, rapid acting analogs
attempt to overcome defects (Lispro, Aspart, glulisin) wherein bioengineered
molecules not only dissociate faster and peak earlier but also have shorter half
lives. Similar advancements have occurred in basal insulin, the basal insulin
supplements till recently were limited to isophane and insulin zinc suspensions
the later were lente and ultra lente. Ultra lente was the longest acting one
but had unpredictable time course of action and was never popular. Isophane
(NPH) and lente insulin have been widely used, the former provides the basal
component in all premixed insulin. The major problem with conventional basal
insulin are:
l Their tendency to peak
l Their unpredictable duration of action and inability provide 24 hours
cover.
Bioengineered insulin (Glargine, detemir) are able to address these issues
to some extent, atleast they do not peak. Detemir is supposed to be more
predictable whereas Glargine has longer duration of action providing a
20-24 hours coverage.
In designing a regimen one has to take in to consideration the following
factors:
a) The degree of insulin deficiency (or the amount of available endogenous
insulin). The need of flexibility in meal plan (timing and quantity)
b) The extent of patient commitment and participation
c) The cost of the chosen insulin.
In patients who have just lost control with OADs and fasting plasma glucose
is <150 mg/dl, addition of a single dose of basal insulin at night might suffice
to provide a decent fasting plasma glucose and as long as OADs are working,
postprandials will be under control with OADs that have been continued in this
scenario. When postprandial control becomes unacceptable or in another patient
who has very high blood glucose levels of above 250 mg/dl, basal insulin alone

57
Insulin Therapy

will be insufficient and additional bolus insulin will be necessary, this can be
achieved by continue the basal insulin with addition of premeal bolus insulin
(Basal bolus regimen or multiple subcutaneous injection or basal insulin along
with bolus insulin can be injected once or twice daily (split mixed regimens)
the mixing being either done by the manufacturers in the bottle (premixed). The
problem is that while the basal bolus regimen is more physiological permitting
flexibility and achieving better control it entails more number of insulin shots
needs when this is not possible monitoring and motivation from recipient, often
on settles for twice daily insulin injection taking the basal and bolus insulin, the
basal component covering the lunch of the morning and basal component of the
evening covering the fasting sugars whereas the rapid acting component covers
the breakfast and the dinner. The patient from the morning and evening has to
take identical food quantity for lunch on time to avoid hypoglycemia and peaking
of basal component at night makes them prone to nocturnal hypoglycemia. It
is clear from the foregoing that it is better for patients to have control rather
than convenience and more severe the insulin deficiency the better taken care
of if patient is on more number of shots.

Dose and species


These two issues have bogged the physicians for long. The issue of
dose can never be settled for simple reason that there is wide variability
amongst individual and same degree of hyperglycemia can have different dose

Control Convenience Flexibility Cost


Single daily + +++ - +
injection
Split mixed ++ ++ - ++
insulin
Multiple +++ +* +++ +++
subcutaneous
injections basal
bolus
*Convenience can be enhanced by pumps but cost is extremely prohibiting.

requirements. All suggestions are for initiation and one has to build up on that
based on individuals responses, titrating the basal components by looking at
the fasting or predinner glucose values and the bolus components by looking
at the postmeal glucose values. By rough rule anyone being given replacement
doses (i.e., split mixed or basal bolus regimen) needs to begin with 0.6-0.7 U/
kg/day whereas those of single dose regimen may start with 0.2 U/kg or flat
dose of 10 units.

58
Management of Type 2 Diabetes

Luckily the issue of insulin species that has generated more heat than
light is dying, with manufactures of animal insulin closing shop. The availability
of unlimited amounts of human insuliln by bioengineering has made this possible,
only if it is matched by slashing the animal insulin the issue of cost can be
buried for good.

When to give insulin therapy in type 2 diabetes mellitus?


In conventional paradigm patient with type 2 diabetes mellitus are offered
OADs after failure of diet and exercise and only when all the available option
for OADs been exhausted despite compliance with lifestyle changes is insulin
considered. Proponents of early insulin therapy feel that this is incorrect,
considering that insulin deficiency is the key to development of diabetes, insulin
therapy can be given much earlier. How early this should be is not clear though
theoretically it could be at diagnosis, after failure of one or two OADs. It is
proposed that the b-cell rest that ensues smoother control and decreases the chances
of hypoglycemia. Though attractive this proposition is not acceptable because
of lack of evidence base proving these points that the modality of euglycemia
treatment is more important than the agent chosen to achieve it besides the lack
of patient acceptance is another issue.
The only time it might be considered is a symptomatic type 2 patient in
whom after initial stabilization control with lesser and lesser doses of insulin and
eventual withdrawl of the same may be possible control is being achieved with
a small dose of OHAs or diet alone.
Awareness about different regimens is useful but one should individualize
treatement and strive for targets within the constraints posed by the cost of
monitoring and therapy and the extent of patient participation.
Good control is possible in all settings, the reason for poor control is nihilism
lack of enthusiasm as much on the part of physicians as on the part of the patients
who also experiences the phenomenon of disease burnout.

Addendum from Editor


Since the receipt of the article one more injectable medication -Incretin analoge
(not Insulin) is available with us for treatment of type 2 Diabetes.
The incretin analogue - Exanatide is available now which has to be taken as
an ijeciton (initial dose 5mcg twice daily, then to increase to 10mcg twice daily).
the beeficial effects of these drugs would be that hypoglycaemia would be less
frequent, as they release insulin in response to a meal and they also help in
weight reduction. The disadvantage of exanatide would be that it has to be used
as an ijection and its cost. A similar molecule in the same group, Liraglutide is
in Phase III clinical trials now

59
Diabetes in Hospital Menka Ramprasad
Apandev Bhattacharyya

Diabetes in hospital: It is important


The prevalence of diabetes mellitus (DM) is increasing worldwide and more
so in India. In our hospital, in a cross sectional survey, we have found the
prevalence of diabetes to be 39% (6% are newly diagnosed)1. It is wellknown
that people with DM get admitted in hospital more commonly for reasons other
than diabetes.
Diabetes control in hospital in hospitalized patients irrespective of reason
for admission improves outcome. DIGAMI (Diabetes Mellitus Insulin Glucose
Infusion in Acute Myocardial Infarction) study examined the importance of early
and aggressive insulin infusion to lower glucose levels to near normal range
regardless of prior diabetic status in patients admitted with acute myocardial
infarction2.
Pomposelli, et al. showed that glucose more than 200 mg on first post-
operative day increased the risk of serious infections by 6-fold3. Capes, et al.
meta-analysis identified admission glucose more than 110 mg is independently
responsible for increased mortality in acute strokes4. Diabetes team consultation
can reduce hospital stay by 56% with reduction in cost5.

Case report
Sixty-two-year-old man admitted with pneumonia, was found to have a
blood sugar of 391 mg%. He was diagnosed to have DM 2 years back, control
was reported to be acceptable on tablets when seen last year by his family
physician 2 months prior to admission. On examination his vitals were stable and
systemic examination revealed crepitations in his right lower chest. Investigations
revealed an elevated white cell count, his urine for ketone bodies was positive
and serum bicarbonate was normal.

Causes of high blood sugar at hospital


 Known DM with poor control of blood glucose or worsened due to
underlying illness.
 Previously unknown DM, diagnosed for the first time at admission.
 Use of glucocorticoids in patients with DM or prediabetes.

60
Management of Type 2 Diabetes

Effect of hyperglycemia on organ systems


 Metabolic state
— Dehydration
— Ketosis
— Natriuresis.
 Immune function
— Phagocyte dysfunction
— Immunosuppression.
 Cardiovascular system
— Induction of cardiac myocyte death through apoptosis
— Exaggeration of reperfusion injury
— Platelet abnormalities
— Electro physiologic changes
— Induces thrombotic state.
 Cerebrovascular system
— Enhanced neuronal damage following induced brain ischemia.
 Vascular changes
— Inflammatory changes through mediators such as interleukin (IL)-6,
TNF-a and IL-18 which are produced in response to hyperglycemia.

Learning Point - 1
 A good glycemic control reduces mortality, morbidity and duration of
hospital stay.

Why diabetes control destabilizes in hospital?


It is a common observation that patients admitted to hospital otherwise
known to be in good control, have blood sugars in unacceptably high range.
 Causes for destabilization of blood glucose
— Stress response
— Change in diet and lack of physical activity
— Infection
— Concomitant medications
— Transfer of therapeutic responsibility
— Priority of care
— Altered mental status.

Learning Point - 2
 Infections and drugs antagonizing insulin action are common causes of poor
control of DM in hospital.

61
Diabetes in Hospital

Assessment
Assessment of a person with DM in hospital can be divided in to four
headings:
 Assessment of glycemic state
 Assessment of metabolic state
 Chronic diabetic complications
 Specific reason for poor glycemic control.

Assessment of glycemic state


 Achieving good glycemic control
Implementing intensive diabetic therapy in the hospital setting requires
frequent and accurate blood glucose checking. The following frequency of
glucometer checking we would recommend:

Frequency of Glucometer Checking


Category of patients Dosage
Who are eating Premeal and at bed time (four/day)
Not eating and on glucose Insulin - potassium drip testing every 4 hours for
correcting insulin doses
Controlled with intravenous Hourly testing until the blood glucose levels are
insulin infusion stable. Thereafter, every second to fourth hourly

 Glycemic target
— Subcutaneous (s.c.) insulin
– Premeal 80-140
– Bedtime 120-180.
— Intravenous (i.v.) insulin 80-140.
 Glucose control
— Good: More than 80%,
— Suboptimal: 40-80%,
— Poor: <40% of the blood glucose recording in the target range.

Assessment of metabolic state


Assessment of metabolic parameters like potassium, level of hydration,
sodium and acid-base parameter are important in the management of a patient
with diabetes.

62
Management of Type 2 Diabetes

 Dehydration: Hyperglycemia causes osmotic diuresis, which leads to


dehydration. The best ways to look for it are axillary moistness and postural
drop in systolic blood pressure.
 Sodium: Hyponatremia in presence of hyperglycemia could be true
hyponatremia secondary to dehydration. It could also be pseudohyponatremia
as a result of osmotic flux of water from intracellular to extracellular
space in presence of hyperglycemia. Therefore to assess the severity of
sodium and water deficit, serum sodium may be corrected by adding
1.6 mEq/dl to the measured serum sodium for each 100 mg/dl of glucose
above 140 mg/dl.6
 Potassium: One of the common causes for hypokalemia related to management
of hyperglycemia is when the patient is on dextrose insulin infusion as insulin
causes intracellular shift of potassium. On the other hand hyperkalemia is
to be anticipated in presence of renal impairment, increased catabolic states
and acidosis.
 Bicarbonate: To assess acid base balance, venous bicarbonate measurement
will be more than sufficient. Arterial blood gas is more cumbersome.
 Hyperglycemic emergency: Diabetic ketoacidosis and hyperosmolar non-
ketotic state are the clinical scenarios, which will need thorough assessment
both clinically and biochemically. (Please refer separate chapter in this
book.)

Chronic diabetic complications

Microvascular Macrovascular

Retinopathy Coronary artery disease

Neuropathy Peripheral vascular disease

Nephropathy Cerebrovascular disease

Renal function can be assessed with serum creatinine and urine routine
examination for protein. In presence of urinary infections, protein estimation
is unreliable. Glomerular filtration rate can be estimated using Cockcroft and
Gault equation.
A retinal examination should be included whenever possible.

Creatinine (140 - age) × Body weight in kg × (1.22 males or 1.04 females)


clearance =
Serum creatinine in mg × 72

63
Diabetes in Hospital

Specific reason for poor glycemic control


Infection itself be a reason for admission. It is important to screen for a septic
foci in case of suspicion. Certain medicines can also worsen the glycemic sontrol.

Drugs Mechanism
Glucocorticoids Impair hepatic and peripheral insulin sensitivity
OC pills Impair insulin sensitivity
Thiazide diuretics Impair insulin action and release
Non-selective b-blockers Induce insulin resistance
Streptozotocin b-cell damage
Pentamidine b-cell damage
Cyclosporin b-cell damage
Diazoxide Inhibits insulin secretion
Indinavir, saquinavir, ritonavir, Cause insulin resistance
nelfinavir, clozapine, olanzepine

Learning Point - 3
 Serum sodium can be falsely low when blood sugar is very high
(pseudohyponatremia).

Principles of management
Considering the numerous contraindications to the use of oral agents in the
hospital, insulin is the clear choice for glucose manipulation in the hospitalized
patients, because of its quicker action, better predictability and convenience of
adjustment7.

Types of Insulin
Type Onset (min) Peak (min) Duration (hrs)
Rapid acting
 Lispro 5-15 30-60 3-4
 Aspart 5-15 30-60 3-4
Short acting
 Regular 30-60 120-180 3-6
Intermediate acting
 NPH 120-240 240-720 10-16
Long acting
 Glargine 180-240 480-960 24
 Detemir 180-240 480-960 20

64
Management of Type 2 Diabetes

Premixed
 75/25
 30/70 30-120 Dual 10-16
 50/50

Use of insulin
The in-patient insulin regimen must be matched or tailored to the
clinical circumstances and individual patient. Insulin dose requirement may
be thought of as consisting “basal” and “nutritional insulin requirement.
During hospitalization in addition to the basal and nutritional, requirement
may increase due to stress, infection and various medications. This will be
the correction dose.

Insulin requirement in health & disease - Conceptual

70

60

50

40
Insulin dose
30

20

10

0
Healthy Sick/eating Sick/NPO

Basal Prandial Correction

Basal and prandial insulin for patients who are eating


Subcutaneous injection of intermediate acting insulin (including
premixed insulin) or long acting insulin analogs (e.g., Glargine) can be used.
Bolus or mealtime insulin (insulin lispro and aspart) is usually administered
before eating.

65
Diabetes in Hospital

Basal Insulin

Basal Insulin

66
Management of Type 2 Diabetes

Learning Point - 4
 Most important patient factor for using s.c. insulin in hospital is the oral intake
status - patient eating reasonably or not.

Insulin regimens and special situations


The most important determinant of selecting insulin regimen is, patient’s
eating status.

Insulin regimens for NPO patients


 Basal insulin for patients who are not eating
While not eating (Nil Per Oral - NPO) patients require basal insulin.
Withholding basal insulin in patients may result in rapid rise in blood
glucose by 45 mg/dl/hr8.
 Intravenous insulin infusion
The best way to give continuous basal insulin is by i.v. infusion. It is used
in acute metabolic complications like diabetic ketoacidosis, preoperative,

67
Diabetes in Hospital

intraoperative and postoperative care, cardiogenic shock, stroke, during high


dose steroid therapy, critical care illness, etc.
 Sliding scale i.v. insulin infusion9
Preparation: 50 units of rapid acting (neutral) insulin is added to 49 ml
of normal saline (1 unit/1 ml) and to be given by an infusion pump. An
example of a scale could be

Blood glucose (mg/dl) Insulin dose (ml/hr)


<80 Nil
81-110 1
111-140 2
141-180 4
181-220 7
221-260 10
>260 Review the scale

 Glucose insulin potassium infusion


Glucose insulin potassium infusion is better used if facility for i.v.
insulin infusion is not available. This can be divided:
— Scheduled regimen
– A standard solution can be prepared in 500 ml of 5% dextrose
normal saline solution with 10 mEq of potassium and 10 units of
short acting insulin
— Supplemental regimen
– Subcutaneous short acting insulin can be given 4 hourly as
supplemental regimen to keep blood glucose in the target range.
An example of a scale could be

Sugar value (mg/dl) Dose of short acting Insulin


<100 No Insulin
100-140 4
141-180 6
180-220 8
221-260 10 and inform

When the requirement of the supplemental insulin goes higher, the insulin
added in the scheduled regimen should be increased.

Transition from i.v. to s.c. therapy10


The transition from i.v. to s.c. insulin can be made once the patient is

68
Management of Type 2 Diabetes

eating, reasonably well and it should ideally be planned so that first s.c. dose
is given before breakfast and excursion of sugars can be observed in the day
time and dose adjusted.

Glycemic management in surgical patients – a quick overview


Whenever possible, surgery should be scheduled after glucose, fluid and
electrolyte status has been optimised.
l Preoperative evaluation:
— Blood glucose control
— Cardiac disease - beware of silent ischemia!
— Renal function.
l On the day before surgery:
— For type 1 DM: Normal insulin during day
— For type 2 DM: Should take usual OHAs except metformin/long acting
sulfonylureas
— NPO after midnight.
l On the operative day:
— Surgery to be scheduled as early in the day as possible
— i.v. glucose started with potassium (rate as per patient’s requirement)
roughly @100 ml/hr.
l Postoperative period:
— Glucometer glucose every 1-2 hours
— Glucose reading to keep in the target of 80-140 mg/dl
— This regime to be continued till patient is eating and drinking, and then
i.v. insulin infusion should be stopped and s.c. insulin to be started.

Enteral feeding and insulin therapy


There is no contraindication for use of any enteral nutritional formulas in
people with diabetes as long as we know the carbohydrate exchange of the
feed. It is important to know the frequency of feeding.
Two common ways of feeding:
l Two hourly interval
l Continuous by volumetric infusion pump.
If feed is given every 2 hourly, it is mainly given from early morning till
approximately 10 p.m. Basal insulin like Glargine or two times intermediate
acting insulin can be given for the patients on continuous infusion pump and
the patients on 2 hourly feed can be put on multiple s.c. injection either in the
basal bolus or modified basal bolus regimen, which ever is appropriate.

69
Diabetes in Hospital

Steroid therapy and insulin


Glucocorticoids act on postinsulin receptor arena to induce dose-dependent
insensitivity of insulin to its target tissues. Due to the effect of glucocorticoids
on postprandial glucose, an emphasis on the prandial insulin would be expected
to have the best results.
Glucocorticoid induced hyperglycemia is best treated with insulin. Patients
with DM on oral drugs starting on high doses of steroids (e.g., prednisolone
30 mg/day or more) should be transferred to insulin at 0.5 U/kg body weight
per day initially, divided between morning and evening doses of short and
intermediate acting insulin. For patients already taking insulin, the dosage may
need to be increased by approximately 50% initially, starting on the same day as
steroid therapy. If methyl prednisolone is used, because of its longer duration of
action there is expected to be a lot of fluctuations in the sugar and practically
it is a very difficult situation as far as the glycemic control is concerned.

Learning Point - 5
 NPO patient needs continuous insulin either by insulin infusion or GIK while
basal bolus regimen is the best for hospitalized patients who are eating
reasonably well.

Discharge planning
Hospital admission should be used as an opportunity to teach patients
who lack diabetes knowledge or who require improvement in self-management
skills.

Diabetic medications
Persons with DM going home after surgery must understand insulin is the
most natural thing for his/her system and not tablets.
A recent hospital discharge is a strong predictor of subsequent serious out
patient hypoglycemia8. We recommend a dose reduction of insulin by 30-40%
(of the last day’s dose) at the time of discharge in view of anticipated increased
amount of physical exercise, relief from stress and improvement in infection, etc.
Prescribing patterns should take into consideration the evidence that among the
sufonylureas, glipizide and glimiperide are associated with less hypoglycemia
than gliclazide and glibenclamide, particularly in the elderly8.

70
Management of Type 2 Diabetes

New onset hyperglycemia


Patients detected to have hyperglycemia for the first time during hospitalization
can either be new diagnosis of DM or stress induced hyperglycemia. In such
a situation, a glucose tolerance test (GTT) 6 weeks after discharge is helpful to
distinguish.

Learning Point - 6
 Hospital admission can be utilised to teach patient how to cope with diabetes,
injection technique, glucometer, etc.

GTT should be done with a free carbohydrate diet for atleast 3 days prior to
the test. A normal GTT will retrospectively diagnose stress-induced hyperglycemia.
It is important to remind the patient that, not only hyperglycemia will come
back in stressful situation in the future but also chance of developing diabetes
subsequently is high. In such situations, we recommend GTT at regular interval
along with lifestyle improvement.

References
1. Deepak PJ, Sunitha K, Nagaraj J and Bhatacharya A. Inpatient management if diabetes:
Survey in a tertiary care centre. Postgrad. Med. 2003;79:585-587.
2. Malmberg K, for the DIGAMI study group. Prospective randomised study of intensive
insulin treatment on long-term survival after acute myocardial infarction in patients
with diabetes. Br. Med. J. 1997;314:1515-1521.
3. Pomposelli JJ, Baxter JK, Babineau TL, et al. Early postoperative glucose control predicts
nosocomial infection rate in diabetic patients. J. Parenter. Enter. Nutr. 1998;22:77-81.
4. Capes S, Hunt D, Malmberg K and Gerstein H. Stress hyperglycaemia and increased
risk of death after myocardial infarction in patients with and without diabetes:
A systematic overview. Lancet 2000;355:773-778.
5. Levatan CS, Sales JR, Wilets IF, et al. Impact of endocrine and diabetes team consultation
on hospital length of stay for patients with diabetes. Am. J. Med. 1995;99:22-28.
6. Teresa A, Robert D, Barrett MD, et al. Hyponatremia: Evaluating the correction factor
for hyperglycaemia. Am. J. Med. 1999;106:399-403.
7. Vanderberg G, Wonters P, Weckers F, et al. Intensive insulin therapy in critically ill
patients. N. Engl. J. Med. 2001;345:1359-1367.
8. Shorr RI, Ray WA, Daugherty JR, et al. Individual sulfonylureas and serious
hypoglycaemia in older people. J. Am. Geriat. Soc. 1996;44:751-755.
9. Quale WS, Seidler AJ and Brancati FL. Glycemic control and sliding scale use in
medical inpatients with diabetes mellitus. Arch. Intern. Med. 1997;157:547-552.
10. Bhattacharya A, Kaushal K, New JP, Dornan TL and Young R. Glucose control during
inpatient management of diabetes. Diabet. Med. 2001;18:322.

71
Hypoglycemia in Subramanian Kannan

Type 2 Diabetes Mellitus Shriraam Mahadevan

Introduction
The incidence of type 2 diabetes mellitus is rapidly increasing worldwide
with figures projecting that India would be harboring the largest population of
diabetics by 2025. Long-term benefits of strict glycemic control on micro and
macrovascular complications of diabetes have been established beyond doubt.
Recent guidelines recommend stricter glycemic control and advocate a goal of
HbA1C levels <6.5. Hypoglycemia is one of the major limiting factors precluding
the achievement of this goal. It causes recurrent symptomatic and sometimes,
atleast temporarily, disabling episodes in many with advanced type 2 diabetes,
and is sometimes fatal. Hence, understanding the importance of hypoglycemia
in the management of diabetes is of paramount importance. In fact, were it not
for the barrier of this complication all diabetics can achieve normal HbA1C over
a life-time of diabetes.

Definition of hypoglycemia
Hypoglycemia is generally defined as a plasma glucose level of <45-50 mg/
dl . However an important framework for making the diagnosis of hypoglycemia
1

is Whipple’s triad which includes: i) Symptoms consistent with hypoglycemia,


ii) low plasma glucose concentration and iii) relief of symptoms after plasma
glucose level is raised.

Prevalence of hypoglycemia in type 2 diabetes


As alluded to before, the benefits of tight glycemic control are well
documented and recent guidelines for glycemic control advocate stricter
goal of HbA1C <6.5%. These recommendations are based on results from two
landmark studies, the Diabetes Control and Complications Trial (DCCT) and
U.K. Prospective Diabetes Study (UKPDS), which demonstrated the benefits
of intensive glycemic control in type 1 and type 2 diabetes, respectively2,3.
The incidence of documented hypoglycemia rises whenever tight glycemic
control is attempted. Over 6 years of follow-up of patients with type 2

72
Management of Type 2 Diabetes

Physiology of Glucose Counter Regulation - “The Glucose Barometer”


Blood glucose
Mechanism levels (mg/dl)

1. Insulin secretion shuts off 70-75

2. Glucagon: (Rapid action) Increases hepatic glycogenolysis, 65-70


increases hepatic gluconeogenesis

3. Epinephrine, norepinephrine: (Rapid action) Inhibits 60-65


glucose utilization by muscle, increases hepatic
gluconeogenesis, stimulates glucagon secretion, inhibits
insulin secretion, stimulates hepatic glycogenolysis

4. Cortisol: (Delayed action) Increased hepatic 55-60


gluconeogenesis, inhibits glucose utilization by muscle

5. Growth hormone: (Delayed action) Inhibits glucose 55-60


utilization by muscle

diabetes in the UKPDS, 2.4% of those using metformin, 3.3% of those using a
sulfonylurea and 11.2% of those using insulin reported severe hypoglycemia4.
The feasibility trial of the Veterans Affairs Cooperative Study on Glycemic
Control and Complications in Type 2 Diabetes (VA CSDM) showed an increase
in mild and moderate, but not severe hypoglycemia in patients undergoing
intensive treatment5 whereas the Kumamoto study showed no increase in
mild or severe episodes in patients with type 2 diabetes undergoing intensive
treatment compared with those undergoing less intensive treatment6. However
these data might have underestimated the problem due to less frequent
testing of blood sugars by type 2 diabetics when compared with their type
1 counterparts, comorbidities like loss of memory, inability to communicate
symptoms, lack of knowledge regarding hypoglycemia and atypical
presentation of hypoglycemia (predominant neuropsychiatric manifestations
rather than autonomic symptoms).
Though with much overlap, the above sequence of events occur fairly
in that order to counter blood glucose lowering to dangerous levels.
Another important protective response which occurs at the brain level is the
upregulation of GLUT 1 (glucose transporter 1) which pumps glucose into
the brain even at very low levels of plasma glucose. Though this seems
protective it may actually prove counter productive especially in a setting
of failure of autonomic responses (hypoglycemia unawareness) as discussed
subsequently.

73
Hypoglycemia in Type 2 Diabetes Mellitus

Clinical symptoms of hypoglycemia


 Neurogenic (autonomic).
 Cholinergic: Sweating, hunger, paresthesia.
 Adrenergic: Palpitations, tremor, anxiety.
 Neuroglycopenic (brain glucose depletion): Headache, visual disturbances,
mental dullness, confusion, seizure, loss of consciousness, focal neurological
deficit.
Very often the symptoms of hypoglycemia in a given patient are fairly
stereotypic (i.e.) constant for every episode in that person. b-blocker therapy may
mask some of the warning adrenergic symptoms but the cholinergic symptoms
especially sweating may be accentuated. Occasionally, symptoms suggestive of
hypoglycemia occur at normal blood glucose levels. This may be due to higher
glycemic thresholds at which chronically uncontrolled diabetics may have
symptoms. However, it may also be important to consider some mimickers of
hypoglycemia which include “dumping” syndrome (especially in patients with
autonomic neuropathy), anxiety disorder, myocardial infarction, seizures, etc.

Counter-regulatory responses in
type 2 diabetic patients
Glucose counter regulatory mechanisms have generally been found to be
intact early in the course of type 2 diabetes7, explaining the low frequency of
hypoglycemia. However hypoglycemia becomes progressively frequent as patient
approaches the insulin-deficient end of the spectrum of type 2 diabetes. Thus, it
would be expected that such patients would exhibit glucose counter regulatory
defects similar to those in type 1 diabetes. Segel, et al. reported that the glucagon
response to falling plasma glucose was virtually absent in advanced insulin-
treated type 2 diabetes. Glycemic thresholds for autonomic and symptomatic
responses to hypoglycemia were also shifted to lower glucose concentrations
by recent antecedent hypoglycemia8.

Clinical syndromes due to compromised glucose


counter regulation in diabetes
 Hypoglycemia unawareness.
 Hypoglycemia-associated autonomic failure (HAAF).
Recurrent episodes of hypoglycemia in a diabetic may blunt the response
of the couter regulatory autonomic system to the falling blood glucose levels.
This is known as HAAF. Consequent to the absence of warning symptoms of

74
Management of Type 2 Diabetes

autonomic overactivity the person might not be aware of the hypoglycemia


even at dangerously low levels of glucose and develops neuroglycopenic
symptoms directly (see flow chart). This clinical syndrome is called hypoglycemia
unawareness. This pathophysiology is further accentuated if the diabetic has
autonomic neuropathy or is on b-blocker therapy. Fortunately, awareness to
hypoglycemia is usually regained within few days of relaxation of glycemic
control which is the main stay of management.

Flowchart on Hypoglycemic Unawareness and Autonomic Failure


Impaired glucagon release/meal-OHA/
insulin mismatch

Hypoglycemia

Reduced sympathoadrenal
response to hypoglycemia

Decreased Decreased (adrenal)


sympathetic nervous epinephrine response
system response

Hypoglycemic Defective glucose


unawareness counter regulation

Recurrent hypoglycemia

Clinical case vignette


A 68-year-old Mr. V was diagnosed to have diabetes recently (FBS
178 mg/dl and Post 75 g BS 250 mg/dl). He was started on T. Daonil 5 mg b.d.
(glibenclamide/gliburide) by his primary physician. One week later he attended
a temple proceeding and skipped his lunch. By 4 p.m. he felt giddy and quickly
lost consciousness. He was taken to a local hospital, where his capillary blood
glucose was 35 mg/dl. He was given 25% dextrose as a bolus dose through
intravenous route. He regained consciousness immediately. He was discharged

75
Hypoglycemia in Type 2 Diabetes Mellitus

on request. Later that day by 11 p.m., he had one episode of convulsion and lost
consciousness. He was re-admitted with hypoglycemia again. This time however
apart from stopping the antidiabetic drug, he was maintained on continuous
glucose infusions and started on oral feeds. By the 5th day he was discharged
after through evaluation of body systems. He was started on T. metformin
250 mg b.d. along with lifestyle modifications.

Clinical Risk Factors for Hypoglycemia


Risk factors Mechanisms
Use of insulin/insulin secretagogue: Doses are Meal-insulin mismatch
excessive, ill-timed, or of the wrong type
Missed meals or snacks, during the overnight Exogenous glucose delivery is
fast, gastroparesis decreased
Alcohol ingestion Endogenous glucose production is
decreased
Exercise, after weight loss, increased fitness, or Sensitivity to insulin is increased,
improved glycemic control, or during treatment glucose utilization is increased
with an insulin sensitizer
Progressive renal failure Insulin clearance is decreased

In addition to the above other factors include concomitant drugs (quinine,


quinolones like gatifloxacin, sulphonamides, salicylates, pentamidine), associated
adrenal insufficiency, liver failure or sepsis and very rarely a co-existent
insulinoma.

Oral antidiabetic drugs induced hypoglycemia


The sulfonylureas account for a substantial proportion of cases of drug-
induced hypoglycemia. Most reported cases of severe hypoglycemia are seen in
patients taking the long-acting preparations like chlorpropamide or glibenclamide9.
However, severe episodes characterized by coma have been reported with all
the agents in common use. In part, the hypoglycemic potential of an agent is
related to its potency, its plasma and biological half-lives, its metabolism, and
the concomitant use of other drugs. For example, liver disease prolongs the
hypoglycemic actions of glibenclamide and glipizide, kidney disease may prolong
the action of insulin (due to impaired clearance) and sulfonylurea drugs can
interact with other agents to cause severe hypoglycemia. The additive (or possibly
synergistic) effects during combined insulin and sulfonylurea therapy account for
an increasing number of such episodes. Drugs that interfere with sulfonylurea
metabolism (e.g., fluconazole) or compete for circulating plasma protein binding

76
Management of Type 2 Diabetes

with sulfonylureas (e.g., sulphonamides, coumarin, clofibrate) can also potentiate


these effects. Metformin is predominantly antihyperglycemic and usually does
not cause hypoglycemia unless used with insulin or insulin secretogogues where
it may improve insulin sensitivity. In the UKPDS, the frequency of severe
hypoglycemia was lower with metformin than with sulfonylureas or insulin3
Non-sulfonylurea insulin secretagogues like repaglinide and nateglinide also can
cause hypoglycemia10. Thiazolidinediones such as pioglitazone and rosiglitazone
sensitize peripheral tissues to insulin and hence may cause hypoglycemia
when insulin is used concomitantly, though hypoglycemia can also occur
with monotherapy or when these drugs are used in combination with other
oral agents, particularly sulfonylureas10. a-glucosidase inhibitors (acarbose and
miglitol) are not associated with hypoglycemia, though in theory their use in a
patient with hypoglycemia (e.g., due to concomitant use of a sulfonylurea) may
prevent ingested carbohydrate from being metabolized to glucose in the gut. It
is therefore recommended that patients taking a-glucosidase inhibitors use oral
glucose instead of food to treat episodes of hypoglycemia.

Factors resulting in prolonged hypoglycemia with


sulphonyureas
Sulphonylureas bind to receptors on islet β-cells leading to insulin release.
Intravenous glucose will rapidly correct hypoglycemia but then acts as a
potent secretagogue to the sulphonylurea-sensitized β-cells. Insulin secretion is
stimulated and hypoglycemia recurs. Hence it is unwise to discharge patients
with sulphonylurea induced hypoglycemia after a satisfactory response to a
glucose bolus; intravenous glucose may be required for several days.

Factors increasing the risk of suphonylurea induced


hypoglycemia
 Advanced age.
 Long-acting agents such as glibenclamide and chlorpropamide. (In the
elderly even shorter acting agents such as gliclazide can cause hypoglycemia,
especially if renal or hepatic dysfunction is present.)
 Starvation/caloric restriction/sustained physical exercise.
 Acute systemic illnesses.
 Alcohol.
 Renal, hepatic and cardiovascular disease.
 Polypharmacy (drugs altering the kinetic of sulfonylurea or insulin).
 Prescription errors in non-diabetic subjects, consumption of spouse’s medication
and deliberate overdose.

77
Hypoglycemia in Type 2 Diabetes Mellitus

Diagnosis of hypoglycemia: “How low is low?”


The symptoms and signs of hypoglycemia are non-specific. Therefore,
documentation of a low plasma or blood glucose concentration, if possible, is very
helpful. While documentation of a low plasma or blood glucose concentration
is preferable, if that is not practical it is better for the patient to self-treat when
he or she suspects hypoglycemia, since the short-term risks of failure to treat
an episode far outweigh those of unnecessary treatment. While plasma glucose
concentrations can be unequivocally low, it is not possible to define hypoglycemia
on the basis of a specific plasma glucose concentration in people with diabetes.
The glycemic thresholds for diabetics are dynamic rather than static. People
with poorly controlled diabetes can suffer symptoms of hypoglycemia at plasma
glucose concentrations higher than those required to elicit symptoms in non-
diabetic individuals, while those with tightly controlled diabetes often tolerate
low glucose levels without symptoms11. In practice, blood glucose levels for
hypoglycemia need to be individualized for a given patient at a given point in
time. Because lower levels impair defenses against subsequent hypoglycemia, a
reasonable goal is a lower limit of 72 mg/dl (4.0 mmol/l)11.

Common errors in diagnosis


 Administration of dextrose before venous sample is drawn for lab study.
 Measurement of capillary blood glucose in patients with fluid overload,
peripheral circulatory failure.
 Blood sugars may be within normal limits in suphonylurea induced
hypoglycemia, particularly with long-acting drugs. A diagnosis of
hypoglycemia should not be ruled out based on a single value.
 Failure to re-check blood sugars in patients with recurrent hypoglycemia
due to sulfonylureas.

Common causes of hypoglycemia in India


 Meal-medication mismatch.
 Common usage of long-acting suphonylureas (glibenclamide).
 Over the counter purchase of antidiabetic drugs.
 Renal failure.
 Lack of knowledge of symptoms of hypoglycemia.
 Month of religious fasting (like Ramadan).
 Alcohol consumption.

78
Management of Type 2 Diabetes

Management
Asymptomatic/mild hypoglycemia (minor episode): Episodes of
asymptomatic hypoglycemia (detected by self-monitoring of blood glucose) and
most episodes of symptomatic hypoglycemia can be effectively self-treated by
taking carbohydrate in the form of plain sugar/glucose, sugar toffee, fruit juice,
a soft drink, milk, biscuits or a meal. Ingestion of a snack or meal shortly after
the plasma glucose concentration is raised is advisable as it will sustain the
blood glucose levels.
Severe hypoglycemia (major episode): Intravenous glucose is the preferable
treatment of severe hypoglycemia, given as 100 ml of 25% dextrose bolus. It is
followed by infusion of D5W to maintain the blood glucose above 100 mg/dl.
It is advisable to avoid repeated boluses of concentrated dextrose solutions as
it will cause reflex surge in insulin secretion and precipitate another episode
of hypoglycemia. Because severe hypoglycemia, particularly that caused by
a sulfonylurea, is often prolonged, subsequent glucose infusion and frequent
feedings are often required. It is important to establish the absence of recurrent
hypoglycemia unequivocally before such a patient is discharged. Glucagon is
less useful in type 2 diabetes because it stimulates insulin secretion as well as
glycogenolysis.

Pharmacological agents for treatment of sulphonylurea


induced hypoglycemia
Suppression of insulin secretion is a logical adjunct to intravenous glucose
therapy in sulphonylurea-induced hypoglycemia. Diazoxide inhibits insulin release,
but hypotension and reflex tachycardia may preclude its use in elderly patients
with coronary heart disease. The long-acting somatostatin analog octreotide has
a potent inhibitory effect on insulin. It should be administered subcutaneously in
an initial dose of 50 μg three times a day and may be required for several days,
especially for long-acting or sustained release sulphonylurea preparations.

Management of hypoglycemic unawareness


In patients with clinical hypoglycemia unawareness, a 2- to 3-week period
of scrupulous avoidance of hypoglycemia is advisable and can be assessed by
return of awareness of hypoglycemia. In practice it can involve acceptance of
somewhat higher glucose levels in the short term. However with the return
of symptoms of developing hypoglycemia, better glycemic control should be
attempted.

79
Hypoglycemia in Type 2 Diabetes Mellitus

Common errors in management


 Failure to administer continuous infusions of dextrose after initial bolus.
 Use of glucagon in type 2 diabetics, alcoholics with hypoglycemia.
 Use of food containing complex sugars as replacement in patients on
acarbose.

Reducing the risk of hypoglycemia


 Patient education in the main pillar of risk reduction. The fear of hypoglycemia
should be allayed and patient should be motivated for better glycemic
goals.
 Avoiding long-acting drugs and polypharmacy, particularly in elderly.
 A regular check for renal, hepatic, cardiac dysfunction.
 Strict abstinence of alcohol.
 To recognize atypical (neuropsychiatric) manifestations of hypoglycemia and
adjust the dose of medications.

Conclusions
The threat and incidence of hypoglycemia is a major limiting factor in
intensive glycemic management of diabetes. Nonetheless, it is possible to both
improve glycemic control and minimize hypoglycemic risks by understanding
the physiological counter regulatory responses and aggressively monitoring
glycemic therapy. Every effort needs to be made to minimize the frequency
and magnitude of hypoglycemia. Pending the prevention and cure of diabetes
or the development of treatment methods that provide glucose-regulated insulin
replacement or secretion, we need to learn to replace insulin/antidiabetic drugs
in a much more physiological fashion; to prevent, correct, or compensate for
compromised glucose counter regulation so that we are able to achieve near-
euglycemia safely in people with diabetes.

Points to remember
 Hypoglycemia in diabetics is commonly due to meal-medicine mismatch.
 Patient education is the key in prevention.
 Episodes of hypoglycemia should not discourage the physician and the
patient from a better and stricter glycemic control.

References
1. Harrison’s Principles of Internal Medicine 16th Edition.
2. The DCCT Research Group: The effect of intensive treatment of diabetes on the
development and progression of long term complication in insulin-dependent diabetes
mellitus. N. Engl. J. Med. 1993;329:977-986.

80
Management of Type 2 Diabetes

3. The U.K. Prospective Diabetes Study Group: Intensive blood-glucose control with
sulfonylureas or insulin compared with conventional treatment and risk of complication
in patients with type 2 diabetes. Lancet 1998;352:837-853.
4. The United Kingdom Prospective Diabetes Study Group: U.K. prospective diabetes
study. 16. Overview of 6 years’ therapy of type II diabetes: A progressive disease.
Diabetes 1995;44:1249-1258.
5. Abraira C, Colwell JA, Nuttall FQ, Sawin CT, Nagel NJ, Comstock JP, Emanuele NV,
Levin SR, Henderson W and Lee HS. Veterans Affairs Cooperative Study on glycemic
control and complications in type II diabetes (VA CSDM): Results of the feasibility
trial. Diabetes Care 1995;18:1113-1123.
6. Ohkubo Y, Kishikawa H, Araki E, et al. Intensive insulin therapy prevents the
progression of diabetic microvascular complications in Japanese patients with non–
insulin-dependent diabetes mellitus: A randomized prospective 6-year study. Diabetes
Res. Clin. Pract. 1995;28:103-117.
7. Cryer PE. Hypoglycaemia: The limiting factor in the glycaemic management of type
I and type II diabetes. Diabetologia 2002;45:937-948.
8. Segel SA, Paramore DS and Cryer PE. Hypoglycemia-associated autonomic failure in
advanced type 2 diabetes. Diabetes 2002;51:724-732.
9. Burge MR, Schmitz-Fiorentino K, Fischette C, Qualls CR and Schade DS. A prospective
trial of risk factors for sulfonylurea-induced hypoglycemia in type 2 diabetes mellitus.
JAMA 1998;279:137-143.
10. Goldberg RB, Einhorn D, Lucas CP, et al. A randomized placebocontrolled trial of
repaglinide in the treatment of type 2 diabetes. Diabetes Care 1998;21:1897-1903.
11. Cryer PE, Davis SN and Shamoon H. Hypoglycemia in diabetes. Diabetes Care
2003;26(6):1902-1912.

81
Hyperglycemic Emergencies:
Diagnosis and Management
Nilanjan Sengupta

The two common acute hyperglycemic emergencies of diabetes mellitus are


diabetic ketoacidosis (DKA) and hyperglycemic hyperosmolar non-ketotic state
(HHS). They are largely preventable conditions but left untreated can be causes
for morbidity and mortality. Though classically DKA is a common complication
of uncontrolled type 1 diabetes and HHS that of uncontrolled type 2 diabetes
both can occur in both types of patients; DKA can and does occur in patients
of type 2 diabetes, especially under stressful conditions.

Case Report
A 9-year-old boy, apparently healthy previously was admitted to a local
nursing home under care of a pediatrician with cough, expectoration, high fever
obtundation and features of dehydration for the preceding 5 days. Work-up
revealed that he was suffering from community acquired pneumonia which
responded well to empirical parenteral antibiotics.
In the course of investigations his fasting plasma glucose came out to be
410 mg/dl which was persistently high on repeated cross-checking. There was
significant ketonuria and metabolic acidosis. Thus a diagnosis of DKA was
made and he was started on insulin infusion and fluid replacement. Within 48
hours he improved and was put on a multiple subcutaneous insulin regimen
with insulin lispro and protaminated lispro with which he was maintaining
reasonable glycemic control. He was discharged home on the 10th day after
the chest infection was fully under control and the patient, his parents and
class teacher were trained adequately regarding management of type 1 diabetes
including recognition and management of hypoglycemia.
This case demonstrates how type 1 diabetes, hitherto undiagnosed, can
present for the first time with DKA and in this boy chest infection precipitated
the metabolic emergency.

82
Management of Type 2 Diabetes

Diabetic ketoacidosis
Definition: The three cardinal features of DKA are hyperglycemia, acidemia
and ketonemia. Hyperketonemia is not routinely quantified in clinical practice,
and the working diagnostic criteria may be plasma bicarbonate ≤15 mmol/l
with significant (atleast ++ in dipstick test) ketonuria.
Precipitating factors: The commonest identifiable precipitating factor is
infection like urosepsis or pneumonia. Deliberate omission or under dosage of
insulin, especially in patients of low literacy and socio-economic status constitutes
an important precipitating cause of DKA, may be upto 30% cases. Malfunction of
insulin pumps in those who are using the same may be another cause. However,
in upto 40% cases no precipitating cause may be identified1. Finally, as in the
case described above, 20% of presentation may be in a hitherto undiagnosed case
of diabetes, DKA being the initial presenting manifestation in such patients.
Pathogenesis: DKA is the result of absolute deficiency or ineffective
concentration of circulating insulin coupled with counter-regulatory hormone
(glucagon, catecholamines, cortisol and growth hormone) excess. Insulin deficiency
results in unbridled hepatic glucose production resulting in hyperglycemia. This
severe hyperglycemia, in turn, results in osmotic diuresis, dehydration and
electrolyte loss.
Insulin deficiency and counter-regulatory hormone excess lead to enhanced
adipose tissue lipolysis; the excess free fatty acids produced therefrom lead
ultimately to formation of acetyl coA. In DKA, acetyl coA cannot be completely
oxidized in the citric acid cycle but instead is converted in the liver mitochondria
to acetoacetate and 3 hydroxybutyrate, the so-called ketone bodies.
Symptomatology: Diagnosis of DKA is not difficult in typical clinical
situation e.g., in patients of type 1 diabetes having severe hyperglycemia and
intercurrent illness. However, in many patients of type 2 diabetes the diagnosis
is often erroneously not considered based on a misconception that DKA cannot
complicate type 2 diabetes because these patients are not that insulinopenic
compared to the ketosis prone type 1 patients. The worst scenario is when the
subject is not known to have diabetes where the diagnosis may be completely
missed before the person dies: Strange as this may appear, this is not uncommon
in developing countries like India especially in rural areas where awareness
about diabetes and healthcare facilities are inadequate.
The osmotic diuresis engendered by hyperglycemia leads to polyuria,
polydipsia, rapid weight loss and generalized weakness. Patients may complain
of abdominal pain (may masquerade as surgical emergency) and vomiting,
the latter often precipitating an emergency hospital admission. Alteration of

83
Hyperglycemic Emergencies: Diagnosis and Management

sensorium results predominantly from dyseletrolytemia and hyperosmolality;


frank coma occurs in only about 10% of patients and it is not uncommon for
a person with DKA to walk into the doctor’s consulting chamber! Metabolic
acidosis stimulates the medullary respiratory centers resulting in the classical
deep breathing originally described by Adolf Kussmaul. Peripheral vasodilatation
due to metabolic acidosis may lead to hypotension and hypothermia. Severe
dehydration may cause prerenal azotemia.
Work-up: DKA is a medical emergency and correct diagnosis and rapid
institution of therapy are key to success of management. A quick, but through
history should be elicited in an attempt to find out any obvious precipitating
cause and compliance to therapy.
Clinical examination should aim at assessment of severity of fluid loss, degree
of impairment of sensorium, severity of acidosis, and presence of precipitating
illnesses or associated complications. Bedside dry chemistry helps to document
hyperglycemia and ketonuria and to follow up the patient. However, laboratory
tests for plasma glucose, electrolytes, urea, creatinine, complete blood count and
arterial blood gas analysis (if available) should be routinely ordered. Other tests
may be necessary as per suspected clinical conditions, e.g. urine culture, and
chest X-ray.
The anion gap is calculated thus: [sodium] - ([chloride] + [bicarbonate]).
The normal anion gap is 7-9 mmol/l; in DKA and other causes of increased
anion gap acidosis such as CRF, it exceeds 15 mmol/l.
Diagnostic pitfalls: There are a few diagnostic pitfalls in DKA which are
worth remembering. The nail-varnish remover like smell of acetone in breath
is a useful clinical sign but many people are anosmic for acetone. Because
of acidosis mediated vasodilatation, fever may be absent despite presence
of infection. Leucocytosis in DKA represents a stress response and may be
present in absence of infection. There may be factitious hyponatremia because
of hypertriglyceridemia and hyperglycemia. Metabolic acidosis, by effecting
extracellular shift, may temporarily elevate plasma potassium level despite
severe total body potassium deficit. Assay interference by ketone bodies may
falsely elevate serum creatinine level. Liver enzymes, pancreatic amylase and
creatine phosphokinase may be non-specifically elevated. Dipstick testing may
be negative for ketones if lactic acidosis or alcoholic ketoacidosis co-exist (ketosis
sans ketonuria). Again, dipstick reaction may become temporarily stronger as
the condition improves since 3 hydroxybutyrate gets converted to acetoacetate
in the pathway of oxidation.

84
Management of Type 2 Diabetes

Treatment: Treatment of DKA consists of:


l Rehydration and electrolyte repletion
l Insulin therapy
l Treatment of precipitating cause and comorbidities
l Other supportive measures.
Fluid repletion is to commence with isotonic saline; usually 6-10 liter of fluid
need be infused in the first 24 hours. When plasma glucose has come down to
250-300 mg/dl, 5% dextrose is to be used to prevent osmotic disequilibrium. This
is to be continued till the patient can be shifted to an appropriate subcutaneous
insulin regimen.
Depending on the level of serum potassium, potassium chloride
supplementation has to be done from the second liter of normal saline
onwards.
Generally, bicarbonate should be avoided since it has many disadvantages: It
may cause paradoxical CSF acidosis, may have adverse effects on the oxyhemoglobin
dissociation curve aggravating tissue hypoxia, can cause overshoot alkalosis, and
may, in fact, accelerate ketogenesis and lactate production: Moreover 8.4% sodium
bicarbonate is a tissue irritant and extravasation may lead to tissue necrosis2.
However, in the severely acidotic patient bicarbonate may be used as an isotonic
solution, taking care not to aim at complete correction of acidosis.
Insulin is best administered as a continuous infusion @ 5-10 u/hour through
syringe driven pump guided by hourly capillary blood glucose values. This
infusion is to be continued till the patient is taking food orally and the ambient
blood glucose level is around 100-200 mg/dl when it can be abandoned for an
appropriate subcutaneous regimen.

A Progress Log or Flow Chart as Shown, helps in following the Patients well in a
busy ITU and Minimizes Confusion
Date Time Pulse BP Sensorium CBG Ketones pH Anion Intake Output Insulin
gap

85
Hyperglycemic Emergencies: Diagnosis and Management

In absence of facilities for insulin infusion, an intramuscular regimen may


be employed. This consists of an initial intramuscular bolus of 20u of short
acting insulin, followed by 5-10u hourly till the patient is fit for conversion into
a subcutaneous regimen.
Monitoring: Ideally, patients of DKA should be monitored in an intensive
therapeutic unit setting, with clinical follow-up, hourly finger-stick blood glucose,
and periodic urinary ketone testing; pH and calculated anion gap assessments
help in following progress.

DKA monitoring flow chart


Complications: The complications of DKA, which may be responsible for
considerable morbidity and mortality, are cerebral edema (may be due to osmotic
fluid shift, common in children, has ~90% mortality), adult respiratory distress
syndrome, thromboembolism and rhinocerebral mucormycosis.
Prevention: Much has been discussed in literature regarding treatment
of DKA, rightly so, because this is a life-threatening medical emergency. But
one must appreciate that metabolic emergencies of diabetes like DKA or HHS
are better prevented than treated. If a person with diabetes, irrespective of
the type and the treatment he is on diligently follows the treatment regimen
prescribed, is sincere in periodic clinical and biochemical follow up and takes
necessary remedial measures in face of rising plasma glucose there is no reason
why he should suffer from DKA or HHS. In a country like India where still
illiteracy and poverty are rampant many a time patients of type 1 diabetes stop
insulin in search of “easier” traditional remedies and end-up having DKA; every
medical practitioner, right from the primary care physician to the superspecialist
endocrinologist should impress upon the absolute necessity of insulin in
type 1 diabetes to the patient himself and his relatives. Patients who are
on insulin should be explained sick day rules and encouraged not to stop
insulin altogether in fear of getting hypoglycemia, if oral food intake is
impaired due to intercurrent illness; an early visit to the treating physician
in such states prevents many a medical catastrophe.

DKA Management: Key Points


l Fluid and electrolyte repletion.
l Low dose insulin therapy, preferably by intravenous infusion @5-10 u/hour.
l Bicarbonate not necessary in most patients and is best avoided.
l 5% dextrose to start when ambient blood glucose reaches 250 mg/dl.
l Blood pH and calculated anion gap are the best biochemical markers for follow-
up; persistence of ketonuria alone does not indicate clinical deterioration.

86
Management of Type 2 Diabetes

Hyperglycemic hyperosmolar non-ketotic state


HHS occurs typically in middle aged or elderly individuals with diabetes,
commonly type 2 diabetes. The usual precipitating factors are infection, stroke
or myocardial infarction; certain drugs like diuretics, b-blockers and recently, a
few atypical antipsychotics have also been implicated3.
Patients may present with severe hyperglycemia (blood glucose often
above 600 mg/dl), gross dehydration and hyperosmolality, focal neurodeficit
and altered sensorium. The essential differences between DKA and HHS are
enumerated in the Table.

Table. Differences between DKA and HHS


Parameter DKA HHS
Patient profile Usually type 1 diabetes Usually type 2 diabetes
mellitus/children mellitus/elderly
Ketosis Present Absent/trace
Metabolic acidosis Present Absent
Vomiting Present Absent
Dehydration Mild-to-moderate Severe
Hyperglycemia Moderate-to-severe Severe
Fluid requirement Less More
Insulin requirement More Less
Typical complication Cerebral edema Thromboembolism
Mortality More Less

However, one must appreciate that despite these points of differences,


clinically DKA and HHS are not two water-tight entities and in a particular
individual there may be considerable overlap.
Treatment of HHS consists of rehydration, replenishment of electrolytes and
insulin therapy similar to DKA; however in HHS the fluid deficit is more and
insulin requirement may be less.
The notable complications of HHS are thromboembolism and
rhabdomyolysis.

Conclusion
The acute metabolic complications of diabetes mellitus, viz DKA and HHS
are largely preventable life-threatening situations. Proper diabetes education and
treatment may prevent these complications. Should, however, they occur, correct
diagnosis, and prompt evaluation and treatment following established protocols

87
Hyperglycemic Emergencies: Diagnosis and Management

are key to success. Treatment of these emergencies should not be the prerogative
of the specialist alone; every physician should be able to diagnose and treat such
patients so as to minimize morbidity, mortality and treatment cost.
l Hyperglycemic emergencies of diabetes are DKA, common in children
with type 1 diabetes and HHS, common in middle aged and elderly with
type 2 diabetes.
l Both are characterized by severe hyperglycemia, osmotic diuresis,
dehydration and dyselectrolytemia; significant ketonemia and acidosis
occurs in DKA only.
l Frank coma occurs only in a minority of patients, depending upon severity
of metabolic derangement and serum osmolality; hence the terms ketotic
coma and hyperosmolar coma are best avoided.
l Fluid and electrolyte repletion and low dose insulin therapy constitute the
mainstay of treatment which can be undertaken even in a primary care
setting.
l Proper education and treatment of diabetes can largely prevent these life-
threatening metabolic complications.

References
1. Chapman J, Wright AD, Nattrass M and FitzGerald MG. Recurrent diabetic ketoacidosis.
Diabet. Med. 1998;5:659-661.
2. Hale PJ, Crase J and Nattrass M. Metabolic effects of bicarbonate in the treatment of
diabetic ketaoacidosis. BMJ 1984;289:1035-1038.
3. Koller E, Schneider B, Bennett K and Dubitsky G. Clozapine-associated diabetes.
Am. J. Med. 2001;111:716-723.

88
Type 2 Diabetes Mellitus Going Auti B Rajendra
Rajendra Pradeepa
Through Pregnancy Viswanathan Mohan

Introduction
Diabetes is becoming an epidemic in India which is showing an alarming
rise in the number of people with this disorder, especially type 2 diabetes,
with the number of new cases increasing and occurring at a younger age due
to obesity. With its population over 1 billion, India leads the world with its
largest number of diabetic subjects (31.7 million). This figure is predicted to
rise to 79.4 million in 2030.1,2 Consequently the number of cases arising in
women of child-bearing age is increasing leading to an increasing number
of pregnant women with type 2 diabetes and is associated with an increased
risk of maternal and perinatal morbidity and mortality. It is estimated that
diabetes complicates between 1-20% of all pregnancies worldwide. Of all the
diabetes seen during pregnancy about 10% are pregestational diabetes (i.e.,
diabetes which precedes the pregnancy), while the majority (90%) represents
gestational diabetes (i.e., diabetes brought on by pregnancy). Pregnancy in
women with pregestational diabetes is especially high risk. In this chapter,
we will review this issue, and discuss the effects of type 2 diabetes on
pregnancy.

Physiological changes in normal pregnancy and


associated metabolic derangements in type 2 diabetes
complicating pregnancy
In normal pregnancy there is facilitated anabolism in fed state, which is
characterized by insulin resistance. Insulin resistance is progressively increased
during pregnancy and parallels the growth of fetoplacemental unit and level
of hormones secreted by placenta i.e., human placental lactogen, progesterone,
prolactin and cortisol along with maternal factors like food intake, increased
adiposity and physical inactivity. The physiological response to this is the
characterized hyperinsulinemia of pregnancy, due to over production of insulin
by pancreatic b-cells to overcome the excess of counter regulating hormones.
Fasting state in pregnancy is characterized by accelerated catabolism. There is

89
Type 2 Diabetes Mellitus Going Through Pregnancy

reduction in circulating glucose concentration in the postabsorptive and the


fasting condition along with accelerated lipolysis and ketogenesis, which is more
marked towards the end of the pregnancy i.e., in the third trimester. Thus, the
glucose will be utilized by the growing fetus while the free fatty acids and
ketone bodies will be utilized by the mother.

Metabolic changes in pregestational diabetes


A relative or absolute deficiency of insulin leaves the metabolic effects of
fetoplacental unit on maternal cholesterol and fat metabolism, unchecked. Hence,
there is hyperglycemia, increased ketones, altered metabolic levels and increased
amino acid levels which affects the fetoplacental development. Thus, the salient
feature of type 2 diabetes affecting pregnancy, are characterised by accelerated
insulin resistance to greater extent towards end of pregnancy, increased insulin
requirement upto 50-100% greater than normal, increased lipolysis in later half
of pregnancy and increased risk of diabetic ketoacidosis (DKA) in the latter
half of pregnancy.
Though glucose measurement is the usual bench mark for metabolic assessment,
other metabolities like ketone bodies also have their impact on embryonic and
fetal development.

Impact of maternal diabetes on pregnancy


The metabolic disturbances of diabetes are responsible for disturbances in
fetal growth as well as obstetric complications along with worsening of diabetic
complications. Etiological factors responsible for diabetic embryopathy include
altered metabolic fuels, maternal hyperglycemia as well as hypoglycemia, maternal
hyperketonemia, free oxygen radicals and genetic susceptibility.

Effect on the fetus


Diabetes in a pregnant woman can be detrimental to her fetus for four
reasons. First, diabetic women have increased spontaneous abortion rates compared
with the rates in non-diabetic pregnant women. This increased risk however,
can be reduced if near euglycemia is achieved at the time of conception and
the during the first trimester. Second, prevalence of major congenital anomalies
including neural tube defects and cardiac anomalies; affected tissues are formed
during the early part of the first trimester period of organogenesis. The other
two negative consequences that diabetes may have on the fetus are macrosomia
(traditionally defined as birth weight in excess of 4,000 g or above the percentile
for gestational age) and neonatal morbidities. Figure 1 depicts the fetal problems

90
Management of Type 2 Diabetes

associated with maternal hyperglycemia during different periods of gestation.


Macrosomia can make delivery more difficult and thus can lead to increased
rates of cesarean or shoulder dystocias during vaginal deliveries3.

Figure 1: Fetal Problems Associated with Maternal


Hyperglycemia during Different Period of Gestation

Maternal hyperglycemia

Effects on the Fetus

First trimester Second trimester Third trimester

Respiratory distress syndrome Hyperbilirubinemia


Placental insufficiency Hypertropic cardiomyopathy Hypomagnesemia
Growth Retardation Polyhydramnios Intrauterine death
Malformations Pre-eclampsia Hypocalcemia
Fetal wastage Erythraemia Hypoglycemia
Fetal loss Macrosomia

Effect on the pregnant woman


Hypoglycemia may occur in the first trimester of the pregnancy, due to
combincation of physiological adaptation, attempt for strict control and nausea
of early pregnancy. Traditionally both proliferative retinopathy and nephropathy
were considered contraindications to pregnancy. Background retinopathy often
worsens, probably because of the rapid correction of hyperglycemia a situation
also observed in non-pregnant diabetic women. If the vision threatening
proliferative retinopathy or macular edema are treated with laser before or during
pregnancy, the eyes remain quiescent without further deterioration.
In women with diabetic nephropathy, increased proteinuria and a rise in
blood pressure are common and this may require preterm delivery; irreversible
deterioration in renal function occurs occasionally. Less than 5% of diabetic
women who become pregnant have overt nephropathy (i.e., dipstick positive or
clinical proteinuria). Clinical proteinuria usually increases during pregnancy but
returns to prepregnancy levels after delivery. Efforts must be made to control
hypertension as it may lead to pre-eclampsia and hydramnios.
Peripheral neuropathy has no bearing on the outcome of pregnancy. However,

91
Type 2 Diabetes Mellitus Going Through Pregnancy

symptomatic autonomic neuropathy involving the stomach (gastroparesis


diabeticorum) can cause intractable vomiting, with its attendant risk to metabolic
control and nutrition. Myocardial infarction is infrequent in women of child-
bearing age, occurring with an estimated incidence of 1 in 10,000 pregnancies,
in the general population. Macrovasular disease per se presents no risk to fetus
but has led to the increased morbidity of pregnant diabetic women who have it.
Hence careful cardiac evaluation is recommended for such women contemplating
pregnancy.
DKA in pregnancy, overall incidence is low. The predisposing factors
are usually, b-agonist use, emesis, poor management, patient non-compliance,
infection, corticostroids and insulin pump failure4.

Management of type 2 diabetes complicating


pregnancy
Management of diabetic pregnancies should begin before conception. The
9 months of pregnancy represents a period of considerable commitment for a
diabetic woman. In pregestational diabetic pregnancies, the goal of therapy is to
maintain normal maternal pre- and postprandial glucoses to avoid the immediate
and long-term risks to the offspring. The trade-off of tight blood glucose control
is an increased risk of hypoglycemia, often observed in type 1 patients, rather
than in those with type 2 or gestational diabetes.

Prepregnancy counseling
Prepregnancy counseling (Table 1) to the diabetic women should include a
frank discussion of how pregnancy will affect the complications of diabetes in
both the near and long-term. Till the best possible HbA1C levels are achieved along

Table 1. Prepregnancy counselsing


Understanding the risks Perinatal mortality
Congenital anomalies
Maternal mortality
Diabetic complications in pregnancy
Obstetric complications
Inheritance of diabetes in offsprings
What a diabetic pregnancy involves Frequent antenatal visits and close
supervision
Strict blood glucose control (HbA1C ~6%)
Appropriate diet
Other advice Folic acid supplement
Contraception

92
Management of Type 2 Diabetes

with good metabolic control, low dose estrogen, combined with contraceptive
pills along with barrier methods can be used as safe contraceptive method.

Prepregnancy control and assessment


The diabetic women needs to understand many issues regarding a future
pregnancy, while the doctor needs to know much about the individual in order
to give appropriate advice. The careful assessment of diabetic child-bearing age
women includes history of duration of diabetes, medications, insulin regimen
status, dietary adherence, exercise, glycemic status, history of DKA, infections,
hypoglycemic episodes, chronic diabetic complications and thyroid assessment.
The menstrual history, polycystic ovarian disease (PCOD), bad obstetric history,
contraceptive use along with assessment of psychosocial status, support network,
stress factors, family members, patient compliance are helpful in preparation of
patient to face pregnancy5.
The treatment goals has to be decide if the goals are not being meet, all
that is needed is, the revision of management plan, reassessment of goals and
frequency of visits. In patients with type 2 diabetes one can aim for HbA1C’s
around 6% and repeat HbA1C till conception occurs, at 6-8 weeks intervals. If
conception does not occur, reassessment from fertility stand point is required.
Metformin in PCOD diabetic patients is useful. No tetratogenecity is noted even
if pregnancy occurs. Glitazone should be avoided in child-bearing age.

Management
Monitoring and insulin therapy
All women with pregestational diabetes should be seen early in the first
trimester, to optimize glycemic control during the critical period of organogenesis.
The standard care for monitoring glucose metabolism in pregnant women is self-
monitoring of blood glucose (SMBG) levels atleast four times a day, at breakfast
and 1 hour after each meal. Emphasis is on minimizing the postprandial peaks
of glucose, as they have the greatest adverse influence on fetal development.
Blood glucose objectives allow as limits: 70-100 mg/dl before meals, upto
140 mg/dl 1 hour and 120 mg/dl 2 hours after meals. Fructosamine assay and
HbA1C serve as indicators of glycemic control in pregnancies complicated by
diabetes. Fructoasamine is associated with glycemic control over the previous
1-3 weeks possibly making it more appropriate marker for glycemic control.
Insulin requirements usually change during pregnancy. The goal of insulin
therapy is to prevent premeal and postprandial hyperglycemia and to avoid
debilitating hypoglycemic reactions. Human insulin which are least immunogenic

93
Type 2 Diabetes Mellitus Going Through Pregnancy

should be used and SMBG should guide the doses and timing of the insulin
regimen as it prevents antibody formation. In general, women with type 2
diabetes should change treatment to insulin prior to conception or early in the
first trimester because the safety of currently available oral antidiabetic agents
has not been firmly established. They may require multiple-injection regimens
employing short- and intermediate-acting insulin. The short-acting analogs are
safe in pregnancy. Insulin requirements in pregnant women with type 2 diabetes
are generally ~0.9 U/kg/day during the first trimester (i.e., comparable with
type 1 diabetes women), but rise to 1.6 U/kg/day during the second and third
trimesters, significantly higher than in type 1 diabetes (1.2 U/kg/day)6.
The continuous subscutaneous insulin infusion pump (CSII) is consider the
best to achieve excellent control of glycemia. It delivers, continuous basal insulin,
along with bolus insulin as per requirement. Hence it avoids mean glucose
excursions as well as hypoglycemia. The elimination of need for multiple daily
injection and it’s portable size, allows ambulatory use. Cost is the only limiting
factor, hence cannot be used widely.

Nutrition therapy
While managing hyperglycemia of pregnancy, one should aim to provide
adequate nutrition to fetal and maternal health so as to promote appropriate
weight gain with maintainence of euglycemia and prevent ketoacidosis. Dietary
prescriptions are individualized for prepregnancy body weight to height, activity
level, and ethnic and personal preferences.
Unless a woman begins pregnancy with depleted body reserves, energy
needs do not increase in the first trimester. An additional 300 kcal/day are
suggested during the second and third trimester for increases in maternal blood
volume and increases in breast, uterus and adipose tissue, placental growth, fetal
growth and amniotic fluids. The calories are distributed as 60-65% carbohydrates,
<20% fat, and 1.0-1.5 g/kg ideal body weight protein. Routine iron, folic acid,
calcium supplementation is advised. The bed time snack is mandatory to prevent
nocturnal hypoglycemia. Non-caloric sweeteners may be used in moderation.
The restriction of sodium is advised if hypertension/edema is present. At any
given point no hypocaloric diet is advised7.

Other therapies (oral agents and exercise)


Oral hypoglycemic agents are unlikely to achieve sufficiently good glycemic
control, especially in the face of worsening insulin resistance. Patients with type
2 diabetes at the time of (unplanned) conception are often on sulfonylurea agents

94
Management of Type 2 Diabetes

and/or metformin. Oral agents should be discontinued as soon as pregnancy is


confirmed and they should be started on insulin regimen. However glibenclamide
and metformin happens to be safe, as they cross placenta and secretes in breast
milk to a negligible amount, but not recommended routinely8. If a pregnancy is
planned by a woman with type 2 diabetes, the oral agent can be discontinued
and the patient begun on insulin before conception9.
Programs of moderate physical exercise have been shown to lower maternal
glucose concentrations in women with diabetes. The review of exercise plan,
timing, duration and intensity before conception is mandatory to achieve glycemic
control. Prepregnancy exercise program should include 30-45 minutes moderate
exercise daily. Continuation of exercise during pregnancy and avoiding exercise
in supine position in first trimester to prevent hypotension is necessary. Palpation
of uterus to detect contractions during exercise and if positive, discontinuation
of exercise is advised. Checking blood sugar level before exercise program and
avoiding exercise at the time of peak of insulin action is necessary to prevent
hypoglycemia.
The absolute contraindications for exercise during pregnancy are preterm
labor, premature rupture of membrane, incompetent cervix, persistent second
and third trimester bleeding, intrauterine growth restriction, placenta previa
>26 weeks and preganacy induced hypertension. While, the warning signs
to stop exercise are, vaginal bleeding, faintness, decreased fetal activity,
generalized edema and low back pain10.
Post partum management
The post partum period in women with diabetes allows both the physician
and mother to relax from the intensive medical and obstetric management
as insulin resistance sharply comes down. The plan should be individualized
to address glycemic management and surveillance, nutritional management,
contraception prescription, future pregnancy planning and lifestyle changes.
All post partum patients should be encouraged to breast feed. Insulin dosages
are usually lowered in intrapartum and in post partum period. Patient can be
discharged from hospital between 2-4 days of post partum depending upon
mode of delivery. Regular SMBG and additional adjustments in daily insulin
needs is required.

Conclusion
Preconception counseling and intensive therapy regimens remain the focus
of management programs targeted at women with diabetes. Contraception until
tight blood glucose control is one of the prerequisites and careful surveillance

95
Type 2 Diabetes Mellitus Going Through Pregnancy

and management of associated disorders such as retinopathy, nephropathy and


chronic hypertension are mandatory. This needs multidisciplinary approach where
patients and family members are active member of team along with diabetologist,

Case Study
l A 30-year-old woman, with 6 years duration of type 2 diabetes, on oral antidiabetic
drugs [OHA] with a bad obstetric history (BOH) was referred to our center for
prepregnancy glycemic evaluation. Her body mass index was 28.6 and she had
a BOH of two spontaneous abortions and one still birth. Investigations revealed
her glycemic control was suboptimal, confirmed by a HbA1C value of 9%, and a
fasting and postprandial blood sugar levels of 200 and 268 mg/dl, respectively.
Diabetes related complications included mild non-proliferative diabetic retinopathy
in both eyes and micralbuminuria.
l She was counselled to postpone her pregnancy until good glycemic control was
achieved. OHA was withdrawn and insulin therapy, diet and exercise was initiated.
Within 6 months her HbA1C fell to 6.5%, she tested micralbuminuria negative,
while non-proliferative diabetic retinopathy showed no pregression. Subsequently,
she became pregnant.
l Her insulin requirement prior to pregnancy was 16 units, which was increased to 30
units in the first trimester, to 84 units in third trimester. Additionally she was advised
SMBG, counterbalanced with diet and exercise. Clinically her ocular status, blood
pressure and microalbuminuria were monitored. Post partum glycemic evaluation
at our center revealed both mother and child to be in good condition with the
full-term neonate weighing 2.8 kg. Post partum maternal insulin was decreased
to 26 units and she was encouraged to breastfeed. This case illustrates and
emphasizes the importance of optimizing and monitoring glycemic control prior to
and throughout pregnancy to reduce pregnancy-related maternal and fetal risk.

Key Points
1. It is estimated that diabetes complicates between 1-20% of all pregnancies
worldwide. Of all the diabetes seen during pregnancy about 10% are pregestational
diabetes while the majority (90%) represents gestational diabetes. Pregnancy in
women with pregestational diabetes is especially high risk.
2. Pregestational diabetic women must achieve normoglycemia before pregnancy in
order to minimize the risk of spontaneous abortion or fetal malformation and to
prevent pregression of diabetic complications during pregnancy.
3. Contraception until tight blood glucose control is achieved.
4. Management of diabetes in pregnancy includes SMBG, frequent HbA1C testing,
carbohydrate restriction, exercise and insulin therapy.
5. A multidisciplinary approach where patients and family members are active member
of team along with diabetologist, obstetrician, diabetes educators and nutritionist
is mandatory for successful pregnancy outcome.

96
Management of Type 2 Diabetes

Learning Points
1. Initiate insulin therapy in preconceptional care.
2. Type 2 diabetic women should maintain a HbA1C of <6.0% in preconception
period.
3. Preprandial glucose concentrations during pregnancy should be kept to <100 mg/dl,
1-hour and 2-hour postprandial glucose concentrations should be no >140 mg/dl
and 120 mg/dl, respectively to minimize the risk of macrosomia.
4. Warning signs to stop exercise during pregnancy are, vaginal bleeding, faintness,
decreased fetal activity, generalized edema and low back pain.
5. Insulin requirement falls during intrapartum and post partum period.

obstetrician, diabetes educators and nutritionist. Therefore improvement of


pregnancy planning, adequate metabolic control from conception to delivery
can improve fetal and maternal outcomes in those pregnancies complicated by
type 2 diabetes mellitus. Furthermore, diabetes screening in high-risk women
prior to pregnancy is warranted.

References
1. Wild S, Roglic G, Green A, et al. Global prevalence of diabetes: Estimates for the
year 2000 and projections for 2030. Diabetes Care 2004;27:1047-1053.
2. Pradeepa R and Mohan V. The chaning scenario of diabetes epidemic: Implications
for India. J. Indian Med. Assoc. 2002;100:144-148.
3. Kitzmiller JL and Davidson MB. Diabetes and Pregnancy. In: Diabetes Mellitus Diagnosis
and Treatment 4th Edition, Davidson MB (Ed.), WB Saunders Company, London 1998:
313-343.
4. Kamalakannan D, Baskar V, Barton DM Abdu TAM. Diabetic ketoacidosis in
pregnancy. Postgrad. Med. J. 2003;79:454-457.
5. American Diabetes Association. Preconception care of women with diabetes. Position
Statements. Diabetes Care 2004;27:S76-S78.
6. Langer O, Aanyaegbunam A, Brustman L, et al. Pregestational diabetes: Insulin
requirements throughout pregnancy. Am. J. Obstet. Gynecol. 1988;159:616-621.
7. American Diabetes Association. Nutrition principles and recommendations in
diabetes. Diabetes Care 2004;27:S36.
8. Homko CJ, Sivan E and Reece AE. Is there a role for oral antihyperglycemics
in gestational diabetes and type 2 diabetes during pregnancy? Treat Endocrinol. 2004;
3:133-139.
9. Pradeepa R and Mohan V. Recent trends in the management of diabetes complicating
pregnancy. In: Management of Obstetrics and its Related Problems Varma TR (Ed.), Jaypee
Bros, New Delhi 2004:33-47.
10. Harris GD. Diabetes management and exercise in pregnant patients with diabetes.
Clinical Diabetes 2005;23:165-169.

97
Diabetic Retinopathy Chandra Kumar
Sri Ganesh
Gauri Kharosekar

Definition
Diabetic retinopathy (DR) is a micro-
angiopathy of the retinal arterioles, capillaries
and the venules, which occurs due to long
standing systemic disease diabetes mellitus
(DM).

Prevalence and incidence in India


The prevalence of DR in patients with type 2 DM is much lesser as compared
to type 1 DM patients.
In young/juvenile type 1 diabetics DR doesn’t occur upto 3-5 years of onset
of systemic disease. Similar results have been seen for type 2 diabetics; but the
time and duration of diabetes have been difficult to determine precisely.
In patients with DM diagnosed before 30 years of age incidence of DR is
50% at 10 years and 90% after 30 years.
In a population-based assessment of DR in an urban Indian population
according to BJO-99.
 1.8% of people more then 30 years had DR.
 89.3% had mild-moderate non-proliferative diabetic retinopathy (NPDR) out
of which 12% had clinically significant macular edema (CSME).
 One-third of those had visual acuity <6/12.
 10.7% patients had proliferative diabetic retinopathy (PDR).
In one more study in an urban Indian population in October 96 to
June 97.
Total of 2,522 subjects were interviewed and examined, of these 124 had
reported DM.
 (22.4%) 28 were >30-year-old had DR.
 (50%) 14 had mild NPDR.
 (39.3%) 11 had moderate NPDR.
 (7.1%) 2 had severe NPDR.
 (3.6%) 1 had PDR.
 Of the 28 subjects - 4 (14.3%) had CSME.

98
Management of Type 2 Diabetes

In one more study for assessment of prevalence of retinopathy in newly


diagnosed diabetic patients in Southern India.
 Four hundred and forty-eight of newly diagnosed type 2 DM patients.
 Of these 32 had DR.
Conclusion: Overall prevalence of retinopathy at diagnosis among clinic
based South Indian patients. With type 2 DM appears to be lower than that
reported among Europeans.

Pathogenesis
DR is a microangiopathy primarily affecting precapillary arterioles;
capillaries and postcapillary venules.
Exhibits features of both microvascular occlusion and leakage.

Microvascular occlusion

Capillary changes Hematological changes


 Thickening of basement membrane  Deformation of RBCs
 Damage and proliferation of endothelial  Increased rouleax formation
cells and aggregation  Increased platelet stickiness
 Loss of pericytes

Occlusion leads to retinal ischemia.

Two consequences of retinal hypoxia due to ischemia


1) Arteriovenous shunts-associated with capillary dropouts called as intraretinal
microvascular abnormalities (IRMA).
2) Neovascularization-caused by growth factors (vasoformative substances)
elaborated by hypoxic retinal tissue in an attempt to revasularize the hypoxic
retina. Vascular endothelial growth factor is recognized to be responsible.

Leakage
Breakdown of inner blood retinal barrier (tight junctions between the
endothelial cells) → leakage of plasma constituents into the retina.
This results in saccular outpouchings of the
vessel wall → termed microaneurysms.
These may leak or may become
thrombosed.
Increased vascular permeability leads to
h’rrages and edema.

99
Diabetic Retinopathy

Hemorrages
 Superficial h’rrages (flame shaped) - present in
the nerve fiber layer.
 Deep h’rrages (dot and blot) - present in the
outer plexiform layer (Henle’s layer).

Diffuse retinal edema


Occurs due to extensive capillary dilatation
and leakage.

Localized retinal edema


Caused by focal leakage.
Chronic localized edema leads to deposition of hard exudates at the junction
of normal and edematous retina.
These are composed of lipoprotein and lipid filled macrophages.
Cotton Wool Spots – present in severe NPDR are due to focal infarcts of
the nerve fiber layer due to interruption of axoplasmic transport.

Risk factors
Duration of diabetes
More than the control of diabetes it’s the duration of diabetes, which has
been found to be more important. More the duration more the chance of
having DR.

Systemic hypertension
In addition to diabetes if hypertension is associated, it is seen to aggravate
the onset and progression of retinopathy changes, more so if uncontrolled blood
pressure levels.

Associated diabetic nephropathy


Associated nephropathy or changes in the renal function tests is also found
to affect the progression of retinopathy.
Its also found that renal transplant could improve DR.

Pregnancy
Pregnancy associated with systemic diabetes or gestational diabetes; also
increases the progression of DR due to hormonal changes.

100
Management of Type 2 Diabetes

Fluctuating levels of sugars


Fluctuating blood sugar levels are known to progress the time in which DR
starts and also progress the DR changes.

Modifiable factors like smoking, alcohol and hyperlipidemia


Smoking and alcohol consumption increases the arteriosclerosis changes
and increases sugar levels thus; progression of DR. Hyperlipidemia is also
associated with progression of DR.

Certain benefits of intensive metabolic control


 Delays onset of DR; but doesn’t prevent it.
 Slows progression of NPDR.
 Decreases rate of conversion of NPDR to PDR.
 Decreases incidence of macular edema.
 Decreases need for photocoagulation.

Screening

Eye Examination Schedule


Time of Recommended time Routine minimum
onset of DM for first visit follow up
<30 years of age 5 years after onset Yearly
Age 30 years or older At the time of diagnosis Yearly
Before pregnancy Before/soon after conception Atleast every 3 months

Recommended Follow-up Schedule


Status of retinopathy Follow up (monthly)
No retinopathy/only micro aneurysms 12
Mild/moderate NPDR without macular edema 6-12
Mild/moderate NPDR with macular edema that is not 4-6
clinically significant
Mild/moderate NPDR with CSME 3-4
Severe/very severe NPDR 3-4

Classification
There are many described classifications of DR but the routinely and most
followed of them all is the modified ETDRS (Early Treatment Diabetic Retinopathy
Study classification).

101
Diabetic Retinopathy

Modified ETDRS classification


Non-proliferative diabetic retinopathy
A. Mild NPDR.
Atleast one microaneurysm.
Definition not met for B, C, D, E, F.

B. Moderate NPDR.
>/= standard photograph no.2A, soft exudates,
venous beading, IRMAs definitely present.
Definition not met for C, D, E, F.

C. Severe NPDR.
H’rrages/microaneurysms >/= standard photograph
no.2A in all four quadrants.
Venous beading in two or more quadrants.
IRMAs >/= standard photograph no.8A in atleast
one quadrant.

D. Very Severe NPDR.


Any two or more of C.
Definition not met for E, F.

Proliferative diabetic retinopathy


Composed of NVD or NVE or both, preretinal
h’rrage or fibrous tissue proliferation.

NVD NVE
E. Early PDR.
New vessels.
Definition not met for F.

102
Management of Type 2 Diabetes

F. High risk PDR.


NVD ≥ 1/3-1/2 disc area.
NVD and vitreous or preretinal h’rrage.
NVD ≥ 1/2 disc area preretinal or vitreous h’rrage.

Clinically significant macular edema


1) Thickening of the retina located ≤500 microns from
the center of the macula.
2) Hard exudates with thickening of the adjacent
retina located ≤500 microns from the center of
the macula.
3) A zone of retinal thickening, 1 disc are or larger
in size located ≤1 disc diameter from the center
of the macula.

Treatment options
Argon laser photocoagulation-done in patients with CSME or diabetic
maculopathy.
Focal treatment is preferred
Spot size – 50-100 microns.
Duration – 0.1 second
Sufficient power to obtain moderate intensity burn.

Grid treatment
Used for areas of diffuse retinal edema more than 500 microns from center
of fovea and 500 microns from temporal margin of optic disc.
Spot size - 100-200 microns
Duration - 0.1 second
After this treatment - 70% of eyes achieve stable VA
- 15% show improvement
- 15% may subsequently deteriorate.

103
Diabetic Retinopathy

Pan retinal photocoagulation (PRP)


Done in patients with PDR with or without high risk characteristics.

Aim
Conversion of hypoxic retina to anoxic retina so that the oxygen demand
decreases. Hence new vessels are not formed.
PRP helps in regression of new vessels and prevents complications like
VH and TRD.
Spot size - 500 m with Goldman’s lens.
Duration - 0.05-0.1 second
Power - enough to produce gentle burn.
Initial treatment consists of 2,000-3,000 burns in a scatter pattern extending
from posterior pole to peripheral retina in one or more sessions.

Advanced diabetic eye disease


VH and TRD may need Pars plana vitrectomy with release of traction and
FGE with EL with SO injection.

Diabetic Retinopathy Study


Primary outcome measurement was severe visual loss,
which is defined as VA of <5/200 on two consecutive
follow-ups 4 months apart.

Outcome
Fifty percent or > reduction in rates of severe visual loss in eyes treated with
PRP compared to untreated control eyes during follow up of upto 5 years.

Early Treatment Diabetic Retinopathy Study


Study evaluating photocoagulation and aspirin treatment of diabetic patients
with < high risk PDR in BE.
Primary measurement of outcome was mild visual loss which is drop in
three or more lines of Snellen’s equivalent.
Outcome:
Fifty percent or >reduction in rates of mild visual loss in laser treated eyes
with CSME.
When NPDR becomes severe and reaches high-risk stage, Scatter PRP can
be considered and should not be delayed.
Aspirin did not alter rates of progression of DR nor had any influence on
final visual outcome.

104
Management of Type 2 Diabetes

Diabetic Retinopathy Vitrectomy Study (DRVS)


Trial investigating role of vitrectomy in managing eyes with very severe
PDR.
Outcome:
In type I (IDDM) patients with severe VH benefit of early vitrectomy was
clearly demonstrated but no such advantage was found in mixed or type 2
(NIDDM) patients.

Diabetes Control and Complications Trial (DCCT)


Trial designed to study connection between metabolic control and retinal,
renal and neurological complications in type 1 (IDDM) patients.
Outcome:
Intensive insulin treatment can delay onset and slow the progression of DR
in type 1 patients.

Suggested readings
1. Retina - Vol: 2 - Medical Retina Stephen Ryan.
2. Clinical Ophthalmology Jack Kanski.
3. Basic and Clinical Sciences Course. American Academy of Ophthalmology.
4. Investigative Ophthalmology and Visual Sciences 2005.
5. British Journal of Ophthalmology 2002 Sep.

105
Diabetic Neuropathy
Uday Murdgod

Diabetic neuropathy is defined as the presence of symptoms and/or signs


of peripheral nerve dysfunction in people with diabetes after exclusion of other
causes. It is the most common neurological complication of diabetes and is
probably the leading cause of peripheral neuropathy. Diabetic neuropathy is a
syndrome comprising a series of separate clinical disorders that affect distinct
components of the peripheral nervous system.
The reported prevalence of diabetic neuropathy varies from 5 to 100%
depending on the criteria used for diagnosis, but 50% is a generally accepted
prevalence rate. The incidence of neuropathy increases with the duration of
diabetes and the degree of hyperglycemia. Neuropathy appears to be related to
other microvascular complications of diabetes. By the time a diabetic patient has
severe neuropathy, retinopathy and albuminuria are also usually present.
Diabetic neuropathy leads to great morbidity and mortality resulting in a
huge economic burden for care of the patient with diabetes mellitus.

Classification
Diabetic neuropathies are generally classified according to clinical presentation.
There is a significant overlap between these syndromes.
 Symmetrical polyneuropathies
— Distal sensory or sensorimotor polyneuropathy
— Small fiber neuropathy
— Autonomic polyneuropathy
— Large fiber neuropathy.
 Asymmetrical neuropathies
— Cranial neuropathy
— Truncal neuropathy
— Limb mononeuropathy
— Lumbosacral radiculoplexopathy
— Entrapment neuropathy.
 Combinations
— Polyradiculoneuropathy

106
Management of Type 2 Diabetes

— Diabetic neuropathic cachexia


— Symmetrical polyneuropathies.

Pathology
Mononeuropathies are thought to be due to occlusion of the vasa nervorum.
In symmetrical distal polyneuropathy, the sural nerve shows axonal degeneration,
loss of myelinated fibers and demyelination. There is almost always a vasculopathy
with thickened capillary wall and hyperplastic endothelial cells narrowing or
occluding the lumen. The demyelinating process may be the result of progressive
axonal atrophy or damage to Schwann cells caused by ischemia or metabolic
disturbances.

Pathogenesis
Metabolic and ischemic changes are thought to lead to the development of
neuropathy in diabetes. The pathogenesis may consist of several mechanisms:
Hyperglycemia activates polyol pathway in nerve tissue through the enzyme
aldose reductase leading to accumulation of sorbitol and fructose in nerve.
Activation of aldose reductase depletes NADPH which results in decreased levels
of nitric oxide, a potent vasodilator, and lead to impaired nerve blood flow.
Hyperglycemia induces non-enzymatic glycation of structural nerve proteins.
Advanced glycation end-products can interfere with axonal transport.
Hyperglycemia increases vascular resistance and reduces nerve blood flow
causing endoneurial hypoxia, leading to impaired axonal transport and axonal
atrophy.
Activation of protein kinase C (PKC), initiating a cascade of stress
responses.
Increased free radical formation and oxidative stress may occupy a central
position. This conclusion is based on the observations that antioxidant therapy
improves nerve blood-flow in diabetes, and that ischemic hypoxia promotes
oxidative stress.

Aldose Mitochondrial
reductase dysfunction
pathway Programmed
cell death

↑glucose ↑Oxidative Neuropathy


stress

Aldose
reductase ↑ischemia
pathway Nerve growth
factor

Figure 1. Pathogenesis of diabetic neuropathy.

107
Diabetic Neuropathy

Clinical features
Distal symmetrical polyneuropathy
This is the most common type of diabetic neuropathy. Sensory symptoms
may be positive, such as pain, burning, tingling or shock-like sensations, or
negative, such as numbness. Mild weakness of the distal muscles of the lower
extremities is seen in most patients.
Distal symmetrical polyneuropathy may be further classified depending on
the nerve fiber type most involved: Large-fiber and small-fiber variants.
Involvement of small fibers causes a reduction in pinprick and temperature
sensation in a stocking and glove distribution. The ankle tendon reflex is
usually diminished or absent and in severe disease the knee reflexes may also
be absent.
In advanced neuropathy there may be severe large fiber disease, with
impairment of propioception. In such cases, sensory ataxia associated with a
positive Romberg’s sign is seen.
There may be generalized muscle wasting. Wasting of the small muscles
of the foot results in pressure points at the metatarsal heads, leading to callus
formation and recurrent foot ulceration.
In patients with advanced neuropathy, there is involvement of the upper
limbs. Fine movements of the fingers are affected.
An acute painful neuropathy may be precipitated following initiation
of treatment with insulin (treatment-induced neuropathy). Burning pain and
paresthesias in the distal lower extremities may persist for weeks to months.
In patients with newly diagnosed diabetes, there may be transient pain and
paresthesias in the distal lower extremities (hyperglycemic neuropathy) which
resolve when the hyperglycemia is brought under control.
Diabetic neuropathic cachexia is a term referring to an acute painful diabetic
neuropathy associated with severe weight loss, depression, insomnia and impotence
in men. This is common in men with poor glycemic control.
Sensory loss may lead to repetitive, often unnoticed, injuries, foot ulcers and
distal joint destruction (acrodystrophic neuropathy). Chronic foot ulceration is
one of the more severe complications of diabetes mellitus.

Diabetic autonomic neuropathy


The typical clinical features include orthostatic hypotension, resting tachycardia
or a heart rate unresponsive to respiration. These are caused by failure of the

108
Management of Type 2 Diabetes

sympathetic nervous system and vagal denervation of the heart. An increased


incidence of painless myocardial infarction is seen in these patients.
Other autonomic dysfunctions include gastrointestinal motility abnormalities
such as delayed gastric emptying, diabetic diarrhea or constipation. Bladder
atony leads to urinary retention and finally overflow incontinence. Impotence
is common in men with autonomic neuropathy. It involves both erectile failure
and retrograde ejaculation. Sudomotor abnormalities produce distal anhidrosis.
Gustatory sweating is characterized by profuse sweating in the face and forehead
soon after food intake.

Lumbosacral radiculoplexopathy or asymmetrical proximal


diabetic neuropathy
It usually presents with severe pain in the lower back, hip and anterior thigh.
Asymmetrical weakness and wasting of pelvi-femoral muscles may occur either
abruptly or insidiously. It is uncommon in patients <40 years of age. Weight
loss occurs in more than half of patients. Recovery may take upto 24 months,
because of the slow rate of axonal regeneration. The typical EMG findings include
low amplitude femoral nerve motor responses, prominent fibrillation potentials
in thoracic and lumbar paraspinal muscles, and active denervation in affected
muscles. Sural nerve biopsies show ischemic injury and perivascular infiltrates,
suggesting an immune mechanism. Uncontrolled studies suggest immunotherapy
may be beneficial.

Truncal neuropathy
It usually involves T4-T12 roots, causing pain or dysesthesias in areas of the
chest or abdomen in a highly variable pattern. The clinical picture may mimic
intra-abdominal, intrathoracic or intraspinal disease, or even herpes zoster. The
symptoms persist for several months before subsiding. EMG studies show active
denervation in paraspinal and abdominal muscles.

Limb mononeuropathy
There are two basic mechanisms for single mononeuropathies: Nerve infarction
or entrapment. Neuropathy due to nerve infarction is characterized by abrupt
onset of pain followed by variable weakness and atrophy. The recovery is slow.
Median, ulnar and peroneal nerves are commonly affected. EMG studies show
axonal loss.
Mononeuropathies due to nerve entrapment are more common and
characterized by insidious onset of symptoms. EMG studies show focal
conduction block and/or slowing.

109
Diabetic Neuropathy

Multiple mononeuropathies
This term refers to the involvement of two or more nerves. One nerve
is usually affected acutely with other nerves sequentially involved. Multiple
proximal nerves may be affected leading to “diabetic amyotrophy”. Occlusion
of vasa nervorum leads to nerve infarction. Systemic vasculitis must always be
considered in the differential diagnosis.

Cranial mononeuropathies
The third cranial nerve is the most commonly affected nerve, followed by the
fourth, sixth and seventh nerves. Sparing of the pupil is the hallmark of diabetic
third nerve palsy. Acute ischemia is the cause of these neuropathies. Recovery
usually occurs over 3-6 months. Infectious conditions such as rhinocerebral
mucormycosis and malignant external otitis should be considered in appropriate
circumstances.

Entrapment neuropathy
The possibility of diabetes should be considered in cases of entrapment
neuropathy. An increased risk of carpal tunnel syndrome is seen in diabetic
patients. Asymptomatic electrophysiological abnormalities consistent with carpal
tunnel syndrome are seen three times more commonly than symptomatic carpal
tunnel syndrome in people with diabetes. Ulnar neuropathy at the elbow and
peroneal neuropathy at the fibular head are also seen. Nerve ischemia and
endoneurial hypoxia are presumed to be predisposing factors, though the exact
reason for increased incidence of nerve entrapment in diabetes is unknown. Surgical
decompression should be considered only when there is a significant deficit.

Chronic inflammatory demyelinating polyneuropathy (CIDP)


A few studies have shown evidence that CIDP may be superimposed on
diabetic neuropathy due to a secondary immunological event. Prominent motor
involvement, marked decrease in nerve conduction velocity, elevated cerebrospinal
fluid (CSF) protein with oligoclonal bands, and inflammatory infiltrates in nerve
biopsy confirm CIDP. Treatment is as for isolated CIDP.

Electrophysiological testing
The diagnosis of peripheral neuropathy can be made on a clinical basis.
Electrophysiological studies are not always required to establish the diagnosis.
Nerve conduction studies and electromyography may help confirm the clinical
diagnosis, identify patterns of neuropathy, monitor progress or remission, and
detect asymptomatic cases. In patients with pure autonomic features, or localized

110
Management of Type 2 Diabetes

pain from truncal neuropathy, electrophysiological studies may demonstrate


generalized peripheral nerve involvement. Documentation of electrophysiological
abnormalities in asymptomatic diabetics may be of interest in population studies,
clinical trials and in investigating pathogenesis of diabetic neuropathy.
There is no pattern of electrophysiological abnormalities that is pathognomonic
of diabetic neuropathy. Some patterns are suggestive. In asymptomatic or mild
cases, the only change may be slowing of nerve conduction velocities. In overt
cases, there is usually a combination of changes suggesting both demyelination
and axonal degeneration. Changes of axonal degeneration predominate in advanced
cases. Denervation in paraspinal muscles can be demonstrated in patients with
thoracic or lumbar radiculopathy. In entrapment neuropathy, there is slowing
of conduction across entrapment points.

Treatment of diabetic neuropathy


Treatment may be divided into therapies directed at correcting the pathogenetic
mechanisms and those that offer symptomatic relief. Studies have shown that
optimization of blood glucose levels has a major effect in preventing microvascular
complications, including neuropathy. Therapies include lipoic acid, nerve growth
factor analog and aldose reductase inhibitors.
Symptomatic treatments include physical approaches to prevent foot injuries
and ulcers, and psychological support for the patient. Pain is the major symptom
requiring pharmacological treatment. Tricyclic antidepressants, anticonvulsants
and opiates are the mainstays of therapy for neuropathic pain. Table 2 lists the
drugs used in treatment of painful diabetic neuropathy. Table 3 provides the
treatment options for symptoms related to autonomic neuropathy.

Suggested readings
1. Thomas PK and Tomlinson DR. Diabetic and hypoglycemic neuropathy. In: Peripheral
Neuropathy Dyck PJ and Thomas PK (Eds.), WB Saunders, Philadelphia 1993:
1219-1250.
2. Kelkar P. Diabetic neuropathy. Seminars in Neurology 2005;25:168-173.
3. Stevens MJ, Feldman EL and Greene DA. The etiology of diabetic neuropathy:
The combined roles of metabolic and vascular defects. Diabet. Med. 1995;12:566.
4. Sumner CJ, Sheth S, Griffin JW, et al. The spectrum of neuropathy in diabetes and
impaired glucose tolerance. Neurology 2003;60:108-111.
5. Bosch EP and Smith BE. Disorders of peripheral nerves. In: Neurology in Clinical Practice
Bradley WG, Daroff RB, Fenichel GM and Jankovic J (Eds.), Butterworth Heinemann
2004:2357-2365.
6. Windebank AJ and Feldman EL. Diabetes and the nervous system. In: Neurology and
General Medicine Aminoff MJ (Ed.), Churchill Livingstone 2001:341-364.
7. Llewelyn JG. The diabetic neuropathies: types, diagnosis and management. J. Neurol.
Neurosurg. Psychiatry 2003;74:15-19.

111
Diabetic Neuropathy

Table 1. Features to suggest non-diabetic etiology


for neuropathy in diabetes
Clinical features` Diagnosis
Family history of neuropathy, Pes cavus, Hereditary motor sensory neuropathy
hammeroes
Prominent early autonomic neuropathy Amyloidosis*
Acute onset, CSF protein >100 mg/dl Inflammatory neuropathy (CIDP)*
Elevated ESR, positive rheumatoid factor or Necrotizing angiopathy due to
antinuclear antibody collagen vascular disease*
Monoclonal gammopathy in serum Primary systemic amyloidosis
Myeloma
Lymphoma
Monoclonal gammopathy-associated
neuropathy
*Nerve biopsy indicated

Table 2. Drugs used in treatment of painful diabetic neuropathy


Drug Dose
Antidepressants
Amitriptyline 50-150 mg at night
Nortriptyline 50-150 mg at night
Duloxetine 30-60 mg once daily
Anticonvulsants
Carbamazepine 600-1,400 mg in 2-3 doses daily
Oxcarbazepine 900-2,400 mg in 2 doses daily
Lamotrigine 100-400 mg in 1-2 doses daily
Gabapentin 900-2,400 mg in 3 doses daily
Pregabalin 150-600 mg in 2 doses daily
Opioids
Tramadol 100-400 mg in 3-4 doses daily
Others such as morphine, oxycodone
Mexiletine 150-450 mg daily
Capsaicin cream 0.075% 3-4 times daily
Lidocaine patch
Transcutaneous nerve stimulation

112
Management of Type 2 Diabetes

Table 3. Management of autonomic neuropathy


Postural hypotension Sleeping with head end elevated, full length elastic stockings,
fludrocortisone, ephedrine, midodrine
Gastroparesis Domperidone, metoclopramide, erythromycin, jejunostomy
Diarrhea Tetracycline, codeine phosphate
Cystopathy Frequent voiding, prazosin, intermittent self catheterization
Erectile dysfunction Intracavernous papavarine, sildenafil
Gustatory sweating Propantheline bromide, glycopyrrolate cream

Case Report
A 52-year-old man, known to have type 2 diabetes mellitus, presented with
1 month history of severe burning and tingling paresthesia in both feet. He had similar
but less intense paresthesia in both hands for 2 weeks. The symptoms were worse
at night and disturbed his sleep. He could stand and walk with difficulty due to the
pain. He had stopped working. He was on oral hypoglycemic drugs and diabetes was
poorly controlled. Neurologic examination revealed mild impairment of touch and pain
sensations in both feet and absent ankle reflexes bilaterally; rest of the examination
was normal. Nerve conduction studies showed only mild slowing of conduction velocities
in motor and sensory nerves in the lower limbs. He had received amitriptyline 25 mg/
day and gabapentin 600 mg/day with no relief. Blood counts, ESR, creatinine, electrolytes,
thyroid function tests and serum vitamin B12 levels were normal. Insulin was required to
control diabetes. He required amitriptyline 150 mg and gabapentin 1,200 mg/day, with
the addition of tramadol 150 g daily for 2 weeks, to obtain pain relief. A month later,
the intensity of pain had subsided significantly and he could go back to his job.
This case highlights the fact that diabetic peripheral neuropathy can be extremely
painful and debilitating, and electrophysiological studies may be normal or only mildly
abnormal. It is important to exclude other causes of neuropathy (Table 3) optimize
diabetes control and provide adequate analgesia.

113
Sexual Dysfunction in
Vageesh Ayyar
Men and Women with Diabetes

Introduction
In keeping with many other areas of diabetes care, there have been
considerable advances in our understanding of sexual dysfunction in
diabetes. Sexual function is a complex concept encompassing far more
than the simple sexual act. Infact it includes marital, physiological,
behavioral and relational aspect of human sexual life which are
variously influenced by psychological factors, as well as social and organic
factors.

Male sexual dysfunction


It is now recognized that erectile dysfunction (ED) is perhaps the
most common complication in diabetic men. Until recently, ED was one of
the most neglected complications of diabetes. Luckily, awareness of ED as
a significant and common complication of diabetes has increased in recent
years, mainly because of increasing knowledge of male sexual function and
the rapidly expanding armamentarium of novel treatment options.
A sexually competent male must have a series of events with multiple
mechanisms intact for normal erectile function. He must:
1) Have desire for his sexual partner (libido)
2) Able to divert blood from the iliac artery into the corpora cavernosae to
achieve penile tumescence and rigidity (erection)
3) Able to discharge semen (ejaculation)
4) Experience a sense of pleasure (orgasm).
A man is considered to have ED if he cannot achieve or sustain an erection
of sufficient rigidity for sexual intercourse.

Epidemiology
Despite the variation in populations studied and differing methodologies
used to define ED, the overall prevalence of ED is universally consistent
30-50%, and is far higher than in non-diabetic men where this figure doesn’t
exceed 20%.

114
Management of Type 2 Diabetes

Physiology
Normal male sexual function requires a complex interaction of vascular,
neurological, hormonal and psychological systems. The combination of increased
intracavernosal blood flow and reduced venous outflow allows a man to acquire
and maintain a firm erection. Nitric oxide plays a significant role. It facilitates
the relaxation of intracavernosal trabeculae and thereby maximizing blood flow
and penile engorgement. Low intracavernosal nitric oxide synthase levels are
found in people with diabetes, smokers, and men with testosterone deficiency.
Erections also require neural input to redirect blood flow into the corpora
cavernosae. Psychogenic erections secondary to sexual images or auditory stimuli
relay sensual input to the spinal cord at T-11 to L-2. Neural impulses flow to the
pelvic vascular bed, redirecting blood flow into the corpora cavernosae. Reflex
erections secondary to tactile stimulus to the penis or genital area activate a
reflex arc with sacral roots at S2 to S4. Nocturnal erections occur during rapid-
eye-movement (REM) sleep and occur 3-4 times nightly.

Etiology
The causes of ED are numerous but generally fall into two categories: Organic
or psychogenic. The organic causes can be subdivided into five categories:
Vascular, traumatic/postsurgical, neurological, endocrine-induced and drug-
induced. In people with diabetes, the main risk factors are neuropathy, vascular
insufficiency, poor glycemic control, hypertension, low testosterone levels and
possibly a history of smoking.

Pathology of ED
The natural history of ED in people with diabetes is normally gradual.
And it is both due to autonomic neuropathy and endothelial dysfunction.
There is some evidence to suggest that autonomic neuropathy may
predominate in type 1 diabetes, where as vascular dysfunction may be
more important in type 2 diabetes. In addition to endothelial dysfunction
and autonomic neuropathy, ED is associated with various other conditions
common in diabetes (Table 1).

Significance of ED inpatients with diabetes


Many of the risk factors of ED are the same as those for cardiovascular
disease, which in patients with diabetes may be asymptomatic. ED may be the
first warning sign of underlying cardiovascular problems. ED may be an early

115
Sexual Dysfunction in Men and Women with Diabetes

Table 1. Factors contributing to ED


 Functional and structural changes within the carpora cavernosa

 Neurological factors

 Vascular factors

 Endocrine factors

 Iatrogenic (antihypertensives, antidepressants, tranquilizers, etc.)

 Psychiatric disorders

 Structural penile abnormalities (viz. peyonies disease)

 Venous leaks

 Infections (Balanitis)

sign or symptom of cardiovascular disease. The same vascular endothelial injuries


that occur in the coronary arteries likely occur in the cavernosal arteries, the
primary vessels supplying the penile erectile tissue.

Management of ED
The initial step in evaluating ED is a thorough sexual history and physical
examination. The history can help in distinguishing between the primary and
psychogenic causes. It is important to explore the onset, progression and duration
of the problem. A medical history focused on risk factors, such as cigarette
smoking, hypertension, alcoholism, drug abuse, trauma, and endocrine problems
including hypothyroidism, low testosterone levels and hyperprolactinemia, is
very important. Laboratory investigations should include levels of testosterone,
prolactin and ferritin, among others (Table 2). Primary care physician should
manage the vast majority of patients with ED. However there are several
indications for referrals to specialists (Table 3).
Initially, preventive measures will help reduce the risk of developing ED.
Improving glycemic control and hypertension, ceasing cigarette smoking and
reducing excessive alcohol intake have been shown to benefit patients with
ED. Avoiding or substituting medications that may contribute to ED is also
helpful.

First-line therapy
First-line interventions are characterized by ease of administration, reversibility,
non-invasive, “low cost”, which include.

116
Management of Type 2 Diabetes

Table 2. Evaluation and management of ED

Medication history: It is extremely important to consider side effects and drug


interactions
 Antihypertensives: b-blockers, thiazide diuretics
 Agents acting on the central nervous system: Tricyclic antidepressant drugs,
selective serotonin reuptake inhibitors, phenothiazines, butyrophenones, atypical
antidepressants
 Agents affecting the endocrine system: Anti-androgens, GnRH agonists and
antagonists, estrogens, metoclopramide, fibric acid derivatives, alcohol, Marijuana

Hormonal status
 Leutinizing hormone, follicle-stimulating hormone, prolactin
 Serum testosterone
 Ferritin (to evaluate for hemochromatosis)

Autonomic neuropathy
 ECG (R-R variability), heart rate variability
 Orthostatic blood pressure readings

Vascular disease
 Doppler studies of penile blood flow
 Pharmacodynamic testing using vasoactive compounds
 Pudendal angiography and cavernosometry

Psychosocial assessment
 Combine with nocturnal penile tumescence test
 Marital counseling

Table 3. Referral to specialist

 Young patient with presumed pure cavernosal artery insufficiency secondary to


pelvic/perineal trauma
 Patient with significant penile curvatures
 Patients with aortic aneurysms or bulbo sacral disease
 Patients with endocrinopathies
 Patients with psychiatric or psychosexual disorders
 Patient or physician request for specialized evaluation
 Medical or legal reasons

117
Sexual Dysfunction in Men and Women with Diabetes

Oral erectogenic agents


Phosphodiesterase type 5 (PDE-5) inhibitors: These drugs are potent and
selective inhibitors of cyclic guanosine monophosphate (cGMP) specific PDE-5.
They prevent the breakdown of cGMP and prolong and improve smooth muscle
relaxation. Sildenafil was the first drug in this class, followed by the newer
agents tadalafil and vardenafil. Sildenafil citrate reported to improved erections
in 59% of those with type 1 diabetes and 63% of those with type 2 diabetes.
Improvement was noted regardless of age, race, ED severity and duration, or
presence of various comorbities. Sildenafil should be taken 1-2 hours before
intercourse. The initial dose for sildenafil is 50 mg, and the dose can be increased to
100 mg. Side effects of sildenafil include headaches, lightheadedness,
dizziness and flushing. Syncope and myocardial infarction, the most serious
side effects, are seen in men who are also taking nitrates for coronary heart
disease. Sildenafil is strongly contraindicated in men who take nitrates. with
active coronary ischemia, congestive heart failure, complicated multidrug
antihypertensive regimen, who use drugs that prolong the half-life of sildenafil
by blocking CYP3A4.
a2-adrenergic receptor blocker: Another oral treatment that has been
used with very little success is yohimbine. This is an a2-adrenergic receptor
blocker that increases cholinergic and decreases adrenergic tone. It stimulates
the mid-brain and increases libido. Optimal results occur when used in men
with psychogenic ED. Side effects include anxiety and insomnia.

Vacuum-constriction devices
Vacuum tumescence devices work irrespective of the underlying etiology
of ED. Diabetic men with ED report a success rate of 75%. Most men find the
technique acceptable, especially if they have tried and failed oral therapy. Some
individuals may find it cumbersome. It can also be added on to one of the other
treatment modalities to enhance a partial response.

Second-line therapies
Second-line therapy is indicated in case of partial or minimal response to
first-line therapy or when drugs are not well tolerated.

Intracavernosal therapy
Several vasoactive substances can be used to stimulate the erectile process.
These substances can be delivered directly into the corpora cavernosa by
injection. Papaverine is a non-specific PDE, and alprostadil is a prostaglandin
E1 derivative. These two drugs, when given via the intracavernosal route,

118
Management of Type 2 Diabetes

relax the smooth muscle of the corpora cavernosa. Another agent in use is
phentolamine. Phenotolamine is a competitive inhibitor of a-adrenergic receptors,
which reduces sympathetic tone. Some practitioners use a combination of
these agents. For ED, intracavernosal therapy has a success rate of ~80–90%.
However, half of the men discontinue the use of intracavernosal injections
due to pain, loss of effect, or lack of interest.

Intraurethral prostaglandin therapy


An intraurethral alprostadil suppository system was developed in an
attempt to avoid the problems and issues related to injection therapy. Although
it avoids the side effects of injection therapy, it may cause urethral pain in
~30% of the users. It has not been effective in those who have failed injection
therapy.

Third-line therapy
Surgery
With the availability of various newer treatment modalities, the use of
penile prostheses (Fig. 1) has declined. However, there is an 86% success rate
at 5 years, and 91% of erections are suitable for coitus. Diabetes, however, poses
a risk for prosthesis-associated infection and can often necessitate the removal
of the prosthesis and possible worsening of the primary problem. Rarely,
severely compromised blood flow could lead to device failure. Revascularization
might help some of these patients, but it is difficult to select patients with a
predictable good outcome.
Also, revascularization is
relatively contraindicated
in men with diabetes. In
some patients, there could
be venous incompetence,
which can be improved by
ligation of the deep dorsal
vein and any incompetent
circumflex veins. Surgery
should be reserved for
clear-cut cases of vascular
or venous insufficiency
in young patients with
Figure 1. Penile prosthesis. recent-onset diabetes.

119
Sexual Dysfunction in Men and Women with Diabetes

Conclusion on male sexual dysfunction


Knowledge of sexual dysfunction is rapidly expanding, and effective
new treatments are now available, including oral medications, injectable
drugs, vacuum devices and inflatable prostheses. It is therefore important
for both physicians and patients to be educated and aware of the causes and
treatments of ED. ED is an under-recognized, under-discussed, and commonly
untreated complication of diabetes. But it is also one of the most treatable
diabetic complications. It is a “couples disorder,” affecting both the patient
and his partner.

Female sexual dysfunction


Female sexual dysfunction in diabetes is less studied than male sexual
dysfunction. In physiological terms the female equivalent of male ED is
reduced vasocongestion of the vulva and vagina leading to impaired arousal
and reduced vaginal lubrication.
The reported prevalence of impaired sexual arousal and inadequate
lubrication is anywhere between 14-45% which is significantly higher than
non-diabetic women. It is universal experience that women with diabetes
rarely complain of sexual problems. There has been very little research
into the pathophysiology of sexual dysfunction in women. But several
studies have reported a little surprisingly, that there does not appear to
be a strong relationship between neuropathy and female sexual dysfunction.
The mechanism of action of PDE-5 inhibitors would suggest they might
improve arousal and vaginal lubrication. There have been no studies
published yet on the effect of sildenafil on sexual dysfunction in diabetic
women, but it is unlikely to have such a dramatic impact as in diabetic men
with ED.
Sexuality is an important aspect of quality-of-life, and contrary to
general opinion, women with diabetes are at increased risk for sexual
problems. It is even possible that an arousal problem involving slow or
inadequate vaginal lubrication in women with diabetes is an easily treatable
complication of diabetes. Good psychosexual counseling in which patients can
discuss their embarrassment about asking about, buying, and using vaginal
lubricants, can help women overcome the painful sexual problems they may
be facing.

120
Management of Type 2 Diabetes

Suggested readings
1. Thethi, TK, Asafu-Adjaye NO and Fonseca VA. Erectile dysfunction. Clinical Diabetes
2005;23:105-113.
2. Chu NV and Edelman SV. Diabetes and erectile dysfunction. Clinical Diabetes 2001;
19:45-47.
3. Penson DF and Wessells H. Erectile dysfunction in diabetic patients. Diabetes
Spectrum 2004;17:225-230.
4. Morales A. Erectile dysfunction: An overview. Clin. Geriatr. Med. 2003;19:529-538.
5. Enzlin P, Mathieu C and Demytteanere K. Diabetes and female sexual functioning:
A state-of-the-art. Diabetes Spectrum 2003;16:256-259.
6. Cummings MH. Erectile dysfunction in diabetes mellitus. In: International Textbook
of Diabetes Mellitus 3rd Edition, DeFronzo RA, Ferannini E, Keen H and Zimmet P
(Eds.), John Wiley & Sons: Chichester 2004:1333-1342.
7. Munarriz R, Traish A and Goldstein I. Erectile dysfunction and diabetes. In: Joslin’s
Diabetes Mellitus Kahn CR, et al. (Eds.), 14th Edition, Lippincott Williams & Wilkins,
Philadelphia, 2005:999-1015.

121
Diabetic Nephropathy in India Tiwari

It has been predicted that worldwide the prevalence of diabetes in adults


would increase to 5.4% by the year 2025 from the prevalence rate of 4.0% in
1995. Consequently the number of adults with diabetes in the world would
rise from 135 million in 1995 to 300 million in the year 2025. It is obvious so
far that this rise will occur in developing countries. Indeveloping countries like
India, most of the diabetic patients are in the age range of 45–64 years, while
in developed countries most of them are 65 years. Therefore diabetic patients
in developing countries are even more vulnerable to develop the microvascular
complications of diabetes including diabetic nephropathy1.
Diabetic nephropathy is one of the leading causes of chronic renal failure in
India. It has been reported that among 4,837 patients with chronic renal failure
seen over a period of 10 years, the prevalence of diabetic nephropathy was 30.3%
followed by chronic interstitial nephritis (23.0%) and chronic glomerulonephritis
(17.7%)2.
The high prevalence of renal involvement, despite correction for
environmental factors among Asian type 2 diabetic patients, has recently been
recognized, with upto 50% showing albuminuria as compared with 15% in
Western populations3-5.
Asian diabetic patients lose functional renal reserve earlier in the course of
nephropathy than whites due to defective nitric oxide production6.

Enormity of problem - difference from the west


Studies from the UK have found that type 2 diabetes is three to four times
more common in South Asians than in Europeans. Asian-Indians have been
identified as one of the ethnic groups with a high prevalence of type 2 diabetes
and a high familial aggregation of type 2 diabetes. The prevalence of type 2
diabetes was as high as 50% among the offspring of conjugal type 2 diabetic
parents in India, which is the highest prevalence rate reported until now. In a
population-based survey in an urban population in South India, it was found
that there was a 40% increase in the age - standardized prevalence of diabetes
over a period of 6 years, from 8.2% in 1988–1989 to 11.6% in 1994-1995.5

122
Management of Type 2 Diabetes

Diabetic nephropathy is clinically defined by the presence of persistent


proteinuria of >500 mg/day in a diabetic patient who has concomitant
diabetic retinopathy and hypertension and in the absence of clinical or
laboratory evidence of other kidney or renal tract disease. The presence
of diabetic retinopathy is an important pre-requisite because in its
absence, albuminuria in a type 2 diabetic patient may be due to diabetic
or non-diabetic glomerulosclerosis and the chances for both are equal. Nearly
30% of chronic renal failure in India is due to diabetic nephropathy 7.

Early diagnosis of risk factors for diabetic


nephropathy
Diabetic patients with microalbuminuria (MAU) are at a high risk for
developing overt nephropathy and cardiovascular complications. Early screening
for MAU in diabetic patients allows for aggressive intervention with a view to
prevent end-stage renal disease (ESRD).

Management of diabetic nephropathy


The single most important to accelerate progression of diabetic nephropathy
is “hypertension”. A range of drugs such as the a-blockers, b-blockers, calcium
channel blockers, angiotensin converting enzyme inhibitor (ACEI) and angiotensin
receptor blocker (ARB) are accessible to target hypertension. And the golden
key rule is to effectively use the best drug in a tailor-made fashion. ACEI
and ARB are still the first choice of drug for treating hypertension in diabetes
while, others should be used in conjugation when required. Studies like HOPE,
Steno and BRILLIANT study have demonstrated the effectiveness of ACEI in
terms of reducing albumin excretion rate independent of their blood pressure
lowering capacity.
Early screening for MAU is the key for early detection of the devastating
complication. Once identified, intensive treatment to control the blood sugar
should be the primary objective.
A target blood pressure of <130/80 mmHg as recommended by the Joint
National Committee VII should be achieved in all diabetic patients. A bulk
of available evidences such as that blockade of renin angiotensin system has
additional benefits in diabetic nephropathy and that ACEI and ARB should be
used as first-line antihypertensive therapy in diabetic patients with any evidence
of nephropathy8.

Screening for diabetic nephropathy


In type 1 diabetes MAU rarely occurs within 5-10 years of duration or
before puberty. Hence screening should begin with onset of puberty or after
5 years diseased duration.

123
Diabetic Nephropathy in India

In type 2 diabetes, where the precise onset of the disease cannot be


jotted down, screening should begin at diagnosis. In a study conducted in
205 subjects, Vijay, et al. found that 12.2% of patients had persistent MAU
during diagnosis of diabetes itself. Once MAU has been identified the patient
should have measurements every 3-6 months (flow chart). Familial factors
may play a role in the development of diabetic nephropathy. Patients with
positive family history for diabetic nephropathy should be screened at an
earlier stage9.
In 2002 the National Kidney Foundation (NKF) developed a new classification
of the stages of kidney disease of all causes. Physicians can now determine
patients’ stage of renal disease and explain the current disease state more
easily to patients and other healthcare providers. In the new system, the stages
of chronic kidney disease (CKD) are determined by glomerular filtration rate
(GFR) (Table 1).

Table 1. Stages of CKD based on GFR


Stage Description GFR (ml/min)
0 With risk factors >90
I Kidney damage with nromal of decrease GFR >90
II Mild 60-89
III Moderate 30-59
IV Severe 15-29
V Kidney Failure <15

Referral for nephrologist’s consultation


The 2006 ADA position statement states that a referral for renal consultation
should be made if GFR has fallen to <60 ml/min or if difficulties occur in
management of hypertension or hyperkalemia. Physicians caring for individuals
with renal disease should have expertise in treating the comorbidities of CKD,
such as hyperparathyroidism, hyperphosphatemia, hypocalcemia, and metabolic
bone disease, and in the prudent timing of placement of dialysis access and
initiation of renal replacement therapy.
The nephrology community now recommends the estimation of creatinine
clearance as the most reliable practical measure of kidney function in clinical
practice. There are several available formulae, but that formulated by Cockcroft and
Gault (Fig. 1) is most widely used and accounts for body size, sex and age.

124
Management of Type 2 Diabetes

(140 – age) (weight in kg)*


Estimated creatinine clearance = ___________________________
Plasma creatinine × 72
“for females, this value is further multiplied by 0.85.

Figure 1. The Cockcroft-Gault formula for estimating creatinine clearance.

However, the use of this formula to estimate creatinine clearance is only


valid when serum creatinine is relatively stable. It is invalid in acute or subacute
renal failure, when serum creatinine is changing rapidly.
In summary, screening for diabetic nephropathy is best achieved by measuring
the albumin-to-creatinine ratio in spot urine samples and estimating creatinine
clearance from serum creatinine and other known patient characteristics.
Early referral to a nephrologist when estimated creatinine clearance is
<60 ml/min is recommended to allow adequate time to prepare for renal
replacement therapy, dialysis or transplantation, resulting in lower morbidity
and mortality10.

References
1. King H, Aubert RE and Herman WH. Global burden of diabetes 1995–2025. Prevalence,
numerical estimates and projections. Diabetes Care 1998;21:1414-1431.
2. Mani MK. Patterns of renal disease in indigenous populations in India. Nephrology
1998;4:S4-S7.
3. Mather HM, Chaturvedi N and Kehely AM. Comparison of prevalence and risk factors
for microalbuminuria in South Asians and Europeans with type 2 diabetes mellitus.
Diabet. Med. 1998;15:672-677.
4. Chan JC, Cheung CK, Swaminathan R, Nicholls MG and Cockram CS. Obesity,
albuminuria and hypertension among Hong Kong Chinese with non-insulin-dependent
diabetes mellitus. Postgrad. Med. J. 1993;69:204-210.
5. Viswanathan V. Type 2 diabetes and diabetic nephropathy in India: Magnitude of
the problem. Nephrol. Dial. Transplant 1999;14:2805-2807.
6. Earle KA, Mehrotra S, Dalton RN, Denver E and Swaminathan R. Defective nitric
oxide production and functional renal reserve in patients with type 2 diabetes who
have microalbuminuria of African and Asian compared with white origin. J. Am. Soc.
Nephrol. 2001;12:2125-2130.
7. Official Publication of the Indian Society of Nephrology. Indian J. Nephrol. 2005 July-
Sep.;15(3, Suppl. 1).
8. Official Publication of the Indian Society of Nephrology. Indian J. Nephrol. 2004 Oct.-
Dec. 2004;14(4).
9. Vijay V, Seena R, Lalitha S, Snehalatha C, Muthu J and Ramachandran A. Significance
of microalbuminuria at diagnosis of type 2 diabetes. Diabetes Bulletin. International J.
Diabet. Develop. Count. 1998;18:5-6.
10. Loon NR. Diabetic kidney disease: Preventing dialysis and transplantation. Clin. Diabet.
2003;21:55-62.

125
Diabetic Nephropathy in India

Flow Chart. A Suggested Scheme for screening for MAU in Diabetes

Postpuberty
More than 1 year diabetes
In stable glucose control
Free of other acute intercurrent illness

Dipstick test for proteinuria

Negative Positive

Laboratory
Test for MAU Confirmatory tests
Albumin concentration
Albumin; creatinine ratio

Negative Positive

Negative Positive

Rescreen annually
Laboratory
Confirmatory tests

Negative Positive

Manage appropriately
Rescreen annually

Source: Official Publication of the Indian Society of Nephrology.


Indian J. Nephrol. 2004 Oct.-Dec. 2004;14(4).

126
A Look at the Feet Vijay Viswanathan

Introduction
Diabetic foot problems cause more amputations than any other pathology.
However amputations are not inevitable. Early recognition of the at-risk foot,
prompt institution of preventive measures and the provision of rapid and
intensive treatment of foot complications can reduce the number of amputations
in diabetic patients1.
The aim of this chapter is to enable practitioners to take control of diabetic
foot problems. It attempts to give enough simple practical information which
would enable them to improve the outcome of a diabetic foot complication.

Importance of regular examination of feet


Once the presence of diabetes is recognized, a proper foot examination is
essential.
The goal of a foot examination should be to identify:
 A normal feet
 A feet at risk
 Presence of neuropathy
 Presence of ischemia
 Presence of foot lesions.
The procedure for screening should be inexpensive, quantifiable and
reproducible.
The examiner should insist on removal of the shoes and socks or stockings
even if the patient is reluctant, as it is not uncommon to see an active ulcer or
even ganrenous changes, of which the patient is totally ignorant. One should
look for neuropathic changes like dry skin, fissures, deformities, callus and
abnormal shape of foot, ulceration, prominent veins and nail lesions. Careful
attention should be given to the interdigital spaces. A significant ischemia is
characterized by loss of hair on the dorsum of foot and a dependent rubor.
Vijay, et al. found that in 1,010 patients, foot problems such as dry skin, feel
fissures, etc. were more common among patients with lower educational status
and also among those who wore their footwear for shorter periods if time2.
Hence a detailed foot examination is required for such patients.

127
A Look at the Feet

We should feel the feet of the patient to find out whether the foot is warm
or cold, examine the peripheral pulsations like dorsalis pedis, which can be felt
lateral to the extensor hallucis longus tendon, and the posterior tibial, which
is above and behind the medial malleolus. The femoral artery should also be
palpated and ausclated for the presence of bruit. The plantar aspects of the feet
should be felt for presence of any bony prominences.
The footwear should not be ill fitting or tight. It should have a broad toe
box, a heel height of 5 cm or less and heel counter should be stiff enough to
prevent excessive movement of the foot within the shoe. Those patients who do
not like to wear shoes should be advised to wear sandals with Velcro uppers
and a heel counter. Chappals and “Hawaii” slippers with a grip between the
great toe and the second toe should be discouraged. One should also inspect
the foot wear for signs of wear especially on the outsole. The footwear should
not have any protruding objects inside, the uppers should have a soft lining
and the insole should be soft.

Deformity
It is important to recognize deformity in the foot. Deformity often leads
to bony prominences, which are associated with high mechanical pressures
on the overlying skin. This results in ulceration, particularly in the absence of
protective pain sensation and when shoes are unsuitable. Ideally, the deformity
should be recognized early and accommodated in properly fitting footwear
before ulceration occurs.

Common deformities include


 Clawed toes: Clawed toes have fixed flexion deformities at the interphalangeal
joints and are associated with callus and ulceration of the apices and dorsal
aspects of the interphalangeal joints. Clawed toes may result from acute
rupture of the plantar fascia.
 Pes cavus: Normally the dorsum of the foot is domed due to the medial
longitudinal arch which extends between the first metatarsal head and the
calcaneus. When it is abnormally high, the deformity is called pes cavus.
 Hallux rigidus: This is limited joint mobility of the first metatarso-
phalangeal joint with loss of dorsiflexion and results in excessive pressure
on the plantar surface of the first toe, causing callus formation.
 Hallux valgus: Hallus valgus is a deformity of the first metatarso-phalangeal
joint with lateral deviation of the hallux and a medical prominence on the

128
Management of Type 2 Diabetes

margin of the foot and this site is particularly vulnerable in the neuroischemic
foot and frequently breaks down under pressure from a tight shoe.
 Hammer toe: Hammer toe is a flexion deformity of the proximal
interphalangeal joint of a lesser toe with hyperextension of the associated
metatarso-phalangeal and distal interphalangeal joints. The toe is at risk of
dorsal ulceration.
 Charcot foot: Bone and joint damage in the metatarsal-tarsal region leads
to two classical deformities: The rocker-bottom deformity in which there
is displacement and subluxation of the tarsus downwards, and the medial
convexity, which results from displacement of the talonavicular joint or
from tarso-metatarsal dislocation. Both are often associated with a bony
prominence. If these deformities are not accommodated in properly fitting
footwear, ulceration at vulnerable pressure joints often develops.

Complications of the diabetic foot and their diagnosis


Neuropathy
Upto 35% of all diabetic patients have symptomatic neuropathy and in
these patients neuropathy will only be detected by clinical examination.
A typical scenario is a patient who fails to complain of pain, even when
significant foot lesions are present. Painless ulceration is definite evidence of a
peripheral neuropathy.
The presentation peripheral neuropathy is related to dysfunction of sensory,
motor and autonomic nerves. Simple inspection will usually reveal signs of
motor and autonomic neuropathy but sensory neuropathy must be detected by
a sensory screening text or a simple sensory examination.
It is advisable to test dorsiflexion of the foot to detect a foot drop secondary
to a common peroneal nerve palsy which is usually unilateral and will affect
the patient’s gait.
The classic sings of an autonomic neuropath are:
 A dry skin with fissuring
 Distended veins over the dorsum of the foot and ankle.
The dry skin in secondary to decreased sweating. The sweating loss normally
occurs in a stocking distribution, which can extend upto the knee, and above which
there may be excessive sweating. Patchy sweating loss sometimes occurs.
The distended veins are secondary to arteriovenous shunting associated
with autonomic neuropathy.

129
A Look at the Feet

Sensory neuropathy can be simply detected by:


 Monofilaments
 Biothesiometry.
If these are not available, then a simple clinical examination detecting sensation
to light touch using a cotton wisp and vibration using a 128-Hz tuning fork will
suffice, comparing a proximal site with a distal site to confirm a symmetrical
stocking – like distribution of the neuropathy.
The advantage of the assessment with monofilaments or biothesiometry
is the detection of whether the patient has lost protective pain sensation that
would render him susceptible to foot ulceration.
(a) Monofilament testing:
Nylon monofilament of 5.07 size equivalent of 10 g of linear force should be
used. It tests sense of touch. When applied perpendicular to the foot, it buckles
at a given force of 10 g. The patient should be able to sense the monofilament by
the time it buckles. The filament should be pressed at several sites e.g., plantar
aspects of the first toe, the first, third, and the fifth metatarsal heads, the heel,
and dorsum of the foot. The patient’s inability to feel the filament indicates a
loss of protective sensation.
(b) Biothesiometry:
The degree of neuropathy can be further quantified by the use of the
biothesiometer. When applied to the foot, it delivers a vibratory stimulus, which
increases as the voltage is raised. The biothesiometer is widely used for the
quantitative assessment of the vibration perception threshold (VPT). The VPT
is assessed at the pulp of the great toe. The patient can be considered to have
a high risk foot with VPT >25 volts.

Ischemia
Ischemia results from atherosclerosis of the arteries of the leg. In the ischemic
foot the skin is thin, shiny and without hair.
(a) Ankle/brachial pressure index (ABI):
A hand-held Doppler can be used to confirm the presence of peripheral
pulses and to quantify the vascular supply. When used together with a
sphymomanometer, the ankle and brachial systolic pressures can be measured
and the ratio then calculated. In normal subjects, the ankle systolic pressure is
higher than brachial systolic pressure. The normal ABI is >0.9. Absent or feeble
ABI >0.9 indicates ischemia. Conversely, the presence of pulses and ABI >0.9
rules out significant ischemia.

130
Management of Type 2 Diabetes

(b) Plantar pressure measurement:


The feet can be evaluated for high pressure points by a simple inexpensive
technique using an ink mat. The patient is asked to stand or walk on the mat.
Ink is applied on the other side of the mat, with a roller. While interpreting
the footprint, darker area indicates a high pressure point. This test is however
not quantitative and has its limitations. A quantitative measurement of plantar
pressure is now available for a barefoot, as well as in-shoe, using a computer
and special software. However being expensive, it cannot be made available at
every center. Once the patient has been classified into either non-risk or high
risk category, a further evaluation depending upon the risk category can be
carried out3.

Warning signs and when to refer


The natural history of the diabetic foot can be divided into six stages as
shown in Table.
Table. Staging the diabetic foot
Stage Clinical condition
1 Normal
2 High risk
3 Ulcerated
4 Cellulitic
5 Necrotic
6 Major amputation

 Stage 1: The foot is not at risk. The patient does not have the risk factors
of neuropathy, ischemia, deformity, callus and swelling.
 Stage 2: The patient has developed one or more of the risk factors for
ulceration of the foot.
 Stage 3: The foot has a skin breakdown.
 Stage 4: The ulcer has developed into infection.
 Stage 5: Necrosis has supervened. In the neuropathic foot, infection is
usually the cause.
 Stage 6: The foot cannot be saved and will need a major amputation.

Importance of team work and coordinated foot clinic


for diabetes
Aggressive follow-up and thorough evaluation are all vital elements in reducing
the morbidity and mortality from diabetic lower extremity limb infections4.

131
A Look at the Feet

Palpable pulses Vascular intact Vascular absent Vascular absent


Sensation intact Sensation absent Sensation absent Sensation absent
No lesions Preulcerative
lesion
Yearly foot Check every Check every Check monthly
examination 3 months custom 3 months custom custom footwear
footwear risk footwear risk ongoing education
education education

Successful management of the diabetic foot needs the expertise of a


multidisciplinary team. No one person can take control of the diabetic foot.
Members of the team should include diabetologist, vascular surgeons, general
surgeons, podiatrist/chiropodist, nurse, foot care educator.
And orthotist. It is helpful if the team works closely together, within the
focus of the diabetic foot clinic, and also meets regularly for ward rounds. Each
team members should be available quickly in an emergency. All members of the
team must realize that absence of pain often delays the patient’s presentation
because he does not take the lesion seriously. We should remember that the
central figure in the team is the patient himself5.
Acknowledgement: Mrs. Seena Rajasekar, the diabetic foot educator for her
help in preparing this manuscript.

References
1. Edmonds ME and Foster AVM. Managing the diabetic foot. 2000:1-23.
2. Viswanathan V, Rajasekar S, Snehalatha C and Ramachandran A. Routine foot
examination: The first step towards prevention of diabetic foot amputation. Pract.
Diabetes Int. 2007;17:112-114.
3. Pendset S. Diabetic foot: A clinical Atlas. 2003.
4. Levin ME. Diabetes and peripheral neuropathy (Editorial). Diabetes Care 1998;21:1.
5. Levin and O’ Neal’s. The Diabetic Foot 6th Edition, Bowker JH and Pfeifer MA (Eds.)
2002.

Learning Box
1. Simple foot examination is an inexpensive and reproducible method to identify
high risk foot.
2. Identification of neuropathy using appropriate testing method is essential.
3. Doppler measurements can be used to determine ischemic changes.
4. Plantar pressure measurement is an added tool in the determination of high
risk foot.
5. A team approach is the best method towards treatment of diabetic foot
complications.

132
Type 2 Diabetes Mellitus: Ajay K Ajmani

More than Sugar Sachin K Jain

Type 2 diabetes mellitus (DM) is a common disorder. Its prevalence is


increasing worldwide largely due to increased obesity, sedentary lifestyle and
aging population. Its worldwide prevalence is expected to increase from 2.8% in
year 2000 to 4.4% in year 2020 (Wild et al). In India from 31.7 million in 2000
it is expected to increase to 79.4 million in 2030.
Type 2 DM is defined by presence of elevated glucose. However it is
associated with number of key metabolic and clinical derangements (metabolic
syndrome or syndrome X) and insulin resistance is the key to its pathophysiology.
Morbidity and mortality in a diabetic subject is largely due to development of
microvascular and macrovascular complications.
Mortality in DM is mostly due to cardiovascular events. Western data shows
that 80% of type 2 DM develop and die of macrovascular disease (Narayan, et
al) Indian studies show cardiovascular causes are responsible for 47.5% deaths
in type 2 DM subjects.
Control of hyperglycemia is closely related to complications and deaths.
Increase in HbA1C of 1% is associated with 28% increase in deaths in a prospective
study (Khaw, et al. 2001). Strict control of hyperglycemia reduces microvascular
end-points but does not significantly impact macrovascular complications. In
UKPDS 0.9% reduction in HbA1C was associated significant 25% reduction in
microvascular end-points but reduction in myocardial infarction (MI) at 16% was
statistically not significant. The lack of clear cut benefit with glucose control for
cardiovascular events calls for attention to other aspects of diabetes control and
components of metabolic syndrome.
These may be:
 Still stricter control of hyperglycemia
 Attention to postprandial glucose control
 Attention to type of agents used for glucose control
 Control of blood pressure
 Lipid management
 Use of antiplatelet therapy.

133
Type 2 Diabetes Mellitus: More than Sugar

Strict control of hyperglycemia


One explanation for lack of macrovascular benefit in UKDS may be the fact
that the risk of coronary disease is increased in upper half of normal glycemic
range (Khaw, et al.). Men between 40-79 years of age with HbA1C between 5.0%
and 5.4% had 2.7-fold greater risk of dying from ischemic heart disease over
subsequent 4 years compared to men with HbA1C <5%. Since HbA1C of <5%
is unlikely to be achieved in diabetic subjects with current therapeutic agents,
reducing glycemic risk for macrovascular risk is less likely to be as effective as
in microvascular disease reduction.
Use of newer agents such as insulin analogs, meglitinides, exenatide, DPP IV
inhibitors may help to achieve better control but as yet is unproven. ACCORD
(Action to Control Cardiovascular Risk in Diabetes, results expected 2009) study
may answer some of these questions (Prisant et al).

Postprandial hyperglycemia
Number of studies have shown that postmeal blood glucose levels are
associated with greater risk of cardiovascular events and mortality. The diabetes
intervention study showed that postmeal blood glucose but not fasting plasma
glucose (FPG) was an independent risk factor for MI and coronary heart disease
(CHD) mortality. The Honolulu Heart Study showed that the risk of fatal and
total CHD increased significantly with post meal glucose levels (Curb, et al.).
The DECODE (Diabetes Epidemiology: Collaborative Analysis of Diagnostic
Criteria in Europe) study showed 2-hour postload plasma glucose concentration
were associated with increased morality risk independent of FPG whereas the
association of FPG levels with morality was dependent on 2-hour postload
glucose levels.
Besides glucose levels hyperinsulinemia especially postmeal hyperinsulinemia
has been found to be associated with CHD in Paris prospective study and
Helsinki Policemen study. The mechanism of complications related to postmeal
glucose excursions are rise in free radical formation, labile non-enzymatic
glycation, increase in coagulation factors fibrinopeptide A, factor VII, increase
in atherosclerotic factors like ICAM-1.

Attention to type of agents used for glucose control


In UKPDS there was no significant reduction in macrovascular end-points with
sulfonylurea or insulin. However in obese subjects metformin therapy resulted in
significantly fewer MIs. The metformin group experienced 36% reduction in all

134
Management of Type 2 Diabetes

cause mortality in comparison with sulfonylurea or insulin therapy. Glitazones;


another insulin sensitizers reduce insulin resistance, decrease free fatty acid,
decrease levels of proinflammatory cytokines e.g., TNF-α, interleukin-6, raise
adiponectin, decrease coagulation factors--, atherosclerotic factors (Haffner,
et al.). Glitazones improve endothelial functions and decrease carotid intima-
media thickness (Sidhu, et al.) and reduce restenosis (Takagi et al). PROACTIVE
study showed that pioglitazone can decrease secondary composite end-point of
all cause mortality, non-fatal MI and stroke. Glitazones however lead to fluid
retention and weight gain. PROACTIVE showed an increase in LVF.
Above data suggests use of insulin sensitizers can be better in lowering
cardiovascular outcome compared to insulin or insulin secretagogues.
Hyperinsulinemia is important feature among Indians (Snehlata, et al.). Insulin
sensitizers should always be part of therapy in type 2 DM in India unless
contraindicated or not tolerated.

Control of blood pressure


Hypertension is highly prevalent in subjects with type 2 DM. More than
75% type 2 DM have blood pressure ≥130/80 mmHg or are on antihypertensive
therapy. Hypertension was present in 27% of DM subjects in Diabcare Asia
Study.
Hypertension in subjects with type 2 DM increase the risk of renal failure
as well increases the cardiovascular mortality. In SHEP (Systolic Hypertension
in Elderly Program) and Syst-Eur (Systolic Hypertension in Europe) the diabetic
subject showed a cardiovascular disease (CVD) reduction of 34% and 69%,
respectively with treatment of hypertension. In UKPDS tight control of blood
pressure showed with 21% greater risk reduction in MI and 44% greater reduction
in fatal and non-fatal strokes. Tight blood pressure control was associated with 34%
reduction in macrovascular end-points and 25% greater reduction in microvascular
end-points. A 10 mmHg reduction in systolic blood pressure decreased diabetes-
related end-points by 12% and diabetes-related deaths by 15%.
In HOT (Hypertension Optimal Treatment) study, subgroup of DM subjects
showed a reduction of major cardiovascular events from 24.4/1,000 to 11/1,000
with strategy to reduce diastolic blood pressure (DBP) ≤80 mmHg compared
to group with target DBP ≤90 mmHg. Based on this study ADA and other
associations have recommended a target blood pressure of ≤130/80 mmHg.
Number of studies has shown that using angiotensin converting enzyme
(ACE) inhibitors or angiotensin receptor blockers (ARBs) achieve better CVD

135
Type 2 Diabetes Mellitus: More than Sugar

and renal risk reduction compared to other antihypertensive agents. These agents
block the production of angiotensin II a potent vasoconstrictor also involved in
vascular remodeling. Reduction of CVD events may be accomplished independent
of blood pressure reduction. In HOPE study there were 3,577 diabetic subjects.
The rate of combined primary outcome of MI, stroke or cardiovascular death was
significantly lower in the ramipril group than in placebo group, risk reduction
being 25%. This protective effect persisted after correction of difference in
blood pressure (SBP 2.2 mmHg and DBP 1.4 mmHg lower in ramipril group).
Combined microvascular end-points risk was reduced by 16%. In LIFE (Losartan
Intervention for Endpoint Reduction) study, there was 24% greater risk reduction
in losartan group compared to atenelol group as regards primary composite
end-point of cardiovascular death, all MI and all strokes. Several trials have
shown a beneficial effect of ARBs on renal end-points.
Most of DM subjects require two or more hypertensive agents. In DM subjects
ACE inhibitors or ARB with or without a diuretic are initial agents of choice.
Later calcium channel blockers and b-blockers can be added as required.

Lipid management
Lipid abnormalities are common in type 2 DM subjects. In Diabcare Asia
Study 49% had high-density lipoprotein (HDL) <42 mg/dl; 54% had total
cholesterol >201 mg/dl and 51% had triglyceride >150 mg/dl.
An elevated level of low-density lipoprotein (LDL) was the most powerful
predictor of CHD in UKPDS. An increment of 1 mmol/l was associated with
1.57 times increased risk of CVD in a wide range of LDL cholesterol levels. In
MRFIT a decrease of 1 mmol/l of LDL was associated with 50% lower risk of
CHD regardless of baseline LDL levels. The characteristic abnormality of lipids in
DM subjects is elevated triglycerides, low HDL and LDL levels comparable with
non-diabetic individuals. Increased flux of free fatty acids leads to increased very
LDL and abnormal transfer of cholesterol and triglycerides between LDL and
very LDL results in LL which is small and dense making it more atherogenic
(Krauss et al).
HMG-COA reductase inhibitors (statins) are the drugs that have most
convincing data on reduction in LDL cholesterol and cardiovascular protection.
In 4S (Scandinavian Simvastatin Survival Study), in the diabetic subgroup (222
subjects out of total 4,444 subjects with history of CHD) LDL was lowered to
3.03 mmol/l from 4.81 mmol/l. The risk reduction in CVD events was 37%,
similar to non-diabetic individuals. The HPS (Heart Protection Study) provided

136
Management of Type 2 Diabetes

conclusive data that cholesterol reduction leads to substantial lowering of CVD


rates in diabetic populations even in primary prevention setting and at relatively
low LDL levels. Mean LDL cholesterol in this study was 3.2 mmol/l. Simvastatin
reduced LDL cholesterol by 1 mmol/l and decreased CVD risk by 24%. Even
in DM subjects with pretreatment cholesterol <3 mmol/l, there was marginally
significant reduction in first major vascular event. The CARDS (Collaborative
Atorvastatin Diabetes Study) showed that 10 mg of atorvastatin/day reduced
CVD event rate by 37%. The statin reduced the baseline LDL cholesterol of
3.04 mmol/l by 40%. Above studies have led to recommendation that threshold
for initiation of LDL lowering therapy is 2.6 mmol/l (100 mg/dl) and for very
high risk category the target is 1.8 mmol/l (70 mg/dl) or less. In order to achieve
such strict control ezetimibe (cholesterol absorption inhibitor) may be added
which leads to incremental 15-20% reduction in LDL (Feldman et al).
Fibric acid derivatives specifically target very LDL and low HDL cholesterol.
Gemfibrozil has proved to be effective in primary and secondary CVD prevention
in Helsinki heart study and the VA-HIT (Veteran affairs HDL intervention study),
respectively. In VA-HIT study gemfibrozil 1,200 mg/day resulted in 65 increase
in HDL and 205 reduced triglyceride with a 245 decrease in all MIs. The DAIS
(Diabetes Atherosclerosis Intervention Study) 200 mg of micronized fenofibrate
in 418 type 2 DM subjects decreased triglycerides by 28% and was associated
with reduced rate of progression of angiographically confirmed coronary artery
lesions. The trial was not powered to assess clinical parameters.
Niacin is the best therapy for normalizing all lipid abnormalities. It improves
HDL, triglycerides, postprandial lipemia, small dense LDL, fibrinogen and
plasminogen activator inhibitor-1 making it an attractive option for treatment of
dyslipidemia in type 2 DM subjects. It tends to increase blood glucose but that
can be managed by adjustment of therapy. It causes flushing due to vasodilatation
and is best given after aspirin.

Use of antiplatelet therapy


Cardiovascular events are precipitated by occlusion of the vessel by
disruption of atherosclerotic plaque, platelet activation and aggregation resulting
in intravascular thrombosis. Aspirin inhibits cyclooxygenase an essential enzyme
in production of thromboxane A2 in the platelets. Thromboxane is a powerful
stimulant of platelet aggregation and use of aspirin results in effective inactivation
of platelet.
A meta-analysis of 195 randomized trials of antiplatelet therapy demonstrates

137
Type 2 Diabetes Mellitus: More than Sugar

that aspirin reduces the risk of any cardiovascular event in patients with prior
MI, coronary bypass surgery, coronary angioplasty and stroke by 25%. Evidence
of effectiveness of aspirin in primary prevention is less abundant.
The adenosine diphosphate receptor antagonist clopidogrel is an alternative
in secondary prevention setting. Clopidogrel produced additional risk reduction
in DM subjects in subgroup analysis of CAPRIE (Clopidogrel versus aspirin in
patients at Risk of Ischaemic events) (Bhatt, et al.).
ADA advocates use of aspirin (75-162 mg/day) as primary prevention strategy
in every type 2 DM subject >40-year-old and an additional cardiovascular risk
which will include 99% of all patients with type 2 DM subjects.

Conclusions
Cardiovascular events are major cause of morbidity and mortality in type
2 DM subjects. Control of glucose leads to only modest benefits in preventing
macrovascular events. Attention to other components of metabolic syndrome
may improve the outcome. Lifestyle modification and pharmacotherapy both
are important in reaching the targets. Tight blood pressure control, use of lipid
lowering agents and antiplatelet therapy produce additional significant benefits
in preventing vascular complications in type 2 DM subjects. Very few of DM
subjects have glucose, blood pressure, lipids in the recommended target range
(Bhattacharya, et al.). Effect of multifactorial approach to reduce cardiovascular
disease in type 2 DM subjects have been clearly demonstrated in steno trial
(Gaede, et al.).

References
1. Wild S, Roglic G, Green A, et al. Global prevalence of diabetes: Estimates for the
year 2000 and projection for 2030. Diabetes Care 2004;27:1047-1053.
2. Narayan KM, Boyle JP, Thompson TJ, sorensen SW and Williamson DF. Lifetime risk
for diabetes mellitus in the United states. JAMA 2003;290:1884-1890.
3. Nigam A. Mortality in type 2 diabetes mellitus. J. Diabetes Assoc. India 1998;38:
59-62.
4. Khaw KT, Wareham N, Luben R, et al. Glycated haemoglobin, diabetes, and mortality
in men in Norfolk cohort of European prospective investigation of cancer and nutrition
(EPIC-Norfolk) BMJ 2001;322:15-18.
5. Intensive blood-glucose control with sulphonylurea or insulin compared
with conventional treatment and risk of complications in patients with type 2 DM
(UKPDS 33). Lancet 1998;352:835-853.
6. Intensive blood-glucose control with metformin on complications in overweight
patients with type 2 DM (UKPDS 34). Lancet 1998;352:854-856.
7. Prisant LM. Clinical trials and lipid guidelines for type 2 DM. J. Clin. Pharmacol. 2004;
93:27c-31c.
8. Curb JD, Rodriguez BL, Burchfiel CM, Abbot RD, et al. Sudden death, impaired glucose
tolerance and diabetes in Japanese American men. Circulation 1995,91:2591-2595.

138
Management of Type 2 Diabetes

9. Decode Study group, European Diabetes Epidemiology group. Glucose tolerance and
mortality; comparison of WHO and American Diabetes Association diagnostic criteria.
Lancet 1999;354:617-621.
10. Pyorola K, Savolainen E, Kaukola S, et al. Plasma insulin as coronary heart disease
risk factor; relationship to other risk factors and predictive value during 9 ½ year
follow up of the Helsinki Policemen study population. Acta Med. Scand. Suppl. 1985;701:
38-52.
11. Fontbonne AM and Eschwege EM. Insulin and cardiovascular disease; Paris
prospective study. Diabetes Care 1991;14:461-469.
12. National Institute of Diabetes and Kidney diseases. National diabetes statistics fact
sheet: General information and national estimates on diabetes in the United states.
Bethesda (MD); US Department of Health and Human Services, National Institute of
Health, 2004.
13. Haffner SM, Greenberg AS, Weston WM, et al. effect of rosiglitazone treatment on
nontraditional markers of cardiovascular disease in patients with type 2 diabetes
mellitus. Circulation 2002;106:679-684.
14. Sidhu JS, Kaposzta Z, Marcus HS et al. Effect of rosiglitazone on common carotid
intima-media thickness progression in coronary artery disease patients without
diabetes mellitus. Arterio. Thromb. Vasc. Boil. 2004;24:930-934.
15. Takagi T, Yamamuro A, Tamita K, et al. Pioglitazone reduces neointimal tissue
proliferation after coronary stent implantation in patients with type 2 diabetes mellitus:
An intravascular ultrasound scanning study. Am. Heart J. 2003;146:E5.
16. Dormand JA, Charbonnel B, ecland A, et al.; (PROACTIVE Investigators). Secondary
prevention of macrovascular events in patients with T2DM in the PROACTIVE study
(PROspective pioglitAzone Clinical Trial In macroVascular Events): A randomized
control trial. Lancet 2005;366:1279-1289.
17. Snehlatha C, Ramachandran A, Vijay V and Viswanathan M. differences in plasma
insulin responses in urban and rural Indians; a study in southern Indians. Diabet.
Med. 1994;11:445-448.
18. Klag MJ, Whelton PK, Randall BL, et al. Blood pressure and end stage renal disease
in Men. NEJM 1996;334:13-18.
19. Stamler J, Vaccarro O, Neaton JD, et al. diabetes, other risk factors, and 12-yr
cardiovascular mortality for men screened in the Multiple risk intervention trial.
Diabetes Care 1993;16(2);434-444.
20. Curb JD, Pressel SL, Cutler JA, et al. effect of diuretic based antihypertensive
treatment on cardiovascular disease risk in older diabetic patients with isolated systolic
hypertension. Systolic hypertension in elderly program cooperative Research Group.
JAMA 1996;276:1886-1892.
21. Tuomilhto J, Rastenyte D, Birkenhager WH, et al. effects of calcium channel blockade
in older patients with diabetes and systolic hypertension. Systolic hypertension in
Europe trial investigators. NEJM 1999;340:677-684.
22. Tight blood pressure control and risk of macrovascular and microvscular
complications in type 2 diabetes; UKPDS 38. BMJ 1998;317;703-713.
23. Afler AI, Stratton IM, Neil HA, et al. Association of systolic blood pressure and risk of
macrovascular and microvascular disease in type 2 diabetes; (UKPDS 36) prospective
observation study. BMJ 2000;321:412-419.
24. Hansson L, Zanchett A, carruthers SG, et al. effects of intensive blood pressure
lowering and low dose aspirin inpatients with hypertension; principal results of
Hypertension optimal treatment (HOT) randomized trial. HOT study group. Lancet
1998;351:1755-1756.
25. Effects of ramipril on cardiovascular and microvascular outcomes in people with
diabetes mellitus: Result of HOPE study and MICRO-HOPE substudy. Heart outcomes
prevetion evaluation study investigators. Lancet 2000;355:253-259.

139
Type 2 Diabetes Mellitus: More than Sugar

26. Lindholm LH, Ibson H, Dahlof B, et al. Cardiovascular morbidity and mortality in
patients with diabetes in the losartan Intervention endpoint reduction in hypertension
study (LIFE); a randomized trial against atenelol. Lancet 2002;359:1004-1010.
27. Lewis EJ, Hunsicker LG, Clarke WR, et al. renoprotective effect of the angiotensin
irbesartan in patients with nephropathy due to type 2 diabetes. NEJM 2001;345:
851-860.
28. Parving HH, lehnert H, Brochner-Mortensen J, et al. the effect of irbesartan on
the development of diabetic nephropathy in patients with type 2 diabetes. NEJM
2001;345;870-878.
29. Kapur A and Jorgenson LN. DiabCare Asia Study-comparative Status of Current
Diabetes Care in Asia. Novo Nordisk Update 2001 proceedins p3-13.
30. Turner RC, Millns H, Neil HA, et al. Risk factors for coronary artery disease in non
insulin dependent diabetes mellitus (UKPDS 23). BMJ 1998;316:823-828.
31. Krauss RM. Lipid and lipoproteins in patients with type 2 diabetes. Diabetes Care
2004;27:1496-1504.
32. Pyorala K, Pederson TR, Kjekshus J, et al. Cholesterol lowering with simvastatin
improves prognosis of diabetic patients with coronary heart disease: A subgroup
analysis of Scandinavian Simvastatin Survival Study (4S). Diabetes Care 1997;20:
614-620.
33. Collins R, Armitage J, Parish S, et al. MRC/BHF heart Protection Study of cholesterol
lowering with simvastatiin in 5963 people with diabetes: a randomized placebo
control trial. Lancet 2003;361:2005-2016.
34. Colhoun HM, Betteridge DJ, Durrington PN, et al. primary prevention of cardiovascular
disease with atorvastatin in T2DM in the Collaborative Atorvastatin Diabetes study
(CARDS): multicentre randomized placebo control trial. Lancet 2004;364:685-696.
35. Feldman T, Koren M, Nsull Jr W, et al. Treatment of high risk patients with
ezetimibe plus simvastatin co-administration versus simvastatin alone to attin
National Cholesterol Education Program Adult Treatment Panel III low density
lipoprotein cholesterol goals. Am. J. Cardiol. 2004;93:1481-1486.
36. Rubins HB, Robins SJ, Collins D, et al. Diabetes, plasma insulin and cardiovascular
disease: subgroup analysis from the Department of Veteran Affairs high density
lipoprotein intervention trial (VA-HIT). Arch. Intern. Med. 2002;162:2597-2604.
37. Effect of fenofibrate on progression of coronary artery disease in type 2 diabetes:
The Diabetes Atherosclerosis Intervention Study, a randomised study. Lancet 2001;
357:905-910.
38. Elam MB, Hunninghake DB, Davis BK, et al. Effect of niacin on lipid and lipoprotein
levels and glycemic control in patients with diabetes and peripheral arterial disease.
The ADMIT study: A randomised trial. JAMA 2000;284:1263-1270.
39. Antithrombotic Trialist’s Colaboration. Collaborative meta analysis of randomized
trials of antiplatelet therapy for prevention of death, myocardial imfarction, and stroke
in high risk patients. BMJ 2002;324:71-86.
40. Bhatt DL, Marso SP, Hirsch AT, et al. Amplified benefit of clopidogrel versus aspirin
in patients with diabetes mellitus. Am. J. Cardiol. 2002;90:625-628.
41. Bhattacharya A, Joshi R and Menaka R. Early referral to specialist center for diabetes
is warranted. Diabetic Medicine 2006;23(4):P489.
42. Gaede P, Vedel P, Larsen N, et al. Multifactorial intervention and cardiovascular
disease in patients with type 2 diabetes. NEJM 2003;348:383-393.

140
Type 2 Diabetes - Look
KK Aggarwal
at the Heart

Coronary artery disease in people with diabetes:


Indian scenario
Coronary artery disease (CAD) tends to occur earlier in life in Indians than
in other ethnic groups. It is believed to be more severe and follow a malignant
course1. Cross-sectional studies in India have endorsed a prevalence of CAD
which is several-fold higher than in developed countries2,3. Global Burden of
Disease Study estimates that by the year 2020, the burden of atherothrombotic
cardiovascular disease in India will surpass that in any other region in the
world4. The mortality attributable to cardiovascular disease in India is expected
to rise by 103% in men and by 90% in women from 1985 to 2015.5
This predilection to CAD is attributed to a clustering of various traditional
and non-traditional risk factors which are believed to constitute the atherogenic
phenotype characteristic of Indians. CAD among Asian Indians can be broadly
categorized into three distinct forms:
 Type I or malignant type occurs in young individuals (<50 years) with
marked prematurity and severity; this type is accompanied by the absence
or low levels of conventional risk factors and the presence of high levels
of emerging risk factors
 Type II occurs in older individuals (>65 years) with high levels of conventional
risk factors and low levels of emerging risk factors.
 Type III or mixed variety occurs between the ages of 50 and 65 and is
accompanied by varying combinations of conventional and emerging risk
factors.

Magnitude of problem
The CAD rates in urban India over the past 40 years have increased
dramatically and are now similar to that of overseas Indians and several times
higher than in other Asian countries. In the UK, which has one of the highest
mortality rates of CAD, the prevalence of symptomatic CAD in Asian Indians
is similar to Whites (8.5% vs 8.2%), but the asymptomatic or silent CAD is

141
Type 2 Diabetes - Look at the Heart

higher. In the US, the prevalence of CAD in Asian Indians is 4-fold higher than
Whites (10% vs 2.5%) as shown in Graph 1. The prevalence data underestimate
the incidence when case fatality rates are higher, as is the case in the UK.
Therefore, the burden of CAD in Asian Indians is much higher than that
reflected by the prevalence data.

Graph 2: CAD Prevalence in CADI* Study Compared


with US and India (men ages 30-69)10

CADI (n = 1,131) FHS† (n = 896)

2.5 10.2 10.6 9.8 10.0 10.4 10.4 9.8 9.6

*CADI: Coronary Artery Disease in Indians



FHS: Framingham Heart Study

CAD rates in rural India


Despite higher rates of smoking, CAD rates in rural India are about one-half
those in urban India (Table 1)6. A cross-sectional survey done in rural Haryana
in 1998 revealed a CAD prevalence rate of 6% in rural Indians aged 35-64 years7.
This CAD rate is 2-fold higher than contemporary US rates and 3-fold higher
than the 2.1% reported in 1974 from the same village8,9.
The prevalence of CAD in urban India is about double the rate in rural
India and about 4-fold higher than in the US. The rates appear to be higher in
south India with Kerala having a prevalence of 13% in urban areas and 7% in
rural areas (Table 1)10,11.
On the whole there has been a >3-fold increase from 3% prevalence 30 years
ago in urban India. Higher rates of CAD in urban India compared to rural India
suggest important roles for nutritional and environmental factors, or nurture.
There is a significantly higher body mass index (BMI) in urban India compared
to rural India (BMI, 24 vs 20 in men and 25 vs 20 in women). There is also a

142
Management of Type 2 Diabetes

Table 1
Age- Sample CAD
Author Year group Place size prevalence (%)
Urban population
Mathur KS 1960 30-70 Agra (Uttar 1,046 1.05
Pradesh)
Sarvotham SG 1968 30-70 Chandigarh 2,030 6.60
(Punjab)
Mohan V 2001 20-70 Chennai 1,262 11.0
Padmavathi S 1962 30-70 Delhi 1,642 1.04
Chadda SL 1990 25-65 Delhi 13,723 9.67
Reddy KS 1998 35-64 Delhi 2,800 10.5
Gupta R 1995 20-80 Jaipur (Rajasthan) 2,212 7.59
Singh RB 1995 20-70 Moradabad 152 8.55
Gupta SP 1975 30-70 Rohtak 1,407 3.63
Begon R 1995 20-70 Kerala 506 12.65
Rural population
Devan BD 1974 30-70 Haryana 1,506 2.06
Reddy KS 1998 35-64 Haryana 2,800 6.0
Kutty VR 1993 25-65 Kerala 1,130 7.43
Wander GS 1994 30-70 Punjab 1,100 3.09
Gupta R 1994 20-80 Rajasthan 3,148 3.53
Singh RB 1995 20-80 Uttar Pradesh 162 3.09
Jajoo UN 1988 30-70 Vidarbha 2,433 1.69

higher rate of abdominal obesity among the urban population, with urban men
having a waist to hip ratio (WHR) of 0.99 compared to 0.95 among rural men.
This increase in BMI and WHR results in significant dyslipidemia and insulin
resistance and a 3-fold increase in diabetes.

Cardinal features of CAD among Asian Indians


The cardinal features of CAD in Asian Indians around the globe are given
in Table 2. Most notable are the extreme prematurity and severity as discussed
below12.

Conventional risk factors


All conventional risk factors are significantly associated with the risk
of CAD in Asian Indians, as in all other populations. However, compared

143
Type 2 Diabetes - Look at the Heart

Table 2
Higher rates
 2- to 4-fold higher prevalence, incidence, hospitalization and mortality
Greater prematurity
 5-10 yeras earlier onset of first myocardial infarction (MI)
 5- to 10-fold higher rate of MI and death in those, <40 years of age
Greater severity
 Three vessel disease common in the young, even premenopausal women
 Larger MI with greater myocardial damage and complications
Higher prevalence of glucose intolerance
 Insulin resistance syndrome, diabetes and central obesity
Lower prevalence of conventional risk factors
 Hypertension, obesity, cigarette smoking, high cholesterol
 Cholesterol levels similar or lower than Whites but higher than other Asians
Higher rates of CAD at any given level of conventional risk factors
 >2-fold higher rates after adjusting for conventional risk factors
 Underscores the need for lower threshold for intervention
Higher prevalence of emerging risk factors
 High levels of lipoprotein(a), homocysteine, apo B, triglycerides, fibrinogen, plasminogen
activator inhibitor 1
 Low levels of HDL and HDL 2b
 Small dense LDL, small HDL, large very LDL
Higher rates of clinical CAD events for a given degree of atherosclerosis
 2-fold higher than Whites
 4-fold higher than Chinese
 Higher proportion of unstable or vulnerable plaques

with Whites, Asian Indians have a lower prevalence of hypertension,


hypercholesterolemia, obesity and smoking, but a higher prevalence of high
triglycerides, low high-density lipoprotein (HDL), glucose intolerance and
central obesity.
Although the conventional risk factors do not fully explain the excess burden
of CAD, these risk factors appear to be doubly important in Asian Indians, and
remain the principal targets for prevention and treatment. Graph 1.

144
Management of Type 2 Diabetes

Graph 1:

HTN: Hypertension, TC: Total cholesterol, TG: Triglycerides,


Hcy: Hypercholesterolemia, Lp(a): Lipoprotein(a)

Screening for CAD in diabetes


The American College of Cardiology/American Diabetes Association
recommends (Table 3) that cardiac testing be done irrespective of the presence
of CAD symptoms in diabetic patients with two or more atherogenic risk factors,
in view of the high prevalence of CAD. Various screening methods are available
including exercise electrocardiography, exercise echocardiography and exercise
single photon emission-tomography (SPECT).
No studies currently exist of the long-term costs and the effectiveness of CAD
screening strategies in asymptomatic patients with diabetes. Screening for CAD
with exercise echocardiography in asymptomatic diabetic men with hypertension
and smoking seems to be acceptable from a societal perspective13.

Table 3. Indications for cardiac testing in diabetic patients


 Typical or atypical cardiac symptoms
 Resting ECG suggestive of ischemia or infarction
 Peripheral or carotid occlusive arterial disease
 Sedentary lifestyle or plan to begin a vigorous exercise program
 Two or more of the risk factors listed below:
– Total cholesterol ≥240 mg/dl, LDL cholesterol ≥160 mg/dl or HDL cholesterol
<35 mg/dl
– Blood pressure >140/90 mmHg
– Smoking
– Family history of premature CAD
– Positive micro/macroalbuminuria

145
Type 2 Diabetes - Look at the Heart

The bottom line is that CAD is the major cause of mortality in diabetic
patients, so its early detection is important to improve medical intervention and
outcome. Screening is particularly needed for those patients who, apart from
diabetes, show additional atherogenic risk factors.
These high risk diabetic patients should also be screened for CAD with a
provocative test when no symptoms and signs are present. In fact, irrespective
of the presence or absence of angina, ST segment depression during exercise is
a predictor of an increased risk of death.
In addition, among patients with silent myocardial ischemia, the mortality
rate is higher in diabetic than in non-diabetic individuals. Finally, among diabetic
patients, those unable to perform the exercise are at higher risk of mortality. The
exercise stress test is the most widely utilized low-cost, non-invasive screening
test for silent myocardial ischemia.

References
1. Enas EA, Yusuf S and Mehta JL. Prevalence of coronary artery disease in Asian
Indians. Am. J. Cardiol. 1992;70:945–949.
2. Mohan V, Deepa R, Rani SS and Premalatha G. Prevalence of coronary artery disease
and its relationship to lipids in a selected population in South India: The Chennai
Urban Population Study (CUPS No.5). J. Am. Coll. Cardiol. 2001;38:682–687.
3. Reddy KS, Shah P, Shrivastava U, Prabhakaran D, Joshi M, Puri SK, et al. Coronary
heart disease risk factors in an industrial population of north India. Can. J. Cardiol.
1997;13(Suppl. B):3.
4. Murray CJL and Lopez AD. Alternative projection of mortality and morbidity by
cause 1990–2020; Global Burden of Disease Study. Lancet 1997;349:1498–1504.
5. Bulatao RA and Stephens PW. Global estimates and projections of mortality by
cause. Population, Health and Nutrition Department; World Bank, preworking paper.
Washington, DC 1007;1992.
6. Gupta R. Coronary heart disease epidemiology in India: The past, present and future.
In: Coronary Artery Disease in South Asians Rao GHR (Ed.), JAYPEE, New Delhi 2001:
6-28.
7. Reddy KS. Rising burden of cardiovascular diseases in India. In: Coronary Artery
Disease in Indians: A Global Perspective Sethi KK (Ed.), Cardiology Society of India,
Mumbai 1998:63-72.
8. Dewan BD, Malhotra K and Gupta S. Epidemiological study of coronary heart disease
in a rural community in Haryana. Indian Heart J. 1974;26:68-78.
9. Enas EA. Coronary artery epidemic in Indians: A cause for alarm and call for action.
JIMA 2000;98:694-702.
10. Begom R and Singh R. Prevalence of coronary artery disease and risk factors in urban
population of south and north India. Acta Cardiologica 1995;3:227-240.
11. Raman Kutty V, Balakrishnan K, Jayasree A, et al. Prevalence of coronary heart
disease in the rural population of Thiruvananthapuram district, Kerala, India. Int. J.
Cardiol. 1993;39:59-70.
12. Enas EA. Arresting and reversing the epidemic of CAD among Indians. In: Current
Perspectives in Cardiology Kumar A (Ed.), Cardiological Society of India, Chennai
2000:109-128.
13. Consensus development conference on the diagnosis of coronary heart disease in
people with diabetes: 10-11 February 1998, Miami, Florida. American Diabetes
Association. Diabetes Care 1998;21:1551-1559.

146
Are we Communicating to
our Patient’s Well? Some Issues
Sri Nagesh Simha

A familiar story? A young man, a juvenile diabetic, decides to tie the knot,
but fearing that his young bride would reject him if she knew his medical
condition, keeps her in the dark. The marriage is over and off they go for their
honeymoon. No insulin is possible and the young man lands into complications.
He promptly gets himself into the nursing home of the diabetologist who has
been treating him, pleading that the doctor announces that diabetes was just
detected! What does the doctor do? What is happening?
Any illness that threatens the integrity of a person and is either life-threatening
or chronic, evokes a complex response. This is controlled by many factors; the
attitude and knowledge of the patient, the dynamics of his surroundings and
the skill in communication of his treating team. As healthcare professionals,
more so in the case of doctors, we would like to bask in the mistaken notion
that a medical degree has given us the opportunity to drink deep from the sea
of knowledge in communication, making us experts!
The following quote on communication sums up the matter admirably:
Almost invariably, the act of communication is an important part of therapy;
occasionally it is the only constituent. It usually requires greater thought and
planning than a drug prescription, and unfortunately it is commonly administered
in sub therapeutic doses!
Society has evolved many ways to communicate; my favorite one is what
illiterate persons would make use of. The very arrival of a letter would portend
bad news. If the letter did not contain that, the writer always wrote on the corner
of the letter, the equivalent of “all is well”, something that illiterates learnt to
recognize! Alas, medical communication is not that simple!
Is communication necessary? What are the difficulties in communication? Is
there a “right” and a “wrong” way?
There is a school of thought that “ignorance is bliss” Knowledge of illness causes
distress and prevents or reduces hope. Nothing can be farther than the truth.
The need for communication is not always understood. The sacred relationship
of trust between two people, particularly a doctor and patient, is shattered when
information is withheld. This leads to uncertainty, something that the human

147
Are we Communicating to our Patient’s Well? Some Issues

mind is not comfortable with! And the patient needs the time and space to
make necessary changes in his life, sort out unfinished business if the illness
is life-threatening.
Communication is a dynamic process, requiring change when needed. Studies
have shown that focused questioning with a stress on understanding the feelings
of patients elicits much greater information and is much more therapeutic. Avoid
direct questions; rather, use more broad open ended or directive questions.
There are a number of predictive behaviors while communicating with
patients which are demonstrated by health professionals which sadly lead to
distancing and blocking rather than eliciting concerns. They include:
 Ignoring cues. Here, the patient is saying one thing but there is another
hidden meaning or a buried concern which needs to be explored. If this is
missed, it leaves the patient distressed
 Inappropriate encouragement
 Premature reassurance
 Switching the topic
 Passing the buck
 Avoiding the patient
 Minimizing fears
 Using jargon.
Amongst the many issues in communication, the one that causes most
distress is breaking bad news. Communicating distressing news is difficult for the
healthcare professional. Many just do not know how to do it as they have not
received any formal training. They are unsure of handling the patient’s reaction.
Many feel that they will be blamed for giving the bad news and finally, as
human beings, we instinctively want to shield others from distressing news!
Patients too find it difficult to talk to doctors about unpleasant things.
They feel that the doctor is too busy and is only interested in his physical well
being. They are afraid of breaking down in front of the doctor. And, they are
terrified about the truth being revealed to them. The less you know the more
hope you can have!
Prof. Peter Maguire and his team at the Psychological Medicine Group of
the Cancer Research Campaign UK have done seminal work on the technique
of breaking bad news.
The “steps” are simple and if followed, leave the patient less distressed.
These are:

148
Management of Type 2 Diabetes

1) Preparation:
Prior to interviewing a patient, ensure you have all the information. Select a
comfortable room with privacy. You must not be disturbed in between the interview.
Check with the patient if he or she wants anyone else to be present.
2) Does the patient want to know?
This may sound simplistic, but not all patients are ready to hear bad news
when the health professional so chooses to do so. Information must be given
at the pace that the patient wants. The patient has a right to know, but not a duty
to know!
3) What does the patient know?
Many times, the patient would have suspected that all is not well. It is worth
probing this by starting the interview with a question like You have been unwell
for sometime. Could you tell me how all this started? This gives an opportunity to the
patient to express his feelings and may give a clue to the health professional that
there is awareness of a possible serious diagnosis. Patients may say something
like I am afraid that there is something serious. This gives a window to introduce
the diagnosis to the patient.
4) Give a warning shot
On determining the level of awareness, the “bad new” must be broken. This
may be done by initially using words like I am afraid the ulcer that you have is
not a simple ulcer or The lump in your breast is more serious than we though it. This
will usually evoke a statement like Don’t tell me that I have cancer…. To which,
one can say I am afraid so. You have cancer..
5) Allow denial
Denial is a common response to bad news. It is a coping mechanism commonly
used and the patient must be given the space and permission to do so.
6) Explain and give more information if asked for
There is always a tendency for health professionals to load information
onto a patient when he is not ready for it. The professional agenda must come
after the patient’s agenda. Checking with the patient is necessary, and the rate
of information sharing can be decided.
7) Listen to the patient’s concerns.
This will give an excellent idea of the issues that he is grappling with.
8) Encourage ventilation of feelings
By a question such as Now that I have told you that you have diabetes, how
do you feel? This may sound strange and insensitive but surprisingly makes the
patient share his concerns. This will lead to an outpouring of feelings which

149
Are we Communicating to our Patient’s Well? Some Issues

must be made note of and the issues handled in the order of the priority set
by the patient. It is helpful to remember the 11th Commandment; Thou shalt
not ASSUME, else thou shalt make an ASS of U and ME!
9) Summarize and come to a plan of action.
It is always helpful to summarize the interview, checking for any information
gaps and to devise a plan to move forward.
10) Offer availability
This is the time for some reassurance, promising the patient all the support
in coping with the illness.

Denial
Reactions to news about life-threatening or chronic illness vary from person-
to-person. They include disbelief, revolt (accusation), depression (sadness for
health lost), confronting reality and consenting (coping) with serenity. These
are integrating processes. The process of integration is also dependent on how
the patient views his or her chronic disease. Hence patient education plays a
very important role.
Patients could also engage in a distancing process which includes anguish,
denial of emotions, passive resignation and melancholia.
However, a common coping mechanism is denial. It attempts to simplify the
complexities of life, revising or reinterpreting a painful reality. It causes distress
to the health professional and it is important to understand the nuances of it
to be able to handle it. Denial needs to be addressed only if it leads to delay
in taking treatment or pushing the patient to do something disastrous. On the
contrary, if it leads to better coping, reduces anxiety and promotes optimal
functioning, the patient must be given the freedom to be in denial.
Denial is handled by challenging it at a comfortable pace, without causing
psychological harm. Stripping denial aggressively may intensify it or cause
overwhelming distress leading to an anxiety state or depressive illness.
A young lady had a blood sugar test which revealed that she was suffering
from diabetes mellitus. She just could not come to terms with this and went
through a myriad of emotions, finally leading to denial. She refused treatment,
saying that she just did not have diabetes. This denial needs to be addressed
sensitively. A suitable approach would be to acknowledge the denial and to
seek reasons for it. She could be asked It is clear from your blood tests that your
sugars are high in three successive estimations and that you have diabetes. Yet, you do
not agree. Could you tell me why you feel so? This will evoke a response, which

150
Management of Type 2 Diabetes

could be addressed suitably. Should the denial continue with the patient saying
that she just does not believe it, you could proceed by gently placing before her
the consequences of her denial. If you continue to neglect this eminently treatable
condition, you are likely to suffer from the following complications which could endanger
your life. On gently persisting, issues will come out which can be handled and
the denial broken successfully.
Falsely optimistic views about an illness, given insensitively by the treating
team, leads to gross misinformation and great distress too. This may pose some
difficulty in handling it.
Let us come back to the story we started with. The doctor was being asked
to join hands with the patient in withholding important information from the
patient’s wife. This is known as collusion. There are three players in this dynamic
dialogue. The patient, his wife and the doctor.
The patient does not inform his wife because he is worried about her
rejection. He cares for her and wants her at all costs. Sadly, he has not realized
the emotional cost of this collusion. The doctor, caught in a bind, has to tread
a narrow path and be fair to both of them. How does he proceed?
Let us understand collusion first.
Collusion is sharing information with certain important people and withholding
it from certain important person/persons. This is an act of love, done with the
sole intent of “protecting” the person from the distress of the bad news to be
broken.
Healthcare professionals find it extremely difficult to handle collusion. They
usually take the easy way out and consent to the collusion! The emotional cost of
collusion is high. It leads to distrust, poor communication, inability to complete
unfinished business in life, inability in making necessary changes in one’s lifestyle
in a condition like diabetes and increases the fear of the unknown.
The health professional needs to probe the reasons for the collusion,
acknowledge them and gently explain the emotional cost of the collusion,
keeping a comfortable non-aggressive pace. It happens many times that the
person from whom we are trying to keep information away, may already know
or be suspicious of the diagnosis. Hence, with the relative’s consent, the health
professional checks the status of knowledge of the patient and then with the
permission of the relative, breaks the collusion.
The doctor in our story should make all efforts to handle this very sensitive
issue. If the wife comes to finally know the truth, she will be devastated. It
may have a significantly negative impact on their marital life. The doctor should
explore the issues with the husband and counsel him accordingly.

151
Are we Communicating to our Patient’s Well? Some Issues

Every patient is unique and one has to tailor one’s method of communicating.
In a chronic disease like diabetes, the relationship between the doctor and
patient can be a long one. Patients live for years with their diabetes. It calls
for a dynamic relationship, with the patient having to make many changes in
his life. His compliance of the treatment and his willingness to cooperate will
depend on the approach taken by the treating team.
His level of awareness may be low or high, and in these two groups,
attitudes could vary from I know it all, casual, myopic to very knowledgeable and
even totally terrified!
The patient should be convinced that he is ill. He must believe there could
be serious consequences because of this illness. He must believe treatment will
be beneficial. These can be discovered only by “semi directive interviews” which
convince the patient that interest being shown in him is not merely biological.
This shifts the locus of control to the patient, leads to a positive mind set
and helps one cope with the illness. There is some evidence that patients who
are offered a choice of treatment, when technically possible, cope better than
those not given an option, provided they are the kind who want to be involved
in this choice.
Every attempt must be made to make the patient a willing partner in his
own care, listening to his concerns, educating him and encouraging him to seek
more information. And one must ensure our communication contains the triad
of genuineness, non-possessive love and empathy.

Suggested readings
1. Faulkner A and Maguire P. Talking to cancer patients and their relatives. Oxford
Medical Publications 1994.
2. Oxford Textbook of Palliative Medicine.
3. Psychological Medicine Group, Cancer Research Campaign, Manchester UK. Notes
for Training of Trainers Workshop 1998.
4. Morris J and Royle GT. Offering patients a choice for surgery for early breast cancer:
A reduction in anxiety and depression in patients and husbands. Social Science and
Medicine 1988;26:583-585.
5. Institute of Palliative Medicine, Medical College, Calicut, Kerala India. Workbook of
International Workshop on community participation in palliative care. 2004.

152
Slide Quiz Arpandev Bhattacharyya
Menaka Ramprasad

Slide 1

Q 1. What are the changes you see?



Q 2. What are the various stages of diabetic retinopathy?

Q 3. When to refer to an ophthalmologist?

153
Slide Quitz

Slide 2

Q 1. What do the slides depict?



Q 2. What is the screening test for the given problem?

Q 3. Which drug is beneficial in slowing the progression of the disease?

³
Slide 3

Q 1. What does this indicate?



Q 2. What are the syndromes associated with it?

³
154
Management of Type 2 Diabetes

Slide 4

Q 1. What does this show?



Q 2. What is it associated with?

Slide 5

Q 1. What are the CT findings shown here?

155
Slide Quitz

Slide 6

Q 1. What is the sign known as?

Q 2. What does it indicate?

³
Slide 7

Q 1. What does the slide show?



Q 2. What is the most common problem seen in this condition?

Q 3. Why does this happen?

³
156
Slide Quitz Management of Type 2 Diabetes

Slide 8

Q 1. What is the condition known as?



Q 2. How does it happen?

Slide 9

Q 1. What is the foot problem?

Q 2. What are the various risk factors for developing the above problem in a
person with diabetes?

157
Slide Quitz

Slide 10

Q 1. Which infection does this X-ray finding in a patient with


diabetes depict?

Q 2. How best would you treat Diabetes in a such a patient?

158
Slide Quiz Answers
Slide 1
Ans 1. a. Blot hemorrhages
b. Cotton wool spots
c. Circinate exudates
d. Vitreous hemorrhage.

Ans 2. Diabetic retinopathy is broadly classified as:


a. Non-proliferative retinopathy
b. Proliferative retinopathy.

Ans 3. Urgent referral is indicated:


a. New vessels on the disc/elsewhere
b. Preretinal and/or vitreous hemorrhage
c. Retinal detachment.
Early referral is indicated:
a. ”High risk”, preproliferative retinopathy
b. Non-proliferative retinopathy with macular involvement
c. Hemorrhages and/or hard exudates within one disc diameter from
the center of the fovea
d. Reduced visual acuity not corrected pinhole, suggestive of macular
edema.
Routine referral is indicated:
a. Non-proliferative retinopathy with large circinate or plaque exude
within the major temporal arcade but not threatening the macula
b. Retinal findings that are not characteristic of diabetic retinopathy
c. Background retinopathy with reduced vision but without maculopathy
to determine cause of visual loss.

Slide 2
Ans 1. Fig. 1. Normal kidney.
Fig. 2. Kimmelstiel-Wilson lesions/nodules.

Ans 2. Urine analysis for detection of microalbuminuria. Microalbuminuria is


defined as albumin excretion of more than 20 mg/min. This phase
indicates incipient diabetic nephropathy and calls for aggressive
management, at which stage the disease may be potentially reversible.

Ans 3. ACE inhibitors/ARB.

159
Slide Quitz Answers

Slide 3
Ans 1. Acanthosis nigricans.

Ans 2. l Type A syndrome – HAIRAN (hyperandrogenemia, insulin resistance


and AN). It is associated with polycystic ovaries or signs of virilization
(e.g., hirsutism, clitoral hypertrophy). High plasma testosterone levels
are common.
l Type B syndrome generally occurs in women who have uncontrolled
diabetes mellitus, ovarian hyperandrogenism, or autoimmune diseases
such as systemic lupus erythematosus, scleroderma, Sjögren syndrome
or Hashimoto thyroiditis. Circulating antibodies to the insulin receptor
may be present.

Slide 4
Ans 1. Abdominal obesity (central/truncal/android) in a man.

Ans 2. Truncal obesity is closely associated with insulin resistance, glucose


intolerance, hypertension, dyslipidemia and atheromatous disease
(‘syndrome X’).

Slide 5

Ans 1. The CT scan shows increased subcutaneous adipose tissue which is


accompanied by intra-abdominal fat deposition.

Slide 6
Ans 1. The ‘prayer’ sign.
Ans 2. In normal people, finger tips and the palmar surfaces of fingers and
palms can be perfectly apposed but in patients with limited joint mobility,
this is impossible. Flexion contractures at the metacarpophalangeal and
proximal interphalangeal joints are shown. There is swelling of little finger
proximal interphalangeal joints associated with patient’s inability to
extend his wrists fully.

160
Management of Type 2 Diabetes

Slide 7
Ans 1. T o p : A m a c r o s o m i c b a b y b o r n t o a d i a b e t i c m o t h e r .
Bottom: A normal baby born to a non-diabetic mother.
Ans 2. The most common problem seen in the infants is macrosomia (8-50%),
which can result in birth trauma and an increased intervention rate.

Ans 3. Fetal hyperinsulinemia causes obesity, accelerates skeletal maturation,


delays pulmonary maturity and causes increased growth of insulin
sensitive tissues giving hypertrophy of the heart and liver.

Slide 8
Ans 1. Fourniers gangrene, initially described by Jean Alfred Fournier, a Parisian
dermatologist and venereologist. The cardinal points of that description
included: 1) Sudden onset in a healthy young man, 2) rapid progression
to gangrene and 3) absence of a definite cause.

Ans 2. There is an infective process leading to thrombosis of subcutaneous


and cutaneous blood vessels including the perforating vessels. This
results in gangrene of overlying skin. It is more common in diabetics
and in immunosuppressed people.

Slide 9
Ans 1. Diabetic foot showing dry gangrene.

Ans 2. Peripheral neuropathy, autonomic neuropathy, peripheral vascular disease


and previous ulceration are the various risk factors that make a person
with diabetes more prone to foot problems.

Slide 10

Ans 1. Tuberculosis.
Ans 2. Insulin would be the best option.

161
Inside Back Cover
Back Cover

You might also like